Download as pdf or txt
Download as pdf or txt
You are on page 1of 303

BRITISH ACADEMY OET 2.

READING TEST 1
READING SUB-TEST : PART A
 Look at the four texts, A-D, in the separate Text Booklet.

 For each question, 1-20, look through the texts, A-D, to find the relevant information.

 Write your answers on the spaces provided in this Question Paper.


 Answer all the questions within the 15-minute time limit.
 Your answers should be correctly spelt.

TEXT BOOKLET - CANNABIS USE DISORDER

Text A

Cannabis use disorder

Cannabis use disorder is a problematic pattern of cannabis use which leads to


impaired control over cannabis use and difficulty in ceasing use despite its harm.
Drug abuse is a global phenomenon affecting almost every country, with cigarettes,
cannabis, and alcohol being the most commonly used and abused substances.
Among them cannabis is illegal. As compared to other psychoactive substances cannabis,
most commonly known as marijuana, is the most widely used drug worldwide.
Even though there is no international consensus, young adulthood is a period where the
transition takes place from childhood to adulthood. Arnett et al. define this group as
“emerging adults” from 18 to 25 years, those who are neither children nor adults and who
are in between with their own identity and behavior. It is the age of instability, self-focus,
feeling in between, and possibilities. Young adults are the most vulnerable group of people
than any age group to cannabis dependence and related problems which produce more
years lived with disabilities. In 2013, cannabis was used by 80.6% of current illicit drug users
in the United States.
Text B

Globally 2% cause-specific disability-adjusted life years (DALYs) for young


people are attributed to illicit drug including cannabis. The study conducted
revealed that cannabis and other illegal drugs accounted for approximately $8.2
billion of the nearly $40 billion cost of substance abuse in Canada in 2002. The
probability of cannabis addiction in heavy or daily user is enormous. The
vulnerability increases in adolescents whose risk of addiction is 16%, while adults
have 5–10% risk of becoming addicted. Canadian community health survey
reported that the prevalence of cannabis dependence among adolescent and young
adult in 2012 was 5%. A three-year prospective study in the Netherlands on high risk young
adults reporting heavy use in 2013 found that nearly 40% developed
cannabis dependence.

Text C

Another study in Holland revealed that cannabis dependence was 42%. In 2013 the
prevalence of cannabis abuse or dependence was 7.4% among youth in the USA
and the rate was about half (3.55) among adolescents. National Survey on Drug
Use and Health (NSDUH) revealed that cannabis was the illicit drug with the
largest number of persons with past-year dependence or abuse in 2013. Of the 6.9
million persons aged 12 or older who were classified with illicit drug dependence
or abuse in 2013, 4.2 million persons had cannabis dependence or abuse. Another
study conducted in the USA reported that 38.5% of daily cannabis users met
criteria for cannabis dependence. A longitudinal cohort study conducted in
Australia in 2002 among young adults shows a 7% prevalence of cannabis
dependence according to DSM-IV criteria for cannabis dependence. A community
household-based survey with a cross-sectional design in Rwanda that aimed to
determine the prevalence of cannabis dependence among adolescent and young
adults shows 2.54% prevalence of cannabis dependence.
Text D

A recently published (2015) cohort study which considered cannabis abuse and
dependence as cannabis use disorder (CUD) showed the prevalence of cannabis
use disorder throughout the life to be 19.1%, with an average age of onset of 18.6 years.
Cannabis availability, regular use of cannabis, peer pressure, and
common mental disorder were factors having a significant association with
cannabis use disorder in different studies. Gateway hypothesis developed by
Kandel explained that the sequence of drug use occurring starts with legal drug and
proceeds to illegal drugs. Above all, Shashemene is a town in which Rastafarians view
Ethiopia as a promised land live. Cannabis use is a commonpractice among Rastafarians
which brought a major challenge to both youth and law enforcement in the town.
PART A -QUESTIONS AND ANSWER SHEET
Questions 1-7
For each question, 1-7, decide which text (A, B, C or D) the information comes from.
You may use any letter more than once.
In which text can you find information about;
1.increase of the drug influence. Answer ____________
2.Use of the drug in the USA. Answer _____________
3.life time prevalence of cannabis use disorder. Answer ______________
4.use of cannabis despite clinically significant distress or impairment. Answer ________
5. common determinants of drug addiction. Answer ______________
6.addiction to drug common among young community. Answer ________________
7.Prevalence of the drug abuse, drug usage. Answer ________________
Questions 8-14
Answer each of the questions, 8-14, with a word or short phrase from one of the texts.
Each answer may include words, numbers or both. Your answers should be correctly spelt.
8. From how many places data on cannabis addiction has been developed? Answer ____
9. As per the report, how many people in Canada are reported to be cannabis
addict in the first quarter of the 2nd decade of 21st century? Answer _______________
10. How many people are known to be addicted to cannabis as per the report of 2013? Answer _
11. Who are likely to be addicted more commonly? Answer _____________
12. Who are popular for making use of cannabis as its use is customary? Answer ____
13. As per the report, how many people in the USA are reported to be cannabis addict? Answer __
14. What revealed a 7 percent of young adults were cannabis dependent in Australia during 2002?
Answer ______________
Questions 15-20
Complete each of the sentences, 15-20, with a word or short phrase from one of the texts.
Each answer may include words, numbers or both. Your answers should be correctly spelt.

15. In the United States, in 2013, the drug was used by___________ of people.

16. Commonly, adults may have about_________ risk of becoming a drug addict

17. The studies in the landlocked East African country shows __________ of cannabis dependence.

18. The sequence of the drug use may end with____________.

19. It is very much likely that____________ is of higher level among drug addicts

20. _____________ explained that the sequence of drug use occurring starts with legal drug and
proceeds to illegal drugs.

END OF PART A, THIS QUESTIONS PAPER WILL BE COLLECTED


READING SUB-TEST : PART B
In this part of the test, there are six short extracts relating to the work of health professionals .
For questions 1-6, choose the answer (A, B or C) which you think fits best according to the text. Write
your answers on the separate Answer Sheet
Questions 1-6

1. Chronic urticaria;
A. Is caused by an allergic reaction to a food or drug.
B. Arises spontaneously and its cause is unknown.
C. Is a genetic disorder that is very rare.

Chronic urticarial

Chronic urticaria is a common condition characterized by recurrent hives lasting


several weeks or months and is usually idiopathic. Approximately half of the
individuals with chronic urticaria will present with episodes of angioedema that
can be severe and debilitating. There is a 47-year-old Hispanic male who presented
initially for an evaluation of chronic hives following hospitalization due to hiveinduced
anaphylaxis. The individual had a history significant for urticaria and
angioedema beginning in his early 30s. Interestingly, both the individual’s 41-yearold
sister and 12-year-old daughter were also affected by chronic urticaria and
severe angioedema. Whole exome sequencing of the proband and several family
members revealed a heterozygous variant of uncertain significance in exon 2 of
TNFAIP3, denoted as c.65G>A (p.R22Q), in all affected members.
2. Arterial aneurysms;
A. Are caused by narrowing of arteries.
B. are abnormal dilations of the peripheral arteries.
C. Occur due to change in structure and function of the Fibrillin-1.

Arterial aneurysms

Arterial aneurysms are most commonly caused by atherosclerotic disease,


especially in elderly patients aged over 60. Other etiologies, such as connective tissue
disorders, should be investigated in younger patients.. Fibrillin-1 microfibrils, through
interactions with elastin and other proteins, provide structure to elastic and
nonelastic connective tissues. In addition to the architectural functions, Fibrillin-1
plays an important role in regulating TGF-β complexes in the extracellular matrix.
TGF-β signaling controls various processes at the cellular level, such as cellular
growth, differentiation, and apoptosis. When Fibrillin-1 is defective, it disrupts the
normal architecture of connective tissues.
3. What is correct?
A. PE occurs when a clump of material, most often a blood clot, gets wedged into
an artery in your lungs.
B. Patients with DVT are susceptible to PE.
C. PE symptoms, in most of the cases, do not get revealed easily.

Pulmonary embolism

Pulmonary embolism (PE) is regarded as an elusive diagnosis with a non-specific


clinical presentation and has a tendency to be both over- and underdiagnosed in
clinical practice. In the United States of America, venous thromboembolism (VTE)
has been reported as the 3rd commonest cause of mortality.
Most patients with PE are clinically asymptomatic. In fact, PE has been shown to be
present in 60–80% of individuals with confirmed deep vein thrombosis (DVT), despite
the absence of symptom in more than half of these patients. Cardiac arrest following
PE has an associated mortality of up to 70% within the first hour of presentation and
an overall mortality of up to 95%. Approximately 90% of episodes of cardiac arrests
occur within 1-2 hours after the onset of symptoms of PE.
4. What is the notice talking about?
A. Cleaning of the blood of toxins.
B. A major a superficial vein in the arm and disease associated with it.
C. A case report.

Clinically, the cephalic vein is preferred for haemodialysis in patients with chronic
renal failure (CRF), to remove waste products from the blood. The cut-down of the
cephalic vein in the deltopectoral groove is preferred when superior vena caval
infusion is necessary. However, cephalic veins exhibit a wide array of developmental
variations in terms of formation, course, and termination.
During routine gross anatomy dissection of the neck of the patient, a rare case of variation
of the termination of the cephalic vein in both right and left upper limbs have been
observed. Knowledge of the variations of cephalic vein is important not only for
anatomists but also for surgeons and clinicians as the vein is frequently used for
different surgical procedures and for obtaining peripheral venous access as well.
5. What does the table indicate?
A. Tenofovir goes well in coordination with Cycloferon.
B. Cycloferon goes well in coordination with Tenofovir.
C. Adefovir goes well in coordination with Cycloferon.

% of total % of dual Dual


Nb
treatment ( ) therapy ( ) therapies
Cycloferon + Adefovir 55 7.9% 43.3%
Cycloferon + Lamivudine
42 6.1% 33.0%

Adefovir + Lamivudine
9 1.3% 7.1%

Cycloferon + Tenofovir
5 0.7% 4.0%

Tenofovir + Adefovir
4 0.6% 3.2%

Tenofovir + Lamivudine
4 0.6% 3.2%

Pegasys + Tenofovir
3 0.4% 2.4%

Pegasys + Adefovir
2 0.3% 1.6%

Entecavir + Tenofovir
1 0.1% 0.8%

Interferon + Cycloferon
1 0.1% 0.8%

Entecavir + Pegasys
1 0.1% 0.8%

Total 126 18.2%


6. The products which are good moisturizers are marketed by;
A. NOW Solutions, Microfluidics, Decorte.
B. Microfluidics, Decorte, Dior.
C. Microfluidics, Decorte.

Marketed formulations of liposomes


Product name Marketed by Uses

Removes wrinkles and dark


Capture Totale Dior spots and has radiance
effect with sunscreen
Microfluidi
Dermosome Moisturizer
cs
Decorte Moisture Liposome
Decorte Moisturizer
Face Cream

Decorte Moisture Liposome Moisturizes, firms, and brightens


Decorte
Eye Cream the delicate skin around the eyes

Natural Progesterone
NOW Maintenance of healthy
Liposomal
Solutions feminine balance
Skin Cream
Hydration, boosts collagen
C-Vit
synthesis, enhances the skin’s
Liposomal Sesderma
elasticity and firmness, and
Serum
brightens the complexion

Advanced Night Repair Estée


Skin repair
Protective Recovery Complex Lauder
READING SUB-TEST : PART C
In this part of the test, there are two texts about different aspects of healthcare.
For questions 7-22, choose the answer (A, B, C or D) which you think fits best according to the text.
Write your answers on the separate Answer Sheet

Text 1: What is Creutzfeldt-Jakob Disease?

Creutzfeldt-Jakob disease is a degenerative brain disorder that leads to dementia


and, ultimately, death. Symptoms of Creutzfeldt-Jakob disease (CJD) sometimes
resemble those of other dementia-like brain disorders, such as Alzheimer`s, but
Creutzfeldt-Jakob disease usually progresses much more rapidly. Creutzfeldt Jakob
disease captured public attention in the 1990s when some people in the United
Kingdom developed a form of the disease — variant CJD (vCJD) — after eating meat
from diseased cattle. However, "classic" Creutzfeldt-Jakob disease has not been
linked to contaminated beef. Although serious, CJD is rare, and vCJD is the least
common form. Worldwide, there is an estimated one case of Creutzfeldt Jakob
disease diagnosed per million people each year, most frequently in older adults, if
not among children.

Creutzfeldt-Jakob disease is marked by rapid mental deterioration, usually within a


few months. Initial signs and symptoms of CJD typically include personality
changes, anxiety, depression, memory loss, impaired thinking, blurred vision,
insomnia, difficulty speaking, difficulty swallowing and sudden, jerky movements.
As the disease progresses, mental symptoms worsen; most people eventually lapse
into a coma. Heart failure, respiratory failure, pneumonia or other infections are
generally the cause of death. The disease usually runs its course in about seven
months, although a few people may live up to one or two years after diagnosis. In
people with the rarer CJD, psychiatric symptoms may be more prominent in the
beginning, with dementia — the loss of the ability to think, reason and remember -
developing later in the course of the illness. In addition, this variant affects people
at a younger age than classic CJD does, and appears to have the slightly longer
duration of 12 to 14 months.
Creutzfeldt-Jakob disease and its variants belong to a broad group of human and
animal diseases known as transmissible spongiform encephalopathies (TSEs). The
name derives from the spongy holes, visible under a microscope, that develops in
affected brain tissue. The cause of Creutzfeldt-Jakob disease and other TSEs
appears to be abnormal versions of a kind of protein called a prion. Normally,
these proteins are harmless, but when they`re misshapen they become infectious
and can wreak havoc on normal biological processes. The risk of CJD is low. The
disease can`t be transmitted through coughing or sneezing, touching, or sexual
contact. The three ways it develops are: Sporadically: most people with classic
CJD develop the disease for no apparent reason. CJD that occurs without
explanation is termed spontaneous CJD or sporadic CJD and accounts for the
majority of cases. By inheritance: in the United States, about 5 to 10 percent of
people with CJD have a family history of the disease or test positive for a genetic
mutation associated with CJD. This type is referred to as familial CJD. By
contamination: a small number of people have developed CJD after being exposed
to infected human tissue during a medical procedure, such as a cornea or skin
transplant. Also, because standard sterilization methods do not destroy abnormal
prions, a few people have developed CJD after undergoing brain surgery with
contaminated instruments. Cases of CJD related to medical procedures are referred
to as iatrogenic CJD. Variant CJD is linked primarily to eating beef infected with
bovine spongiform encephalopathy (BSE), the medical term for mad cow disease.

Most cases of Creutzfeldt-Jakob disease occur for unknown reasons, and no risk
factors can be identified. However, a few factors seem to be associated with
different kinds of CJD: Age: sporadic CJD tends to develop later in life, usually
around the age of 60. The onset of familial CJD occurs only slightly earlier. On the
other hand, vCJD has affected people at a much younger age, usually in their late
20s. Genetics: people with familial CJD have a genetic mutation that causes the
disease. The disease is inherited in an autosomal dominant fashion, which means
you need to inherit only one copy of the mutated gene, from either parent, to
develop the disease. If you have the mutation, the chance of passing it on to your
children is 50 percent. Genetic analysis in people with iatrogenic and variant CJD
suggests that inheriting identical copies of certain variants of the prion gene may
predispose a person to develop CJD if exposed to contaminated tissue. Exposure to
contaminated tissue: people who`ve received HGH derived from human pituitary
glands or who`ve had dura mater grafts may be at risk of iatrogenic CJD. The risk
of contracting vCJD from eating contaminated beef is difficult to determine. In
general, if countries are effectively implementing public health measures, the risk
is virtually non-existent.

Only a brain biopsy or an examination of brain tissue after death (autopsy) can
confirm the presence of Creutzfeldt-Jakob disease. But doctors can often make an
accurate diagnosis based on your medical and personal history, a neurological
exam, and certain diagnostic tests. The exam is likely to reveal such characteristic
symptoms as muscle twitching and spasms, abnormal reflexes, and coordination
problems. People with CJD may also have areas of blindness and changes in
visual-spatial perception. In addition, doctors commonly use the following tests to
help detect CJD: Electroencephalogram (EEG): using electrodes placed on your
scalp, this test measures your brain`s electrical activity. People with CJD and vCJD
show a characteristically abnormal pattern. Magnetic resonance imaging (MRI):
this technique uses radio waves and a magnetic field to create cross-sectional
images of your head and body. It`s especially useful in diagnosing brain disorders
because of its high-resolution images of the brain`s white matter and gray matter.
Spinal fluid tests: cerebrospinal fluid surrounds and cushions your brain and spinal
cord. In a test called a lumbar puncture — popularly known as a spinal tap -
doctors use a needle to withdraw a small amount of this fluid for testing. The
presence of a particular protein in spinal fluid is often an indication of CJD or vCJD.
No effective treatment exists for Creutzfeldt-Jakob disease or any of its variants. A
number of drugs have been tested - including steroids, antibiotics and antiviral
agents - and have not shown benefits. For that reason, doctors focus on alleviating
pain and other symptoms and on making people with these diseases as comfortable
as possible.
Text 1 : Questions 7-14
7. Which disease progresses faster?
A. Alzheimers
B. Jakob
C. Both Alzheimer`s and Jakob
D. Not given

8. Creutzfeldt-Jakob disease is commonly found among adults or elderly people;


A. False
B. True
C. False, because it is found among children too.
D. Not given

9. People affected with the disease may die after;


A. One year
B. Two years
C. 12-14 months
D Not given

10. Paragraph 3 talks more about;


A. Symptoms
B. Occurrence
C. Transmission
D. Prevention

11. One of the most common risk factors includes;


A. Exposure to contaminated tissue.
B. Age
C. Genetics
D. B and C
12. People who may have the human growth hormone derived from human pituitary glands
may be at risk of iatrogenic CJD.
A. 100% true
B. 100% false
C. 50% true
D. 50% false

13. Confirmation of the Creutzfeldt-Jakob disease can be done only after the death of the person.
A. True
B. False
C. True in some cases
D. Not given

14. What is most helpful in detecting CJD?


A. Electroencephalogram (EEG)
B. Spinal fluid exams
C. MRI
D. All of the above
Text 2: Heat Rash

The skin`s job is to protect the inside of the body from the outside world. It acts as
a preventive barrier against intruders that cause infection, chemicals, or ultraviolet
light from invading or damaging the body. It also plays an important role in the
body`s temperature control. One way that the body cools itself is by sweating, and
allowing that sweat or perspiration to evaporate. Sweat is manufactured in sweat
glands that line the entire body (except for a few small spots like fingernails,
toenails, and the ear canal). Sweat glands are located in the dermis or deep layer of
the skin, and are regulated by the temperature control centers in the brain. Sweat
from the gland gets to the surface of the skin via a duct. A heat rash occurs when
sweat ducts become clogged and the sweat can`t get to the surface of the skin.
Instead, it becomes trapped beneath the skin`s surface causing a mild inflammation
or rash. Heat rash is also called prickly heat or miliaria.

It is uncertain why some people get heat rashes and others don`t. The sweat gland
ducts can become blocked if excessive sweating occurs, and that sweat is not
allowed to evaporate from a specific area. Some examples of how blockage may
occur include the following: creases in the skin, for example the neck, armpit, or
groin which have skin touching adjacent skin, making it difficult for air to
circulate, therefore preventing sweat evaporation; tight clothing that prevents
sweat evaporation; bundling up in heavy clothing or sheets - this may occur when a
person tries to keep warm in the winter or when chilled because of an illness with
fever. Heavy creams or lotions can also clog sweat ducts. Babies have immature
sweat glands that aren`t able to remove the sweat they produce; they can develop
heat rash if they are exposed to warm weather, are overdressed, excessively
bundled, or have a fever. Heat rash may occur as a side effect of some medications,
for example, isotretinoin (Accutane) or clonidine (Catapres)
The most common symptoms of heat rash are red bumps on the skin, and an itchy
or prickly feeling to the skin. These are due to inflammation of the superficial
layers of the skin (the epidermis) and the prickly sensation is similar to the feeling
of mild sunburn. The symptoms of heat rash are the same in infants and adults;
however, since an infant can`t complain about the rash sensation, he or she may be
fussy. Newborns, infants, the elderly, and obese individuals with large areas with
skin-on-skin contact areas (for example, a large overlapping area of abdominal fat
or panniculus) are at risk of developing a heat rash. They are all especially at risk if
they are immobile for long periods of time and parts of the skin aren`t exposed to
circulating air, which results in the inability of the sweat ducts to "breathe"
(evaporative cooling). Heat rashes are more common in places with hot, humid,
climates because people sweat more. Intense exercise associated with lots of
sweating may cause a heat rash, especially if the clothing worn does not allow
adequate air circulation.

The appearance of a heat rash depends upon where the excess sweat gets
deposited in the skin. Tiny blisters that look like small beads of sweat are seen if the
sweat is blocked at the most superficial layers of the skin where the sweat duct
opens on the skin surface. Called miliaria crystallina, it has no symptoms other than
these "sweat bubbles." Classic heat rash or miliaria rubra occurs if the sweat causes
inflammation in the deeper layers of the epidermis. Like any other inflammation,
the area becomes red and the blisters become slightly larger. Because the sweat
glands are blocked and don`t deliver sweat to the skin`s surface, the area involved
is dry and can be irritated, itchy, and sore. This rash is also called prickly heat. Less
frequently, after repeated episodes of prickly heat, the heat rash may inflame the
deeper layer of the skin called the dermis, and cause miliaria profunda. This rash is
made up of larger, harder bumps that are more skin colored. The rash begins almost
immediately after exercise, and again no sweat can be found on the affected areas.
Rarely, this type of heat rash may potentially be dangerous if enough skin is
involved, since the lack of sweating can lead to heat-related illnesses like heat
cramps, heat exhaustion, or heat stroke.
Heat rash or prickly heat is detected by physical examination. Knowing that the
rash appears during sweating or heat, appreciating the location on the body (in skin
creases or where clothes fit tightly) and seeing what the rash looks like is enough
to make the diagnosis. As with many rashes, the health care professional may look
at the involved skin and, because of previous experience, immediately make the
diagnosis. An effective recovery process may depend more on treating heat rash
with remedies such as over-the-counter creams and sprays. Medical treatment for
heat rash may involve antibiotics if the sweat glands become infected.
Text 2 : Questions 15-22
15. Heat rash develops when;
A. Sweat ducts become clogged
B. Sweat can`t come out onto the skin
C. Skin stops developing sweat
D. None

16. One of the most common reasons given for the blockage of the sweat glands is;
A. Excessive sweat is not allowed to evaporate from the skin
B. Creases in the skin which makes circulation difficult
C. Tight clothing
D. Heavy creams and lotions

17. In babies, heat rash often develops due to;


A. Warm weather
B. Overdressing
C. Fever
D. Not given

18. Paragraph 3 talks more about;


A. Risk factors
B. Who is at risk
C. Symptoms
D. Development of the disease

19. Heat rash is common in;


A. Cold areas
B. Areas with higher humidity
C. Desert areas
D. B and C
20. What is the central idea of paragraph 4?
A. Heat rash symptoms.
B. Mode of occurrence.
C. What heat rash looks like.
D. B & C

21. Heat rash may cause miliaria profunda.


A. True in some cases
B. False
C. 100% true
D. Not given

22. Medical treatment for heat rash is effective when the;


A. Blockage is high.
B. Blockage is mild.
C. Blockage is low.
D. Home remedies are ineffective.

END OF READING TEST, THIS BOOKLET WILL BE COLLECTED


Reading test 1 : Answer Key

Part A - Answer key 1 – 7


1: B
2: C
3: D
4: A
5: D
6: A
7: C

Part A - Answer key 8 – 14


8: Six 9
Is: 5%
10: 4.2 Million
11: Young people
12: Rastafarians
13: 7.4%
14: longitudinal cohort study

Part A - Answer key 15 – 20


15: 80.6%
16: 5–10%
17: 2.54% prevalence
18: illegal drugs
19: cannabis addiction
20: Gateway hypothesis
Reading test - part B – answer key
1: B Arises spontaneously and its cause is unknown.
2: A Are caused by narrowing of arteries.
3: B Patients with DVT are susceptible to PE.
4: A Cleaning of the blood of toxins.
5: C Adefovir goes well in coordination with Cycloferon.
6: C Microfluidics, Decorte.

Reading test - part C – answer key

Text 1 - Answer key 7 – 14


7: B Jakob
8: B True
9: D Not given
10: C Transmission
11: D B and C
12: A 100% true
13: C True in some cases
14: D All of the above

Text 2 - Answer key 15 – 22


15: A Sweat ducts become clogged
16: A Excessive sweat is not allowed to evaporate from the skin
17: B Overdressing
18: B Who is at risk
19: B Areas with higher humidity
20: C What heat rash looks like.
21: A True in some cases
22: A Blockage is high.
READING TEST 2
READING SUB-TEST : PART A
 Look at the four texts, A-D, in the separate Text Booklet.
 For each question, 1-20, look through the texts, A-D, to find the relevant information.
 Write your answers on the spaces provided in this Question Paper.
 Answer all the questions within the 15-minute time limit.
 Your answers should be correctly spelt.
TEXT BOOKLET - DNA SEQUENCING TECHNOLOGIES

TEXT A

DNA Sequencing Technologies


Past efforts at sequencing genes were painstaking, time consuming, and labor intensive,
such as when Gilbert and Maxam reported the sequence of 24 base pairs using a
method known as wandering-spot analysis. Thankfully, this situation began to change
during the mid-1970s, when researcher Frederick Sanger developed several faster, more
efficient techniques to sequence DNA. Indeed, Sanger's work in this area was so ground
breaking that it led to his receipt of the Nobel Prize in 1980.
Over the next several decades, technical advances automated, dramatically sped up,
and further refined the Sanger sequencing process. Also called the chain-termination or
dideoxy method, Sanger sequencing involves using a purified DNA polymerase enzyme
to synthesize DNA chains of varying lengths. The key feature of the Sanger method
reaction mixture is the inclusion of dideoxynucleotide triphosphates (ddNTPs). These
chain-terminating dideoxynucleotides lack the 3' hydroxyl (OH) group needed to form
the phosphodiester bond between one nucleotide and the next during DNA strand
elongation. Thus, when a dideoxynucleotide is incorporated into the growing strand, it
inhibits further strand extension. The result of many of these reactions is a number of
DNA fragments of varying length. These fragments are then separated by size using gel
or capillary tube electrophoresis. This procedure is sensitive enough to distinguish DNA
fragments that differ in size by only a single nucleotide.
Text B

In the modern era, whole world has experienced tremendous boost in the field of
molecular diagnosis by use of DNA sequencing technology. The human genome contains
more than 3 billion base pairs that contain all the information about our health and
wellbeing.
The first whole genome sequence of human was published successfully before
decades. It was very expensive and paid billion dollars to complete. The cost paid for,
was much more worthy as it was providing the first fundamental understanding of the
structure and biology of human genome and relation with diseases. Initially, Next
Generation Sequencing technology was generated huge amount data of human variant
later it was proved that this NGS technology is much more effective in the diagnosis
human diseases by use of bioinformatic tools to select pathogenic variants. In present
days, sequencing costs have dramatically declined and therefore it is now routinely using for
diagnosis of many rare inherited diseases including hematology and blood disorder.
Genome Wide sequence analysis is already playing an important role in the hematology
field. This new sequencing technology is going to solve the challenges that researchers in the
field of hematology are going forward.
Text C

Nowadays researchers are making disease-specific targeted NGS panel, which is helping
more quickly and precise diagnosis of specific disease in the field of hematology.
Keeping in mind the growing research in the area of molecular diagnosis, how genome
wide analysis has unlocked new avenues of research, diagnosis, and therapy for benign
hematologic disorders. Recent advances in molecular technologies, mainly next
generation sequencing, inspire us to apply these technologies as a first-line approach for
the identification of potential mutations and to determine the novel causative genes in
patients with blood disorders. Researcher have started preparing targeted NGS panels
for diagnosis of hematologic malignancies, Red cell congenital hemolytic anemia for
diagnosis of all rare cause of haemolytic anemia which covers around 70-80 genes
associated with hemoglobinopathies, which will cover gene related alpha (HBA1/2) and
beta (HBB) globin gene locus analysis, HBD sequence analysis, gene related to RBC
membrane protein disorders, RBC enzymopathies genes, congenital dyserythropoietic
anemia (CDA) and the inherited bone marrow failure syndromes (IBMFS) are a group of
rare genetic blood disorders in which there is usually some form of aplastic anemia
associated with a family history of the similar disorder.
Text D

A panel of genes already identified by WES and association studies as responsible of


CHA or modulators of the clinical course of the disease is already analyzed by mass
sequencing methodology (NGS) including between 60-70 genes. Modifiers, related to
sickle-cell formation, vascular adhesion to endothelium, tolerance to hemolysis and
acute severe events, and the analysis of the RBC glycolytic enzymes are included. The
variants obtained were studied by mapping in the GRCh38/hg38 version of the human
reference genome. For the prioritization of variants, filters related to pathogenicity and
population frequency according to the SnpEff v4.1 and Mutation Taster programs
generally used for analysis. Some are previously used clinical databases of hemolytic
anemia are Human Gene Mutation Database version Professional, ClinVar, Red Cell
Membrane Database Mutations Database, Leiden Open Variation Database ± PKLR. The
allele frequency is generally assessed in the population (1000G and ExAC) and in the
local database. Finally, most of the researchers generally used the ‘in silico’ predictions
of pathogenicity and Sorting Intolerant from Tolerant (SIFT), PolyPhen-2, Mutation
Taster, and Mutation Assessor
PART A -QUESTIONS AND ANSWER SHEET

Questions 1-7
For each question, 1-7, decide which text (A, B, C or D) the information comes from.
You may use any letter more than once.

In which text can you find information about;


1. Modern technologies of DNA sequencing Answer ___________
2. Determination of the precise sequence of nucleotides in a sample of DNA. Answer ___________
3. A source of inspiration Answer _____________
4. Early DNA sequencing technologies Answer ______________
5. Cost-effective solutions in sequencing Answer ____________
6. Clinical databases Answer ____________
7. Detection of the disease-causing genes Answer _____________

Questions 8-14

Answer each of the questions, 8-14, with a word or short phrase from one of the texts.
Each answer may include words, numbers or both. Your answers should be correctly spelt.
8. What is generally analyzed in a large population? Answer ____________
9. What is the term which defines a method in which an electric field pulls molecules across a gel substrate
or hairlike capillary fiber? Answer ____________
10. Which term may mean "performed on the computer or via computer simulation? Answer ___
11. What is known to make up the backbone of the strands of nucleic acid? Answer ____
12. What is characterized by failure of the bone marrow to produce blood? Answer ____________
13. Which method of sequencing usually comprises use of polymerase enzyme for the purpose of building
different types of chains of varied lengths? Answer ____________
14. Which sequencing technology is regarded to be more result-oriented? Answer ____________
Questions 15-20

Complete each of the sentences, 15-20, with a word or short phrase from one of the texts.
Each answer may include words, numbers or both. Your answers should be correctly spelt.

15. Red blood cell ____________ affect genes encoding red blood cell enzymes..

16. Today, researchers are busy developed targeted NGS panels which can effectively be used for the
purpose of diagnosis of_____________.

17. The variants obtained were studied by mapping in the______ of the human reference genome.

18. The key feature of the Sanger method reaction mixture is the inclusion of____________.

19.______________ analysis is effective and can solve challenges which researchers grappling with

20. NGS is considered the _____________ for detection of the disease.

END OF PART A, THIS QUESTIONS PAPER WILL BE COLLECTED


READING SUB-TEST : PART B
In this part of the test, there are six short extracts relating to the work of health professionals .
For questions 1-6, choose the answer (A, B or C) which you think fits best according to the text. Write
your answers on the separate Answer Sheet.

Questions 1-6

1. What does the report indicate?


A. There has been a recent increase in the incidence of myositis worldwide.
B. Infection of the skeletal muscles.
C. Most common in tropical areas but can also occur in temperate zones.

Primary infection of muscle

Primary infection of muscle is usually regarded as a tropical disease, and is rare although
becoming more common in temperate climates. A review of the 230 cases of primary
obturator myositis which have been reported, shows that 82% were in children under 18
years of age. The median age was 9.5 years (3 to 46), with a male:female ratio of
approximately 3:2. The median duration of symptoms prior to presentation was three
days, and the most common symptoms were fever, hip or thigh pain, and inability to fully
bear weight. In 41% there was a history of recent trauma to the hip, such as a fall or
strenuous exercise. Local trauma is a recognised initiating factor for pyomyositis and is
documented in between 21% and 66% of cases.
2. According to the notice given, what is correct?
A. Treatment cost has not gone down yet.
B. CAR-T is considered more effective.
C. After chemotherapy, CAR-T is more reliable.

Food and Drug Administration (FDA) Notice

In the year 2017, Food and Drug Administration (FDA) announced the first approval of a
CAR-T cell therapy for kids and young adults with B-cell ALL. This approval was much
celebrated and brought new hope for a more specific and efficient therapy for ALL. This
result has come through many years of research and is expected to improve the quality of
treatment of patients. It is worth noting that in addition to side effects, the high cost of
treatment is still an obstacle and the side effects. Although there are challenges to be
overcome as in any innovative research, CAR-T cell therapy seems to be the most
promising therapeutic tool against cancer, including ALL, since chemotherapy
introduction in the 1940s.
3. Huntington`s disease;
A. may result in the death of brain cells.
B. may result in loss of intelligence.
C. may result in involuntary movements.

Huntington’s disease (HD)

Huntington’s disease (HD) is a genetic neurodegenerative disorder that results in chorea,


balance and gait impairments, changes in behavior and declines in cognition. Cognitive
decline occurs early in the disease course, occasionally preceding chorea and motor
impairment, and continues to progress throughout the disease process. Cognitive deficits
include difficulty with executive function including trouble with planning and organizing,
problems with working, visual and verbal memory, and impaired concentration. While
gait dysfunction is typically thought to arise primarily from damage to the motor circuitry
of the basal ganglia, studies in elderly populations and other neurologic populations
indicate that gait dysfunction may also be related to changes in cognitive function.
4. What is correct about Hypertension?
A. For subjects with normal or subnormal hypertension,it can prevent or delay hypertension.
B. BP can be well tackled with the use of BP specific medicine.
C. Right changes in lifestyle can curtail down BP values more effectively than various other
medical therapies.

Hypertension - Prevention or Treatment

Adequate changes in lifestyle are the cornerstone for the prevention and treatment of
hypertension. Although rapid medical initiation is necessary for the patients in a high
level of risk, lifestyle changes are fundamental for the therapy. According to the previous
report, lowering effects for stable blood pressure can be equivalent to monotherapy of
medicine. Contrarily, the weak point would be the low level of compliance or adherence
associated with necessary time for adequate action. Adequate changes in lifestyle would
be effective for some group of subjects. For grade 1 hypertensive patients, it can prevent
or delay medical therapy. Moreover, for hypertensive patients continuing on medical
therapy, it can contribute to BP reduction of blood pressure and allow reduction of the
number and doses of antihypertensive agents. Appropriate changes in lifestyle would
decrease other cardiovascular risk factors and improve several clinical conditions.
5. What does the table indicate?
A. Major significant differences were observed in 24-h, daytime and nighttime SBP or DBP
when using the conventional or custom-made pillow.
B. No significant differences were observed in 24-h, daytime and nighttime SBP or DBP
when using the conventional or custom-made pillow.
C. There are significant changes in Systolic Blood Pressure and Diastolic Blood Pressure.

Comparison of 24-h blood pressure monitoring between conventional and custom-made


pillows:
6. What does the report indicate?
A. Jugular venous engorgement and diffuse pulmonary rales were not found.
B. The electrocardiogram (ECG) showed broad R waves.
C. The patient was dyspnoeic with SaO2 74%.

Events
Initial presentation at the emergency room …………..
 3-h chest pain
 Respiratory failure and the need for advanced airway support
 Electrocardiogram with broad R waves, 4 mm ST-segment downsloping in right
precordial leads, right bundle branch block (RBBB), and ST-segment elevation in
posterior leads
 Coronary angiography with total thrombotic occlusion in the proximal segment of the
circumflex artery
 Bare metal stent was placed
 The patient developed cardiogenic shock and intra-aortic balloon pump was placed;
norepinephrine, vasopressin, and dobutamine were administered with
clinical improvement

48 h post-arrival
 Electrocardiogram with disturbance of repolarization only attributable to RBBB
 The patient developed acute renal failure and haemodialysis was initiated
 After 5 days.
 During a haemodialysis session, he developed sustained ventricular tachycardia
 with degeneration in asystole
 CPR was initiated, there wasn’t a return of spontaneous circulation
 Patient decease
READING SUB-TEST : PART C
In this part of the test, there are two texts about different aspects of healthcare.
For questions 7-22, choose the answer (A, B, C or D) which you think fits best according to the text.
Write your answers on the separate Answer Sheet

Text 1: Renal Artery Stenosis

Renal artery stenosis (narrowing) is a decrease in the diameter of the renal arteries. The
resulting restriction of blood flow to the kidneys may lead to impaired kidney function
(renal failure) and high blood pressure (hypertension), referred to as renovascular
hypertension, or RVHT ("reno" for kidney and "vascular" for blood vessel).
Renovascular hypertension is as likely to occur with bilateral stenosis (when arteries to
both kidneys are narrowed) as with unilateral stenosis (when the artery to one kidney is
narrowed). The decreased blood flow to the kidneys impairs renal function. Renal artery
stenosis may cause renal failure in some patients. There is no predictable relationship
between renal failure and renal artery stenosis. Some patients have very severe bilateral
stenosis and normal renal function. Most cases of renal failure are related to diabetes,
hypertension, glomerular sclerosis, contrast nephropathy, drug toxicity and other causes

The majority of renal artery stenosis is caused by atherosclerosis (hardening and


narrowing of blood vessel wall from the inside) similar to the process that occurs in blood
vessels in the heart and other parts of the body. Risk factors for atherosclerosis include
high cholesterol levels, high blood pressure, age, cigarette smoking, diabetes etc. Less
common causes of renal artery stenosis are fibromuscular dysplasia of the vessels
(narrowing of the vessel due to internal thickening of the blood vessel wall), arteritis
(inflammation of the blood vessel), or dissection (tearing and division of the blood vessel
wall).
Narrowing of the kidney arteries is more common in individuals 50 years of age and
older. It is estimated that some degree of narrowing (greater than 50%) is found in about
18% of adults between 65-75 years of age and 42% of those older than 75 years of age.
This may be due to the fact that atherosclerosis is more common in this age group. In
younger patients, the narrowing of the renal artery is usually due to the thickening of the
artery (fibromuscular dysplasia) and it is more common in women than in men. It is
estimated that renal artery stenosis accounts for approximately 1% of mild to moderate
cases of high blood pressure. It may be responsible for more than 10% of cases of
severely elevated or difficult to treat high blood pressure (hypertension).

In general, renal artery stenosis is not associated with any obvious or specific symptoms.
Suspicious signs for renal artery stenosis include high blood pressure that responds
poorly to treatment; severe high blood pressure that develops prior to age 30 or greater
than age 50; an incidental finding (discovered through routine tests or tests performed for
another condition) of one small kidney compared to a normal sized one on the other side.
Typically, unilateral (one-sided) renal artery stenosis may be related to high blood
pressure whereas bilateral (two-sided) renal artery stenosis is more often related to
diminished kidney function.

Several tests exist to detect any evidence of renal artery stenosis, which can be divided
into imaging tests and functional tests. The imaging tests provide a picture of the blood
vessel and its anatomy and reveal the degree of narrowing. The functional tests provide
information about whether the narrowing is significant enough to cause the high blood
pressure or kidney dysfunction. Each of these tests has advantages and disadvantages.
In bilateral (both-sided) and unilateral (one-sided) renal artery stenosis associated with
high blood pressure, controlling the blood pressure with usual blood pressure medications
is the first and the safest treatment. ACE inhibitors or ARB medications with or without a
diuretic (water pill) may be tried first. In some patients, this approach may be associated
with worsening of their kidney function. Therefore, kidney function needs to be followed
closely and if worsening of kidney function is evident, these medications may need to be
stopped. It is worth noting that if renal artery stenosis is found incidentally when
performing a test for another disease and there is no evidence of kidney dysfunction or
high blood pressure then no treatment may be necessary. Sometimes even significant
stenosis may not be associated with high blood pressure or kidney dysfunction. In these
situations, periodic monitoring of blood pressure and kidney function may be advised.
Text 1: Questions 7-14

7. Renovascular hypertension is likely to occur with;


A. Bilateral stenosis.
B. Unilateral stenosis.
C. Both bilateral and unilateral stenosis.
D. When arteries to one or both kidneys are narrowed.

8. Which one of these statements is true, according to paragraph 1?


A. The increase or decrease in blood flow leads to improper functioning of the kidneys
B. Renal failure and renal artery stenosis are closely connected to each other
C. A patient may have normal renal function even if there is higher bilateral stenosis
D. In some cases, bilateral stenosis may affect renal functions too.

9. Renal Artery Stenosis is caused by;


A. Hardening of the blood vessel wall from inside.
B. Hardening of the blood vessels similar to that of the blood vessels in the heart.
C. Narrowing and hardening of the walls like that of blood vessels in the heart. D
Hardening and narrowing of the blood vessels from inside.

10. Thickening of the arteries is more common among;


A. Men
B. Women
C. Children
D. All of the above

11. According to paragraph 4, which one of the following statements is true?


A. There are no specific symptoms of the renal artery stenosis.
B. Higher BP which develops before the age of 30 or after the age of 50 can become the cause of the
renal artery stenosis.
C. Untreatable high BP can be the cause of the renal artery stenosis.
D. Differences in sizes of the kidneys can be a major cause of the renal artery stenosis.
12. Functional tests provide;
A. A clear picture of the blood vessels and functions.
B. A clear idea of whether narrowing is significant to cause high BP or kidney dysfunction.
C. Anatomy of blood vessels.
D. A clear idea of the thickening of the blood vessels.

13. The best possible treatment for renal artery stenosis is;
A. Controlling the blood pressure.
B. Using ARB medications for the quality functioning of the kidneys.
C. Improving the function of the kidney through proper medications.
D. None

14. Renal artery stenosis is closely associated with;


A. Kidney dysfunction.
B. High blood pressure.
C. Low blood pressure.
D. A & B
Text 2: Hematochezia

Rectal bleeding (hematochezia) is used to describe the presence of blood with a bowel
movement. That blood, whether it fills the toilet bowl, or is a streak on the toilet paper
when wiping, or just a few drops in the toilet bowl, is not a normal finding and should not
be ignored. The source of bleeding can be anywhere in the digestive tract, from the nose
and mouth to the rectum and anus. The color can range from bright red to maroon to
black or any shade in between, depending on how much the blood has been exposed to the
digestive juices. Anytime there is blood within the gastrointestinal system, it will
eventually be excreted in stool (feces, bowel movement, BM). The color of the stool will
depend upon the amount of blood, the source of the bleeding and how quickly the stool
moves through the digestive tract. Sometimes, the bleeding is too little to be seen by the
naked eye but can be tested for by a healthcare professional.

Depending upon where and why the bleeding has taken place in the digestive tract, the
stool consistency and color may vary greatly: the stool color may be bright red, maroon,
dark red or black. The bleeding might be hidden, unseen to the naked eye, but able to be
detected by a fecal occult blood test. There may be blood just in the bowel movement or
there may be associated feces. If the feces are formed, the blood may be mixed in with the
stool or it may just coat the surface. The stool may be well-formed or it may lose and
diarrhea like. It may be normal in shape in size or it may become pencil thin. There may
be associated with abdominal pain or the bleeding may be painless.
Hemorrhoids are the most common cause of blood in the stool. Blood vessels located in
the walls of the rectum can swell, become inflamed and bleed. Hemorrhoids can be
caused by straining at stool, diarrhea, pregnancy, obesity and prolonged sitting on the
commode. All these factors increase the pressure within the hemorrhoidal vessels,
causing them to swell. The bleeding is often associated with anal burning or itching.
Bleeding can also occur because of an anal fissure, or a split in the skin of the anus. Hard
constipated stool may cause the skin to split; other causes include pregnancy and anal
intercourse. Anal fissures are also associated with other diseases including inflammatory
bowel disease (Crohn`s disease, ulcerative colitis), cancer and infections. Anal fissures
tend to be very painful, even when sitting. The blood in the stool can also be due to
swallowed blood from a nosebleed, dental work, or other mouth injuries that cause
bleeding.
Rectal bleeding is often diagnosed by history. The health care professional may ask
questions about the circumstances surrounding the rectal bleeding including the color, the
amount of bleeding, any associated symptoms and past medical history. A variety of
medications and food can mimic blood in the stool. Iron supplements and bismuth
(Pepto-Bismol, Kaopectate) can turn stool black, as can beets and licorice. Red food
coloring and beets can turn stool into a reddish hue. Patients who take blood thinners
(anticoagulation medications) are more prone to rectal bleeding. Examples of blood
thinners include warfarin (Coumadin), enoxaparin (Lovenox), aspirin and other
antiplatelet drugs including clopidogrel (Plavix), prasugrel and rivaroxaban (Xarelto).

Physical examination is important to assess the patient`s stability. Vital signs are
important and may include orthostatic vital signs, where the blood pressure and pulse rate
are taken both lying and standing. In a patient with reduced blood volume, the blood
pressure may fall, the pulse rate may rise, and the patient may become lightheaded and
weak when standing. Palpation of the abdomen is performed to look for tender areas,
masses or enlarged organs, especially the liver and spleen. Rectal examination is
performed by inserting a finger into the rectum, with the purpose of feeling for a mass or
other abnormality. The stool color and consistency may be examined when the finger is
withdrawn. The anus also may be examined. Blood tests may be considered if there is
concern about the amount of bleeding or other associated diseases. A complete blood
count (CBC) measures the number of red blood cells, white blood cells and platelets.
Blood clotting tests include PT (protime), INR (international normalized ratio) and PTT
(partial thromboplastin time). Depending upon the situation, other tests may be ordered to
measure electrolytes, and kidney and liver functions.

Most diseases which cause rectal bleeding are likely preventable, but it is not often
possible. Hemorrhoids can be avoided with proper diet and hydration to prevent
constipation and straining to pass stool, but normal pregnancy increases the risk of
hemorrhoid formation as does the patient with an acute diarrheal illness. Avoiding
constipation also decreases the risk of diverticulosis, outpouchings in the lining of the
colon, and the risk of a diverticular bleed but this may be a consequence of a Western
diet. Alcohol abuse increases the risk of rectal bleeding in a variety of ways, from
directly irritating the lining of the GI tract, to decreasing clotting capabilities of blood.
Text 2: Questions 15-22

15. Rectal bleeding describes;


A. Blood in the bowels.
B. Blood in the digestive tract.
C. Blood in the stools.
D. Blood in the rectum.

16. Paragraph 2 talks more about;


A. Which symptoms are associated with rectal bleeding.
B. Causes of the rectal bleeding.
C. Variations in the color of the stools.
D. None

17. Blood in the stool can originate anywhere in the gastrointestinal tract.
A. False
B. True
C. Not given
D. Sometimes true and sometimes false

18. Hemorrhoids can be well defined by which one of the following?


A. Swelling of the rectal walls.
B. Inflammation and bleeding of the rectal walls.
C. Straining
D. All of the above

19. Causes of anal fissure may include;


A. Formation of the constipated stool
B. Inflammatory bowel disease
C. Cancer and infections
D. All of the above
20. According to paragraph 4, which of the following statements is true?
A. Blood thinners can cause rectal bleeding.
B. Blood thinners may sometimes cause rectal bleeding.
C. It is not fully established that rectal bleeding is the direct result of the use of blood thinners.
D. None

21. Which one of the following statements is not included in paragraph 5?


A. Physical examiners will always look for orthostatic vital signs.
B. A patient may have low blood pressure with a higher pulse rate.
C. Palpitations of the abdomen is performed to look for tender areas.
D. Rectal bleeding is identified through taking a stool sample.

22. Which one of these is the preferred blood clotting test?


A. PT
B. INR
C. PTT
D. Not given

END OF READING TEST, THIS BOOKLET WILL BE COLLECTED


Reading test 2 : Answer Key

Part A - Answer key 1 – 7


1:B
2:A
3:C
4:A
5:B
6:D
7:C

Part A - Answer key 8 – 14


8:allele frequency
9: Electrophoresis
10: In silico
11: Phosphodiester bonds
12: Aplastic anaemia
13: Sanger sequencing
14: NGS

Part A - Answer key 15 – 20


15: enzymopathies
16: hematologic malignancies
17: GRCh38/hg38 version
18: dideoxynucleotide triphosphates
19: Genome Wide sequence
20: first-line approach
Reading test - part B – answer key

1: C Most common in tropical areas but can also occur in temperate zones.
2: B CAR-T is considered more effective.
3: C may result in involuntary movements.
4: A For subjects with normal or subnormal hypertension, it can prevent or delay hypertension.
5: A Major significant differences were observed in 24-h, daytime and
nighttime SBP or DBP when using the conventional or custom-made pillow.
6: B The electrocardiogram (ECG) showed broad R waves.

Reading test - part C – answer key

Text 1 - Answer key 7 – 14

7: A Bilateral stenosis.
8: C A patient may have normal renal function even if there is higher bilateral stenosis
9: D Hardening and narrowing of the blood vessels from inside.
10: B Women
11: C Untreatable high BP can be the cause of the renal artery stenosis.
12: B A clear idea of whether narrowing is significant to cause high BP or kidney dysfunction.
13: A Controlling the blood pressure.
14: D A & B

Text 2 - Answer key 15 – 22

15: C Blood in the stools.


16: A Which symptoms are associated with rectal bleeding.
17: B True
18: D All of the above
19: D All of the above
20: A Blood thinners can cause rectal bleeding.
21: D Rectal bleeding is identified through taking a stool sample.
22: D Not given
READING TEST 3
READING SUB-TEST : PART A

 Look at the four texts, A-D, in the separate Text Booklet.

 For each question, 1-20, look through the texts, A-D, to find the relevant information.

 Write your answers on the spaces provided in this Question Paper.


 Answer all the questions within the 15-minute time limit.
 Your answers should be correctly spelt.

TEXT BOOKLET- ANALYSES OF CANCER

Text A

As demonstrated by elegant analyses of cancer in various patients, the continued deletion of


cancer cells expressing T cell targets may enable cancers to evolve to avoid the attack. Despite
these findings, recent results from human cancer have demonstrated that overcoming
negative regulators to T cell responses in lymphoid organs and in the tumor bed is likely to
explain the failure of immune protection in many patients. Factors in the tumor
microenvironment can act to modulate the existing activated antitumor T cell immune
response, acting as an immune rheostat or “immunostat.” This class of molecules, including
PD-L1:PD-1 emphasizes that the immune response in cancer reflects a series of carefully
regulated events that may be optimally addressed not singly but as a group.
The most challenging part is now is to use this new understanding to develop new drugs and
implement clinical strategies.
Text B

For an anticancer immune response to lead to the effective killing of cancer cells, a series of
stepwise events must be initiated and allowed to proceed and expand iteratively. We refer to
these steps as the Cancer-Immunity Cycle. In the first step, neoantigens created by
oncogenesis are released and captured by dendritic cells (DCs) for processing. In order for this
step to yield an anticancer T cell response, it must be accompanied by signals that specify
immunity lest peripheral tolerance to the tumor antigens is induced. Such immunogenic signals
might include proinflammatory cytokines and factors released by dying tumor cells or by the
gut microbiota. Next, DCs present the captured antigens on MHCI and MHCII molecules to T
cells, resulting in the priming and activation of effector T cell responses against the cancer-
specific antigens that are viewed as foreign or against which central tolerance has been
incomplete. The nature of the immune response is determined at this stage, with a critical
balance representing the ratio of T effector cells versus T regulatory cells being key to the final
outcome.
Text C

Finally, the activated effector T cells traffic to and infiltrate the tumor bed, specifically
recognize and bind to cancer cells through the interaction between its T cell receptor (TCR) and
its cognate antigen bound to MHCI, and kill their target cancer cell. The killing of the cancer cell
releases additional tumor-associated antigens (step 1 again) to increase the breadth and depth
of the response in subsequent revolutions of the cycle. In cancer patients, the Cancer-Immunity
Cycle does not perform optimally. Tumor antigens may not be detected, DCs and T cells may
treat antigens as self rather than foreign thereby creating T regulatory cell responses rather
than effector responses, T cells may not properly go well with tumors, may be inhibited from
infiltrating the tumor, or (most importantly) factors in the tumor microenvironment might
suppress those effector cells that are produced.
The goal of cancer immunotherapy is to initiate or reinitiate a self-sustaining cycle
of cancer immunity, enabling it to amplify and propagate, but not so much as to generate
unrestrained autoimmune inflammatory responses. Cancer immunotherapies must, therefore,
be carefully configured to overcome the negative feedback mechanisms. Although checkpoints
and inhibitors are built into each step that opposes continued amplification and can dampen or
arrest the antitumor immune response, the most effective approaches will involve selectively
targeting the rate-limiting step in any given patient. Amplifying the entire cycle may provide
anticancer activity but at the potential cost of unwanted damage to normal cells and tissues.
Many recent clinical results suggest that a common rate limiting step is an immunostat function,
immunosuppression that occurs in the tumor microenvironment
Text D

Attempts to activate or introduce cancer antigen-specific T cells, as well as stimulate the


proliferation of these cells over the last 20 years, have led to mostly no, minimal or modest
appreciable anticancer immune responses. The majority of these efforts involved the use of
therapeutic vaccines because vaccines can be easy to deploy and have historically represented
an approach that has brought enormous medical benefit. Yet, cancer vaccines were limited to
two accounts.
First, until recently, there was a general lack of understanding of how to immunize the patients
for effective cytotoxic T cell responses. This limitation reflects continued uncertainties
concerning the identities of antigens to use, their mode of delivery, the types of adjuvants
required, and the proximal characteristics of the desired T cell response. Second, the presence
of the immunostat in the tumor microenvironment may dampen or disable antitumor immune
responses before clinically relevant tumor kill can occur. Thus, as long as these negative signals
are in place, the prospects for vaccine-based approaches used alone are likely to be limited. -
Mark Stephens, Cardiologist, Care CRM, London.
PART A -QUESTIONS AND ANSWER SHEET

Questions 1-7
For each question, 1-7, decide which text (A, B, C or D) the information comes from. You may use any
letter more than once.
In which text can you find information about;
1.Seven crucial progression acts. Answer ________________
2.Initiating Anticancer Immunity. Answer _________________
3.Checkpoints and (immunostat function) Answer __________________
4 .Didn`t achieve any proper result. Answer __________________
5.Talking about the process of removal of cancer cells. Answer __________________
6.Killing of target cancer cell. Answer _______________
7. Immune editing. Answer ______________

Questions 8-14
Answer each of the questions, 8-14, with a word or short phrase from one of the texts. Each answer
may include words, numbers or both. Your answers should be correctly spelt.

8. Did experiments or endeavors with respect to T-cells lead to any significant results or immune
responses? Answer _________________

9. At which stage the nature of the immune response is determined? Answer ________

10.Which cells may find it difficult to home to tumors? Answer _____

11. What is the term referred to describe the series of events involved in anticancer immune
response? Answer _____________

12. Presence of what will dampen or disable antitumor immune responses in the tumor
microenvironment? Answer ______________

13. What are the most effective approaches in an immunotherapy? Answer _______________

14.What target was not achieved so far in immunizing cancer patients? Answer ________
Questions 15-20

Complete each of the sentences, 15-20, with a word or short phrase from one of the texts. Each
answer may include words, numbers or both. Your answers should be correctly spelt.

15 Destroying the cancer cell releases additional ________________

16 ____is referred to as partial or complete suppression of the immune response of an individual

17 Majority of the processes with respect to activation of the T-cells comprises use of ____

18 Presently, the challenge is how to effectively use this understanding and develop ____

19 In ___________, activated effector T cells may entirely infiltrate the tumor bed.

20 The initial step can provide a response only when it is along with___________, which can clearly
show immunity.

END OF PART A, THIS QUESTIONS PAPER WILL BE COLLECTED


READING SUB-TEST : PART B
In this part of the test, there are six short extracts relating to the work of health professionals .
For questions 1-6, choose the answer (A, B or C) which you think fits best according to the text. Write
your answers on the separate Answer Sheet
Questions 1-6

1. The GCA symptoms can;


A. Begin and grow suddenly or gradually.
B. Begin all of sudden.
C. Occur more gradually.

GCA - Giant Cell Arteritis

The onset of giant cell arteritis (GCA) may be either abrupt or insidious. GCA may begin with
constitutional manifestations such as anorexia, fever, malaise, myalgia, night sweat, and
weight loss. These prodromal symptoms may occur for a few days and may even stretch out to
weeks. The most commonly reported symptoms in patients with GCA are as follows:
• Headache (initial symptom in 33%, present in 72%)
• Neck, torso, shoulder, and pelvic girdle pain that is consistent with polymyalgia rheumatica
(PMR; initial in 25%, present in 58%)
• Fatigue and malaise (initial in 20%, present in 56%)
• Jaw claudication (initial in 4%, present in 40%)
• Fever (initial in 11%, present in 35%)
2. Head anastomosis venture may mean;
A. human head transplantation.
B. B head transplantation in China.
C. C adding head of one patient to the body of the other.
Neurosurgeon Sergio Canavero proposed the HEAVEN procedure – i.e. head anastomosis
venture – several years ago, and has recently received approval from the relevant regulatory
bodies to perform this body-head transplant (BHT) in China. The BHT procedure involves
attaching the donor body (D) to the head of the recipient (R), and discarding the body of R and
head of D. Canavero’s proposed procedure will be incredibly difficult from a medical
standpoint. Aside from medical doubt, the BHT has been met with great resistance from many,
if not most bio- and neuro ethicists.

3. What this notice talk about?


A. Patients with LS should undergo a range of ongoing surveillance activities.
B. Patients with LS are recommended to undergo a range of ongoing surveillance activities.
C. There are awide range of ongoing surveillance activities which are undertaken by caretakers.
LS surveillance support initiatives or ideas have included earlier and more frequent
colonoscopy, consideration for the transvaginal ultrasound and endometrial sampling with the
possible recommendation for a total hysterectomy, and clinical neurologic examination. Upper
endoscopy, annual urinalysis, and pancreatic cancer screening may also be considered based
on family history. Screening guidelines continue to be updated as new research clarifies what
recommendations are beneficial, but few participants receive repeat genetic counseling to
update surveillance and surgical recommendations based on a patient’s specific LS gene
mutation and family history.
4. As per the given notice, hereditary breast cancers;
A. occur commonly due to pathogenic mutations.
B. can easily be detected with advanced strategy.
C. can be treated more cost-effectively.

Hereditary breast cancers account for approximately 10% of all breast cancers, and
approximately 23% of all ovarian cancers are considered hereditary. According to Plakhins et
al., BRCA1 pathogenic founder mutations contribute to 3.77% of all consecutive primary breast
cancers and 9.9% of all consecutive primary ovarian cancers. BRCA1 and BRCA2 pathogenic
founder mutation analysis is a relatively straightforward and cost-effective screening strategy
to identify mutation carriers. In Latvia, all consecutive breast and ovarian cancer cases are
eligible for BRCA1 pathogenic founder mutations (c.181 T > G, c.4035delA, c.5266dupC)
screening, and the costs of the test are covered by the public health care system.
5. What is correct?
A. The average age of PMR patients with amyloidosis is higher than the average age of PMR
patients without TA.
B. the average age of PMR patients without TA (p < 0.0164) is significantly lower than the
average age of PMR patients without amyloidosis.
C. The average age of PMR patients with TA is significantly higher.

The average age of PMR patients


Total (female & male)
Female
Male
PMR versus Ath
0,4494
0,2972
0,4217
PMR with Ath versus without Ath
0,4222
0,2703
0,0004
PMR with TA versus without TA
0,0164
0,0159
0,7951
PMR with amyloidosis versus without amyloidosis
0,0000
0,0000
0,0037
TA with amyloidosis versus without amyloidosis
0,2995
0,3727
0,7228
6. Patient clinical data shows that the patient;
A. is suffering from BP.
B. is suffering from heartdisease.
C. is suffering from chest pain
READING SUB-TEST : PART C
In this part of the test, there are two texts about different aspects of healthcare.
For questions 7-22, choose the answer (A, B, C or D) which you think fits best according to the text.
Write your answers on the separate Answer Sheet

Text 1: A note on Eye Infections – Conjunctivitis

Conjunctivitis is a common and often very contagious condition more commonly


referred to as a pink eye. Conjunctivitis often involves inflammation and swelling
of the conjunctiva or the clear membrane covering the eye and lining the inner
eyelids. It is very common among young children, particularly those in school or
daycare. Elderly people can also be affected. Conjunctivitis typically poses no real
threat to the patient`s vision or wellness. There are multiple causes and risk factors
for conjunctivitis. Some of the more common causes of conjunctivitis include
Allergic reactions, Viral contamination, Bacterial contamination, Exposure to
irritants or chemical pollutants. Infectious conjunctivitis typically results from
bacterial or viral contamination. While bacterial conjunctivitis may be treated with
certain antibiotics, there is no traditional treatment for viral conjunctivitis.
Multiple strains of bacteria are responsible for bacterial conjunctivitis; these
include haemophilus influenzae and Staphylococcus aureus. Viral conjunctivitis is
a common condition affecting children from the time of birth through to their adult
years. Viruses can enter the eye in many ways, resulting from the common cold or
contagion with the flu. Some patients may develop a more serious form of viral
conjunctivitis associated with a herpes infection; in this case, prompt medical
attention is necessary to prevent permanent damage to the eye or vision.
One of the more commonly reported signs and symptoms of conjunctivitis include
discomfort and pain in the eye, accompanied by redness or swelling of the eyelid,
hence the name "pinkeye." Patients may experience reddening of the inner and
outer eyelid or may experience swollen eyelids. Some patients may report
increased sensitivity to light and other irritants including the wind. Many
patients with conjunctivitis will have discharge from the eyes that may be clear,
white, green or yellow in color. Yellow or green discharge from the eye may
suggest an infection and may require antibiotics or other forms of aggressive
treatment. Conjunctivitis can spread from person to person or from eye to eye.
Simply touching one eye then the other can spread viral conjunctivitis. Allergic
conjunctivitis is a non-contagious condition. Typically, children exposed to bacterial
conjunctivitis will have a two to four day window before symptoms appear.
Bacterial infections mostly come from staphylococci and streptococci organisms
that can come from your own skin or upper respiratory tract. The indicating
symptoms of bacterial infections are thick, ropy mucus discharge accompanied
with red, irritated and inflamed eyes. Luckily, bacterial eye infections are easily
treated with antibiotic eye drops and, in most cases, will clear up within a few
short days. Viral infections are commonly caused by an enterovirus and often
associated with an upper respiratory infection or common cold. Eyes are red and
inflamed and become watery and runny. One of the most common viral infections
is epidemic keratoconjunctivitis, also known as EKC; it is highly contagious and
can last up to 2 weeks or more. This viral conjunctivitis is caused by an adenovirus
and does not have a specific treatment to cure the infection. The doctor may
prescribe steroid eye drops and artificial tears to help decrease inflammation, but
mostly the virus simply needs to run its course.

Conjunctivitis may also result from chlamydia and gonococcal infections or STD`s.
Usually, the inner eyelid becomes infected. This condition is more commonly noted
in teens and young adults who are sexually active. When left untreated, this condition
may affect newborn infants born to mothers infected with an STD. Signs may include
a history of pelvic pain or vaginitis as well. Patients with Gonococcal infections may
feel like a foreign object is chronically present within their eye, and are more likely to
experience burning and inflammation. It is possible to transfer these conditions to the
eye from hand contact so it is important, to help prevent the spread of infection, that
frequent hand washing is adopted by patients and family members. Treatment usually
involves use of antibiotics taken topically or orally and concomitant treatment may be
necessary to treat genital and eye infections.
Text 1: Questions 7-14

7. Which part of the eye is affected by conjunctivitis?


A. Conjunctiva
B. White membrane
C. Eyelids
D. Only A and B

8. Conjunctivitis is common among;


A. Children
B. Teens
C. School goers
D. Daycare children

9. What is linked with the herpes infection?


A. Bacterial infections.
B. Viral infections.
C. Allergic reactions.
D. Exposure to chemical pollutants.

10. What is the most common symptom of conjunctivitis?


A. Mild pain in the eye.
B. Watery eyes.
C. Reddening of the inner and outer eyelid.
D. All of the above.

11. The color of discharge from eyes may be;


A. Yellow and green.
B. White, yellow and green.
C. Yellow or green.
D. White or green.
12. Paragraph 3 is focused more on;
A. Types of conjunctivitis.
B. Types of bacterial conjunctivitis.
C. Types of viral conjunctivitis.
D. Types of bacterial and viral conjunctivitis.

13. viral conjunctivitis is caused by;


A. Androvirus
B. Adenovirus
C. Enterovirus
D. Adenonvirus

14. Which is more common in gonococcal infections?


A. Pain around pelvis.
B. Discharge or pus.
C. Inflammation of the eyes.
D. Reddening of the eyes.
Text 2: Osteomyelitis is an infection of the bone.

Osteomyelitis is an infection of the bone. Osteomyelitis can occur in infants,


children, and adults. Different types of bacteria typically affect the different age
groups: in children, osteomyelitis most commonly occurs at the ends of the long
bones of the arms and legs, affecting the hips, knees, shoulders, and wrists,
whereas in adults, it is more common in the bones of the spine (vertebrae), feet, or
in the pelvis. There are several different ways to develop the bone infection of
osteomyelitis. The first is for bacteria to travel through the bloodstream
(bacteremia) and spread to the bone, causing an infection. This mostly occurs
when the patient has an infection elsewhere in the body, such as pneumonia or a
urinary tract infection that spreads through the blood to the bone. An open wound
over a bone can lead to osteomyelitis. A recent surgery or injection around a bone
can also expose the bone to bacteria and lead to osteomyelitis. Patients with
conditions or taking medications that weaken their immune system are at a higher
risk of developing osteomyelitis. Risk factors include cancer, chronic steroid use,
sickle cell disease, human immunodeficiency virus (HIV), diabetes, hemodialysis,
intravenous drug users, and the elderly.

Symptoms of osteomyelitis can vary greatly. In children, osteomyelitis usually


occurs more quickly. They develop pain or tenderness over the affected bone, and
they may have difficulty or inability to use the affected limb or to bear weight or
walk due to severe pain. They may also have fever, chills, and redness at the site
of infection. In adults, the symptoms often develop more gradually and include
fever, chills, irritability, swelling or redness over the affected bone, stiffness, and
nausea. In people with diabetes, peripheral neuropathy, or peripheral vascular
disease, there may be no pain or fever. The only symptom may be an area of skin
breakdown that is worsening or not healing. Acute osteomyelitis occurs with a
rapid onset and is usually accompanied by the symptoms of pain, fever, and
stiffness. It generally occurs after a break in the skin from injury, trauma, surgery,
or skin ulceration from wounds. Chronic osteomyelitis is insidious in onset; it may
be the result of a previous infection of osteomyelitis and, despite multiple courses
of antibiotics, it may reoccur. Symptoms of chronic osteomyelitis are subtle but
may include fever, pain, redness, or discharge at the site of infection.

The diagnosis of osteomyelitis begins with a complete medical history and physical
examination. During the discussion about medical history, the doctor may ask
questions about recent infections elsewhere in the body, past medical history,
medication usage, and family medical history. The physical examination will look for
areas of tenderness, redness, swelling, decreased or painful range of motion, and
open sores. The doctor may then order tests to help diagnose osteomyelitis. Several
blood tests can be used to help determine if there is an
infection present; these include a complete blood count (CBC), the erythrocyte
sedimentation rate (ESR), C-reactive protein (CRP), and blood cultures. None of these
is specific for osteomyelitis but they can suggest that there may be some infection in
the body. Imaging studies of the involved bones may be obtained; these can include
plain radiographs (X-rays), bone scans, computed tomography (CT) scans, magnetic
resonance imaging (MRIs), and ultrasounds. These imaging studies can help identify
changes in the bones that occur with osteomyelitis.
In many cases, osteomyelitis can be effectively treated with antibiotics and pain
medications. If a biopsy is obtained, this can help guide the choice of the best
antibiotic. The duration of treatment of osteomyelitis with antibiotics is usually
four to eight weeks but varies depending on the type of infection and the response
to the treatments. In some cases, the affected area will be immobilized with a
brace to reduce the pain and speed up the treatment. Sometimes, surgery may be
necessary. If there is an area of localized bacteria (abscess), this may need to be
opened, washed out, and drained. If there is damaged soft tissue or bone, this may
need to be removed. If bone needs to be removed, it may need to be replaced with
bone graft or stabilized during surgery.
With early diagnosis and appropriate treatment, the prognosis for osteomyelitis is
good. Antibiotics regimes are used for four to eight weeks and sometimes longer
in the treatment of osteomyelitis depending on the bacteria that caused it and the
response of the patient. Usually, patients can make a full recovery without
longstanding complications.

However, if there is a long delay in diagnosis or treatment, there can be severe


damage to the bone or surrounding soft tissues that can lead to permanent deficits
or make the patient more prone to recurrence. If surgery or bone grafting is
needed, this will prolong the time it takes to recover.
Text 2: Questions 15-22
15. Which of the following statements is not true, according to the information provided in
paragraph 1?
A. An osteomyelitis is common among people of all ages.
B. An osteomyelitis can be caused by bacteria.
C. In most of the cases, people who have pneumonia may develop osteomyelitis.
D. An osteomyelitis is commonly observed in people who are prone to bacterial infections.

16. Which of the following can lead to osteomyelitis?


A. Bacterial infection.
B. An open wound.
C. Underrated bone operation.
D. None

17. Symptoms of osteomyelitis in children may include;


A. Tenderness in the bone;
B. High fever;
C. Inability to walk;
D. All of the above;

18. Which type of osteomyelitis is slow in the beginning?


A. Chronic
B. Acute
C. Both chronic and acute
D. None

19. Which group is more specific for diagnosis?


A. CBC and ESR only
B. Only ESR and CRP
C. CRP, ESR and CBC
D. None
20. According to paragraph 3, involved bone suggests
A. Wounded bone
B. Operated bone area
C. Bacterial infected bone
D. Bone with tenderness

21. According to paragraph 4, which one of the following statements is not correct?
A. Treatment may be completed within 4 to 8 weeks.
B. The duration of the treatment varies depending on the type of osteomyelitis.
C. Working of the affected area is stopped in order to speed up the treatment.
D. Bone is often replaced with bone graft for perfect treatment.

22. According to paragraph 5, delay in treatment of diagnosis may result in;


A. Inability to walk properly.
B. Permanent damage to bone.
C. Removal of affected bone.
D. None

END OF READING TEST, THIS BOOKLET WILL BE COLLECTED


Reading test 3 : Answer Key

Part A - Answer key 1 – 7


1: B
2: D
3: A
4: D
5: B
6: C
7: A

Part A - Answer key 8 – 14


8: No
9: Stage 3
10: T cells
11: Cancer-Immunity Cycle
12: immunostat
13: selectively targeting the rate-limiting step
14: effective cytotoxic T cell responses

Part A - Answer key 15 – 20


15: tumor-associated antigens
16: immunosuppression
17: therapeutic vaccines
18: new drugs
19: step 4
20: signals
Reading test - part B – answer key
1: B Begin All Of Sudden.
2: A human head transplantation.
3: B Patients with LS are recommended to undergo a range of ongoing surveillance activities.
4: B can easily be detected with advanced strategy.
5: C The average age of PMR patients with TA is significantly higher.
6: A is suffering from BP.

Reading test - part C – answer key

Text 1 - Answer key 7 – 14


7: D Only A and B
8: D Daycare children
9: B Viral infections.
10: C Reddening of the inner and outer eyelid.
11: B White, yellow and green.
12: A Types of conjunctivitis.
13: B Adenovirus
14: C Inflammation of the eyes.

Text 2 - Answer key 15 – 22


15: C In most of the cases, people who have pneumonia may develop osteomyelitis.
16: B An open wound.
17: D All of the above;
18: A Chronic
19: D None
20: C Bacterial infected bone
21: D Bone is often replaced with bone graft for perfect treatment.
22: B Permanent damage to bone.
READING TEST 4
READING SUB-TEST : PART A
 Look at the four texts, A-D, in the separate Text Booklet.
 For each question, 1-20, look through the texts, A-D, to find the relevant information.
 Write your answers on the spaces provided in this Question Paper.
 Answer all the questions within the 15-minute time limit.
 Your answers should be correctly spelt.
TEXT BOOKLET- CHRONIC FATIGUE SYNDROME (CFS)

Text A

Chronic fatigue syndrome (CFS)


Chronic fatigue syndrome (CFS) is a highly complex illness that results in
significant disability and a considerably diminished quality of life. Due to
continued questions regarding etiology, the period of onset for the illness is of
considerable interest to researchers in the field. There is fairly little strife as to
whether the illness labels CFS, ME, ME/CFS, myalgic encephalopathy, and SEID
represent one distinct condition, whether they are part of an illness spectrum, or
whether they are simply different terms used to describe the same condition. CFS,
ME, ME/CFS, and the recently named SEID, are often associated with different
case criteria. Early case criteria developed by Holmes. specify that the illness must
have a “new onset of persistent or relapsing, debilitating fatigue” without any
previous history of similar problems. Additionally, Holmes stipulate that the main
symptoms of CFS must occur over a few hours or days, indicating a sudden or
acute onset. According to Holmes, symptoms are only met if they begin at the time
of the fatigue onset or following onset. Another case criteria for CFS is referred to
as the Oxford Criteria, which stipulate that CFS involves a “definite” onset as well
as clear evidence of infection at the time of onset or first symptoms. Similar to the
Oxford Criteria, the Fukuda criteria describe the onset of the fatiguing illness as
“new” and “definite”.
Text B

The terms Myalgic Encephalomyelitis (ME) and Myalgic


Encephalomyelitis/chronic fatigue syndrome (ME/CFS) have corresponding case
criteria, which are different from the Fukuda criteria in that they require what are
considered by many to be key symptoms of the illness (e.g. post-exertional malaise
and cognitive dysfunction). The criteria for ME/CFS specify that an individual
must have a “significant degree of new onset” fatigue. Similar to the Holmes
criteria for CFS, the case criteria for ME/CFS stipulate that symptoms can only be
counted as meeting criteria if they occur or become significantly worse after the
onset of the illness. Carruthers et al. describe onset as “distinct” and assert that
most individuals experience an acute onset; however, they also assert that some
individuals are unhealthy prior to their ME/CFS onset and may not be able to
identify a specific trigger for the development of ME/CFS, or they may experience
a more “gradual” or “insidious” onset.
Text C

Hyde’s Nightingale Definition of ME stipulates that ME is both chronic and


disabling and is characterized by an acute onset. Additionally, Hyde describes ME
as an epidemic or an endemic occurring in two phases. Additionally, Hyde
indicates that ME often follows multiple, minor infections in individuals with
susceptible immune systems or immune systems that are weakened by severe
stressors. Hyde describes the initial phase of ME as the Primary Infection Phase,
which is characterized as an epidemic or endemic infectious disease with an
incubation period of between four and seven days. He describes the second phase
as the Secondary Chronic Phase, occurring with two to seven days of the Primary
Infection Phase. In this phase, Hyde asserts that there are measurable changes in
the central nervous system (CNS) of an affected individual and that this phase is
the chronic form of the disease that is most commonly depictive of ME.
Understanding the cause of ME can be like going through labyrinthine routes in
scientific discovery, as Hyde asserts that all cases of epidemic and primary ME
result from an infectious or autoimmune agent, but he also suggests that there are
often other potential causes that may go unnoticed prior to the onset of illness or as
part of the illness.
Text D

There is not one universally used or empirically derived definition of onset


duration for ME and CFS. Researchers interested in assessing the mode of illness
onset have used various definitions. Often, the distinction is made between sudden
and acute onset and insidious and gradual onset, but the duration length prescribed
to each category differs. For instance, DeLuca et al. defined sudden onset for CFS
as an unrecoverable “viral-like illness” that could be traced to a definite date.
Gradual onset was described as a “slow progression of symptoms over a period of
weeks to several months” or longer. In his study of CFS in an adolescent sample,
Bell defined sudden/acute onset as an “abrupt onset of constant and debilitating
fatigue that could be dated to a specific event or illness” . He described all other
onset patterns as gradual.
PART A -QUESTIONS AND ANSWER SHEET

Questions 1-7
For each question, 1-7, decide which text (A, B, C or D) the information comes from.
You may use any letter more than once.
In which text can you find information about;
1 .Occurs in two different phases. Answer ________
2. All patterns are gradual. Answer_________
3. He believed that ME is more complicated. Answer___________
4. Controversy over the disease conditions. Answer__________
5 .No clear definition. Answer__________
6. Theory which suggests beginning of body weakening. Answer __________
7 .Causes a spike in symptoms and a massive energy crash. Answer___________

Questions 8-14
Answer each of the questions, 8-14, with a word or short phrase from one of the
texts. Each answer may include words, numbers or both. Your answers should be correctly spelt.

8. Who thinks that ME is long lasting and ant it can be acquired extreme onset? Answer______
9 .Who defined onset as incurable? Answer_______
10. According to which principle, CFS involves a definite onset & clear evidence of infection?
Answer_____________
11. Who specified that the CFS symptoms must occur over a few hours or days, indicating an acute
onset? Answer ____________
12 .Who described that sudden onset for CFS as an unrecoverable viral-like illness? Answer__
13 .Whose statement signifies that the affected patient may go through a highly secretive and
detrimental beginning? Answer ___________
14 .Who described onset as more debilitating fatigue? Answer___________
Questions 15-20

Complete each of the sentences, 15-20, with a word or short phrase from one of the
texts. Each answer may include words, numbers or both. Your answers should be correctly spelt.

15 .____________ described the onset phase of ME as the Primary Infection

16. ME may follow ___________ infections in patients with not-so-strong immune system

17 .CFS criteria are said to have much more similarity with respect to____________

18. CFS may comprise a___________ onset and strong evidence of infection when there are first
signs of symptoms.

19. The distinction is made between _____________ and insidious/gradual onset, but the duration
length prescribed to each category differs.

20.____________ may occur due to an infectious or autoimmune agent.

END OF PART A, THIS QUESTIONS PAPER WILL BE COLLECTED


READING SUB-TEST : PART B

In this part of the test, there are six short extracts relating to the work of health professionals . For
questions 1-6, choose the answer (A, B or C) which you think fits best according to the text. Write your
answers on the separate Answer Sheet

Questions 1-6

1 What is correct about gait disturbance?


A. Pelvic tilt is common

B. Fast gait speed and improved gait pattern is the most important goal of stroke
rehabilitation

C. A hemiplegic gait may include body asymmetry, decreased weight bearing on


the affected side.

Gait disturbance

Gait disturbance is the most common problem after stroke. This problem is related
to poor ADL and mobility, and increases the risk of fall in severe cases. The body
alignment of stroke patients becomes asymmetric if they have a hemi-paralysis,
muscle weakness, motor and sensory function decrease. These problems produce a
hemiplegic gait in stroke patients. It may include poor equilibrium reaction, and
impaired selective motor control. Good body alignment is very important clinically
because asymmetry leads to inefficient energy during walking, the risk of
musculoskeletal injury in the unaffected side, and loss of bone density. Excessive
pelvic elevation and the pelvic tilt angle is directly connected to hemiplegic gait
and poor motor function in stroke patient causes an excessive pelvic tilt during gait
2. The notice is talking about;
A. Postoperative care.
B. functions of IV insulin
C. insulin dosage

IV Insulin Infusions
Patients that have required IV insulin infusions can be transitioned to subcutaneous
insulin once infusion rates are stable and glucose controlled, particularly if a diet has been
initiated. Because IV insulin has a very short half-life, the subcutaneous insulin should be
administered prior to discontinuation of IV insulin. The basal infusion rate during fasting is a
good predictor of basal subcutaneous insulin requirements, however, providers often reduce
the amount by 20% upon transition. For instance, if a patient required 1.5 unit/hour of IV
insulin overnight, this would suggest a basal need of approximately 36 units of insulin daily.
However, reducing this by 20% would result in a starting basal dose of 30 units of insulin
daily.

3. The use of molecule-altering technologies;


A. has flared up more concerns.
B. is not suitable.
C. is fast advancing.

Molecule-altering technologies
The advent of molecule-altering technologies and improved synthetic methods has
led to the finding of newer proteins and peptides that resemble human proteins and
peptides. Although, capable of producing potential therapeutic benefits, protein
molecules have serious biopharmaceutical concerns such as, poor shelf- life, rapid
degradation in the physiological environment, poor solubility, immunogenicity and
antigenicity. These concerns can be overcome by utilizing the beneficial properties of
polyethylene glycols and PEGylation. ‘PEGylation’ is the process of chemical attachment of
PEG to bioactive proteins and peptides, to modify their pharmacokinetic and
pharmacodynamic properties.
4. The notice is talking about;
A. when to perform the exercise

B. best exercise timing

C. benefits of exercise

Moderate mid-postprandial exercise

Although an uneasy consensus favoring moderate mid-postprandial exercise as


better suited for glycaemia benefits exists among researchers, pre-meal exercise
has its advantages: little risk for hypoglycemia, enhanced insulin sensitivity, and
improved muscle glycogen content and GLUT-4 protein levels. The downside of
pre-meal exercise is elevated postprandial glucose (PPG), A recent systematic
review concluded that >45 min of aerobic exercise (AE) post-meal offered
consistent glycaemia benefits. Also, the review identified resistance training as an
effective modality in this regard. The patient found a 30-min pre-meal walk
followed by another 30 minutes of post-meal walk blunted the post-meal glucose
surge better than a 60 min pre-meal or post-meal walk. The post-meal walk might
have cleared some of the extra blood glucose arrived from the liver. Split exercise
at lunchtime was comparable to mid-postprandial exercise in improving glycaemia
and oxidative stress, although there was less hyperglycemia after the meal.
5. Zidovudine can;
A. Increase decrease of lactate.

B. Lead to fatty change

C. Be used for treating anemia

Zidovudine

Nausea is another common side effect that may be present in early zidovudine use.
Other side effects of zidovudine may include granulocytopenia, myopathy, lactic
acidosis, hepatomegaly with steatosis, headache. Myopathy may occur within 6-12
months of initiating zidovudine, and has an insidious onset that involves proximal
muscle weakness and exercise-induced myalgias. The mechanism of myopathy is
believed to be mitochondrial toxicity within myocytes. Zidovudine should be used
with caution in patients who have anemia (hemoglobin less than 9.5
grams/deciliter). Reduction of hemoglobin may occur as early as 2 to 4 weeks.
Severe anemia may require dose adjustment, discontinuation, and/or blood
transfusions. Doses should be reduced until bone marrow recovers if the anemia is
significant (hemoglobin less than 7.5 grams/deciliter or reduction of greater than
25% of baseline).
6. The table
A. doesnt show a much significant difference in gentamicin doses between two groups of
patients

B. Use of gentamicin dose is significantly lower

C. Dose of 2 mg/kg is higher in noncritically ill patients.

Amikacin and gentamicin dosing in critically and noncritically ill patients


READING SUB-TEST : PART C
In this part of the test, there are two texts about different aspects of healthcare.
For questions 7-22, choose the answer (A, B, C or D) which you think fits best according to the text.
Write your answers on the separate Answer Sheet

Text 1: What is Yersiniosis?

Yersiniosis is an infectious disease caused by a bacterium of the genus Yersinia. In


the United States, most human illness is caused by one species, Y enterocolitica
(not more than one species often). Infection with Y. enterocolitica can cause a
variety of symptoms depending on the age of the person infected. Infection with Y.
enterocolitica occurs most often in young children; adults may be on a safer side.
Common symptoms in children are fever, abdominal pain, and diarrhea, which is
often bloody. Symptoms typically develop 4 to 7 days after exposure and may last
1 to 3 weeks or longer. In older children and adults, right-sided abdominal pain and
fever may be the predominant symptoms, and may be confused with appendicitis. In
a small proportion of cases, complications such as skin rashes, joint pains, or the
spread of bacteria to the bloodstream can occur.

Y. enterocolitica belongs to a family of rod-shaped bacteria. Other species of


bacteria in this family include Y. pseudotuberculosis, which causes an illness
similar to Y. enterocolitica, and Y. pestis, which cause plague. Only a few strains of
Y. enterocolitica cause illness in humans. The major animal reservoir for Y.
enterocolitica strains that causes human illness is pigs, but other strains are also
found in many other animals including rodents, rabbits, sheep, cattle, horses, dogs,
and cats. In pigs, the bacteria are most likely to be found on the tonsils.

Infection is most often acquired by eating contaminated food, especially raw or


undercooked pork products. The preparation of raw pork intestines (chitterlings)
may be particularly risky. Infants can be infected if their caretakers handle raw
chitterlings and then do not adequately clean their hands before handling the infant
or the infant’s toys, bottles, or pacifiers. Drinking contaminated unpasteurized milk
or untreated water can also transmit the infection. Occasionally Y. enterocolitica
infection occurs after contact with infected animals. On rare occasions, it can be
transmitted as a result of the bacterium passing from the stools or soiled fingers of
one person to the mouth of another person. This may happen when basic hygiene
and handwashing habits are inadequate. Rarely, is the organism transmitted
through contaminated blood during a transfusion.

Y. enterocolitica is a relatively infrequent cause of diarrhea and abdominal pain.


Based on data from the Food-borne Diseases Active Surveillance Network which
measures the burden and sources of specific diseases over time, approximately one
culture-confirmed Y. enterocolitica infection per 100,000 people occurs each year.
Children are infected more often than adults, and the infection is more common in
the winter. Y. enterocolitica infections are generally diagnosed by detecting the
organism in stools. Many laboratories do not routinely test for Y. enterocolitica, so
it is important to notify laboratory personnel when infection with this bacterium is
suspected so that special tests can be done. The organism can also be recovered
from other sites, including the throat, lymph nodes, joint fluid, urine, bile, and
blood. Uncomplicated cases of diarrhea due to Y. enterocolitica usually resolve on
their own without antibiotic treatment. However, in more severe or complicated
infections, antibiotics such as aminoglycosides, doxycycline,
trimethoprimsulfamethoxazole, or fluoroquinolones may be useful.

There are many things which can be done to prevent the infection or the spread of
the infection: Avoid eating raw or undercooked pork. Consume only pasteurized milk or
milk products. Wash hands with soap and water before eating and preparing food, after
contact with animals, and after handling raw meat. After handling raw chitterlings,
clean hands and fingernails scrupulously with soap and water before touching
infants or their toys, bottles, or pacifiers. Someone other than the food handler
should care for children while chitterlings are being prepared. Prevent
crosscontamination in the kitchen - use separate cutting boards for meat and other foods,
carefully clean all cutting boards, counter-tops, and utensils with soap and hot
water after preparing raw meat. Dispose of animal feces in a sanitary manner.
Text 1: Questions 7-14

7. Yersiniosis occurs more commonly in;


A. Children
B. Americans
C. Adults
D. Teens

8. Symptoms such as fever and diarrhea may persist for about;


A. 4 days

B. 7 days

C. 1-3 weeks

D. More than 3 weeks

9. Plague is caused by;


A. Y. pestis

B. Y. enterocolitica

C. Y. pseudotuberculosis

D. None

10. One of the following is not a common form of transmission of the parasite;
A. Contaminated unpasteurized milk

B. Blood transfusion

C. Raw or undercooked pork products

D. After contact with infected animals


11. Paragraph 4 talks about;
A. Prevention of the disease

B. Spread of the infection

C. How common the infection of Y. enterocolitica is

D. Reports by Active Surveillance Network.

12. Paragraph 5 talks about;


A. Diagnosis

B. Treatment

C. Spread of the infection and its control.

D. A & B

13. Which of these can be derived from Paragraph 5?


A. Treatment is not required for the infection caused by Y. enterocolitica.

B. Diarrhea requires no treatment

C. Complications which are mild can be resolved easily.

D. None

14. What is said about chitterlings?


A. Handling raw chitterlings with care is necessary
B. Contamination occurs more due to the careless handling chitterlings
C. Washing hands with soap before touching infants or their toys is vital
D. There should be separate caretakers to look after infants, while chitterlings are prepared
Text 2 : What is an MRI scan?

An MRI (or magnetic resonance imaging) scan is a radiology technique that uses
magnetism, radio waves, and a computer to produce images of body structures.
The MRI scanner is a tube surrounded by a giant circular magnet. The patient is
placed on a moveable bed that is inserted into the magnet. The magnet creates a
strong magnetic field that aligns the protons of hydrogen atoms, which are then
exposed to a beam of radio waves. This spins the various protons of the body, and
they produce a faint signal that is detected by the receiver portion of the MRI
scanner. The receiver information is processed by a computer, and an image is
produced. The image and resolution produced by MRI are quite detailed and can
detect tiny changes of structures within the body. For some procedures, contrast
agents, such as gadolinium, are used to increase the accuracy of the images.

An MRI scan can be used as an extremely accurate method of disease detection


throughout the body. In the head, trauma to the brain can be seen as bleeding or
swelling. Other abnormalities often found include brain aneurysms, strokes, tumors
of the brain, as well as tumors or inflammation of the spine. Neurosurgeons use an
MRI scan not only in defining brain anatomy but in evaluating the integrity of the
spinal cord after trauma. It is also used when considering problems associated with
the vertebrae or intervertebral discs of the spine. An MRI scan can evaluate the
structure of the heart and aorta, where it can detect aneurysms or tears. It provides
valuable information on glands and organs within the abdomen, and accurate
information about the structure of the joints, soft tissues, and bones of the body.
Often, surgery can be deferred or more accurately directed after knowing the
results of an MRI scan.
An MRI scan is a painless radiology technique that has the advantage of avoiding
x-ray radiation exposure. There are no known side effects of an MRI scan. The
benefits of an MRI scan relate to its precise accuracy in detecting structural
abnormalities of the body. Patients who have any metallic materials within the
body must notify their physician prior to the examination or inform the MRI staff;
metallic chips, materials, surgical clips, or foreign material can significantly distort
the images obtained by the MRI scanner. Patients who have heart pacemakers,
metal implants, or metal chips or clips in or around the eyeballs cannot be scanned
with an MRI because of the risk that the magnet may move the metal in these
areas. Similarly, patients with artificial heart valves, metallic ear implants, bullet
fragments, and chemotherapy or insulin pumps should not undergo MRI scanning.
During the MRI scan, the patient lies in a closed area inside the magnetic tube;
some patients can experience a claustrophobic sensation during the procedure.
Therefore, patients with any history of claustrophobia should relate this to the
practitioner who is requesting the test, as well as the radiology staff. A mild
sedative can be given prior to the MRI scan to help alleviate this feeling.

All metallic objects on the body are removed prior to obtaining an MRI scan.
Occasionally, patients will be given a sedative medication to decrease anxiety and
relax the patient during the MRI scan. MRI scanning requires that the patient lies
still for best accuracy; patients lie within a closed environment inside the magnetic
machine. Relaxation is important during the procedure and patients are asked to
breathe normally. Interaction with the MRI technologist is maintained throughout
the test and there are loud, repetitive clicking noises which occur during the test as
the scanning proceeds. Occasionally, patients require injections of liquid
intravenously to enhance the images which are obtained. The MRI scanning time
depends on the exact area of the body studied, but ranges from half an hour to an
hour and a half.
After the MRI scanning is completed, the computer generates visual images of the
area of the body that was scanned. These images can be transferred to film (hard
copy). A radiologist is a physician who is specially trained to interpret images of
the body. The interpretation is transmitted in the form of a report to the practitioner
who requested the MRI scan. The practitioner can then discuss the results with the
patient and/or family.
Text 2: Questions 15-22

15. According to paragraph 1, an image is produced;


A. When a signal is detected by the receiver portion of the scanner.
B. When the signal passes through the receiver, after coupling of the protons of the body.
C. When the signal is identified by the receiver which is accurately processed by a computer.
D. Only when the information, detected by the receiver is processed by a computer.

16. According to paragraph 1, what is the image like?


A. Crystal clear with no patches.
B. Self explanatory
C. Dense
D. Well enough to give a clear idea of the structures within the body.

17. According to paragraph 2, an MRI gives a clear idea about;


A. Brain aneurysms.
B. Strokes of the brain.
C. Brain tumours and spinal cord injury.
D. All

18. An MRI can;


A. Detect brain tumors.
B. Give a clear picture of spinal cord injury and other soft tissues related to it.
C. Give clear pictures to evaluate the structure of the heart and aorta.
D. All
19. Paragraph 3 talks about;
A. Detection of the diseases through an MRI scan.
B. Importance of taking an MRI scan.
C. Risks of an MRI scan.
D. None

20. Metallic materials mentioned in paragraph 3 include;


A. Surgical clips.
B. Artificial joints, metallic bone plates.
C. Only metal clips in or around the eyeballs.
D. A & B

21 .Paragraph 4 talks about;


A. How an MRI is performed.
B. How a patient prepares for an MRI scan.
C. How a patient prepares for an MRI scan and how it is performed.
D. How an MRI scan is different from others.

22. According to paragraph 5, interpreting the images implies;


A. Identifying the disease through scanning.
B. Analysis of the disease.
C. Decoding the report.
D. All of the above.

END OF READING TEST, THIS BOOKLET WILL BE COLLECTED


Reading test 4 : Answer Key

Part A - Answer key 1 – 7


1: C
2:D
3:C
4:A
5:D
6:A
7:B

Part A - Answer key 8 – 14


8: Hyde
9: DeLuca
10: Oxford Criteria
11: Holmes
12: DeLuca et al.
13: Carruthers
14: Bell

Part A - Answer key 15 – 20


15: Hyde
16: Multiple Minor
17: Fukuda Criteria
18: Sudden
19: Hydes
20: Primary Me
Reading test - part B – answer key

1: A hemiplegic gait may include body asymmetry, decreased weight bearing on the affected side.
2: Postoperative care.
3: has flared up more concerns.
4: best exercise timing
5: Lead to fatty change
6: Use of gentamicin dose is significantly lower

Reading test - part C – answer key

Text 1 - Answer key 7 – 14

7: Children
8: 1-3 weeks
9 : Y. pestis
10: Blood transfusion
11: How common the infection of Y. enterocolitica is
12: A & B
13: None
14: Washing hands with soap before touching infants or their toys is vital

Text 2 - Answer key 15 – 22

15: Only when the information, detected by the receiver is processed by a computer.
16 : Dense
17: All
18: All
19: Risks of an MRI scan.
20: A & B
21: How a patient prepares for an MRI scan and how it is performed.
22: All of the above.
READING TEST 5

READING SUB-TEST : PART A


Look at the four texts, A-D, in the separate Text Booklet.
For each question, 1-20, look through the texts, A-D, to find the relevant information.
Write your answers on the spaces provided in this Question Paper.
Answer all the questions within the 15-minute time limit.
Your answers should be correctly spelt.

TEXT BOOKLET- RHEUMATOID ARTHRITIS (RA)

Text A

Rheumatoid arthritis (RA) has wide variability in both its clinical presentation and its
autoantibody profile. Two well-known autoantibodies that are found in between 60-90% of RA
patients are rheumatoid factor (RF) and cyclic citrullinated peptide (CCP) antibody. Seropositivity
for these antibodies is associated with more destructive joint pathology and radiographic
progression of RA. Anti-Sjogren’s Syndrome related Antigen A (SSA) is associated with numerous
autoimmune conditions, including most notably Sjogren’s Syndrome. Anti-SSA is also found in
between 3-16% of RA patients and it is believed to be a clinical indicator of poor prognosis in RA.
Several studies have shown that RA patients with this antibody have a lesser clinical response to
infliximab.
Anti-SSA seropositivity is also associated with secondary Sjogren’s Syndrome. RA with secondary
Sjogren’s Syndrome is associated with worse clinical manifestations and increased antinuclear
antibody (ANA) positivity. The prevalence of ANA and anti-SSA has been shown to be higher in
African American (AA) RA patients compared to Caucasian (CAU) RA patients in two established
RA cohorts.
Text B

In RA, inflammatory cytokines such as tumor necrosis factor-alpha (TNF-α) and interferongamma
(IFN-γ) are the primary inducers of chemokine production. Chemokines then lead to increased
numbers of inflammatory cells, such as macrophages, lymphocytes, and fibroblast-like
synoviocytes, in inflamed synovial tissue. Chemokines also contribute to cartilage degradation
and pannus formation by stimulating the release of various inflammatory cytokines. Several
studies have shown that serum chemokines including CX3CL1, CCL5, CXCL9, and CXCL10 are
increased in active RA patients compared to healthy controls.. Particularly, several studies have
found that CXCL10 could serve as a disease activity marker in RA. Elevated CXCL10 and CXCL13
levels have been shown to be predictive of a favorable response to TNF inhibitor therapy.
Studies have also shown that serum chemokine levels, including CXCL9, CXCL10 and CXCL16
decrease after treatment with disease-modifying antirheumatic drugs or biologic agents.

Text C

Previous studies have shown a wide variation in anti-SSA prevalence across different RA
populations. It is possible that the increased frequency of anti- SSA in AA subjects may be due to
an increased frequency of secondary Sjogren’s Syndrome. Co-existent RA and SS may then
partially explain the increased disease activity and worse clinical outcomes seen in AA RA
patients. However, it was not possible to determine the prevalence of Sjogren’s Syndrome in our
cohort with the available data. The AA group also had a higher prevalence of anti-SSB than the
CAU group (4.26% vs. 1.08%). This was not a statistically significant difference; however, there
were only 5 total patients that were anti-SSB positive. The biological and clinical implications of
the increased prevalence of anti-SSA and ANA in AA RA patients are currently unknown.
However, several studies have suggested that autoantibody profiles may be clinically significant.
Specifically, antiSSA has been shown to be associated with more severe disease in multiple
connective tissue disease and it is also involved in the molecular pathogenesis of immune
dysregulation in Sjogren’s Syndrome.
Text D

In RA, a predominance of Th17 cytokines, including IFN-γ and TNF have been suggested to be of
pathological importance. IFN-γ induces several chemokines including CXCL9, CXCL10, and CXCL11.
Increased CXCL10 has been detected in the serum and synovial fluid of RA patients and in the
saliva of Sjogren’s Syndrome patients compared to healthy controls. Additionally, this chemokine
may have clinical significance as a human phase II clinical trial using an anti-CXCL10 monoclonal
antibody (MDX-1100) showed a significantly increased response rate in RA patients who had an
inadequate response to methotrexate therapy. The study revealed an association between RF
seropositivity and increased CXCL10 levels but it found no association between antiSSA positivity
and CXCL10. Therefore, while the increased clinical severity seen in AA RA patients may be
associated with a higher prevalence of anti-SSA, the presence of this autoantibody does not
appear to directly affect the expression of CXCL10.
PART A -QUESTIONS AND ANSWER SHEET

Questions 1-7
For each of the questions, 1-7, decide which text (A, B, C or D) the information comes from. You
may use any letter more than once.

In which text can you find information about;


1. Not much difference in data found. Answer ______________
2. Enhanced clinical response observed in AA RA patients. Answer _______________
3. doesn’t create a more effective response with respect to drug. Answer _______________
4. Their name is derived from their ability to induce directed chemotaxis in nearby responsive cells.
Answer ______________
5. Despite differences in autoantibody prevalence, it is currently unknown if there is a higher.
occurrence of clinically diagnosed Sjogren’s Syndrome in AA RA patients. Answer __________
6. The AA group is reported to have a large number of anti-SSB. Answer ________________
7. signaling protein involved in systemic inflammation. Answer _______________

Questions 8-14

Answer each of the questions, 8-14, with a word or short phrase from one of the texts.
Each answer may include words, numbers or both. Your answers should be correctly spelt.
8. What is referred to as chemoattractant, which is induced by IFN-γ? Answer _______________
9. What is recommended to be of more importance pathologically? Answer ________________
10. What do previous studies indicate? Answer _______________
11. What is known to be more connected with the severity of multiple connective tissue disease?
Answer ________________
12. What can work as a disease activity marker in RA? Answer _________________
13. What usually increases in RA patients? Answer________________
14. What does the study suggest with respect to CXCL10? Answer _________________
Questions 15-20

Complete each of the sentences, 15-20, with a word or short phrase from one of the texts.
Each answer may include words, numbers or both. Your answers should be correctly spelt.

15 ___________________levels often go down when target-specific drugs are used.

16 Anti-SSA is found in between ________________ of RA patients

17 _______________can be defined as having or being a positive serum reaction especially in a


test for the presence of an antibody

18 Increased________________ levels are known to be predictive of a favourable response to TNF


inhibitor therapy.

19 The reason for the increased frequency of ______________can be directly linked to enhanced
frequency of secondary Sjogren’s Syndrome.

20 The research performed clearly indicate that there is no association between antiSSA
positivity and________________

END OF PART A, THIS QUESTIONS PAPER WILL BE COLLECTED


READING SUB-TEST : PART B

In this part of the test, there are six short extracts relating to the work of health professionals .
For questions 1-6, choose the answer (A, B or C) which you think fits best according to the text.
Write your answers on the separate Answer Sheet

Questions 1-6

1. The manual talks about;


A. Steps to improving interpersonal communication with patients.
B. How to effectively deal with the patients.
C. Steps with regard to clinician-patient communication.

Important Steps - Dealing With Patients

1. Slow down. Communication can be improved by speaking slowly, and by spending just a small
amount of additional time with each patient. This will help foster a patient centered approach to
the clinician-patient interaction. 2. Use plain, nonmedical language.
Explain things to patients like you would explain them to your grandmother. 3. Show or
draw pictures. Visual images can improve the patient’s recall of ideas. 4. Limit the amount of
information provided— and repeat it. Information is best remembered when it is given in small
pieces that are pertinent to the tasks at hand. Repetition further enhances recall. 5. Use the
“teach-back” technique. Confirm that patients understand by asking them to repeat back your
instructions. 6. Create a shame-free environment: Encourage questions. Make patients feel
comfortable asking questions. Consider using the Ask-Me-3 program. Enlist the aid of others
(patient’s family or friends) to promote understanding.
2. Duodenal duplication;
A. is associated with various anomalies.
B. can occur in two different ways.
C. Is a benign congenital defect, acquired during the embryonic development of the digestive
tract

Duodenal duplication

Duodenal duplication is an extremely rare pathology. It represents 4 % of all digestive tract


congenital malformations. It is often connected with intestinal malrotation, scalloped vertebras).
In one of the cases, the degeneration arose on mucous membrane of gastric type within the cyst of
duplication and in the second it was duodenal mucous membrane.The diagnosis of cancer was
made in both cases on the surgical pieces at anatomopathology. There had been no biopsy within
the cyst. This malformation appears in 70% of the cases before one year of the age but the late
revelation is possible. The digestive obstruction is the most frequent mode of revelation. Acute
pancreatitis was reported and sometimes the diagnosis can be delayed many years. The duodenal
duplication, the anomaly of the embryogenesis, is diagnosed most of the time in the childhood,
even by prenatal diagnosis. The average age at the time of the diagnosis is from four months to
nine years. Prevalence is lightly in favour of the male.
3. Anti-reabsorption medications
A. Comprised of agents which limit the rate of bone loss.
B. Decrease the rate at which osteoclasts resorb bone.
C. Can have a detrimental effect on elderly people.

Anti-reabsorption medications
Presently, anti-reabsorption medications are most widely used for treating osteoporosis.
Zoledronic acid (Aclasta) is a common clinical anti-reabsorption medication. As the third
generation of bisphosphonates (BPs), it outperforms the previous nitrogen-containing BPs in
improving patients’ balance and quality of life by inhibiting bone resorption and increasing bone
mineral density (BMD). It is administered via intravenous drip infusion once a year, which,
therefore, brings good compliance with treatment. However, those who have been treated with
zoledronic acid intravenously are likely to suffer from acute side effects, such as fever, bone and
joint pain and flu-like symptoms, especially after the first administration. Although the common
adverse reactions generally disappear within 72 h, or in rare cases, last 7 to 10 days without
recurrence, they have serious impacts on the aging population, especially those with underlying
diseases. Thus, the patients who are afraid of any adverse reactions or have suffered from any
side effects during the first administration may show poor compliance with the second dose and
refuse the clinical application of zoledronic acid.
4. The given notice talks about;
A. How cancer has successfully been dealt with in todays advanced scientific world.
B. Effective cancer treatment has resulted in many cancer patients.
C. Future course of action.

Cancer - Brief Overview


Historically, individuals with cancer have rarely survived past the latency periods inherent to the
development of treatment-induced malignancies, but improvements in systemic and radiation
therapy have increased the time for these second primaries to emerge. In the past 15 years,
advances in cancer diagnostics, therapy and supportive care have significantly increased disease
free survival and overall survival rates in cancer patients. A proper care plan is reported to have
increased 10 or more years beyond their initial diagnosis and introduced the potential for
secondary cancers induced by therapy. It is possible that the incidence of treatment-induced
cancers may increase in the future. Long-term toxicity of cancer treatments both from systemic
chemotherapy and radiation therapy become an important survivorship concern for patients
and their physicians.
5. What is correct about the surgery performed?
A. On average, hospitalization was 7 days and surgery time was 2 hours.
B. The majority of patients remained hospitalized for 5 days or more
C. performed right knee arthroplasty (TRKA) more on males than females.
6. The table clearly shows that;
A. The majority of the patients were male.
B. People who are aged 60-70 are more in number.
C. The majority of the patients were females.
READING SUB-TEST : PART C
In this part of the test, there are two texts about different aspects of healthcare.
For questions 7-22, choose the answer (A, B, C or D) which you think fits best according to the text.
Write your answers on the separate Answer Sheet

Text 1: Paget's Disease

Paget's disease of the bone is an unusual, chronic bone condition that occurs in only
about 1% of people in the United States and slightly more often in men than in women (3
to 2). Individuals with Paget's disease experience rapid bone repair, which causes a
variety of symptoms from softer bones to enlarged bone growth, typically in the pelvis,
lower back (spine), hips, thighs, head (skull) and arms. Medical therapies have proven
effective in reducing the frequency of pain, fractures and arthritis that may be caused by
this condition. Paget's disease typically occurs in the older population and usually only in
a few of their bones. Bones become large and soft, leading to problems such as bending,
breaking, pinched nerves, arthritis and reduced hearing. Effective and safe treatment
methods can help most people with Paget's disease.

Normally, as people age, their bones rebuild at a slower rate. For those with Paget's
disease, however, this process of rebuilding bones takes place at a faster rate. As a result,
the rebuilt bone has an abnormal structure. The involved bone can be soft, leading to
weakness and bending of the pelvis, lower back (spine), hips, thighs, head and arms. Or,
the rebuilt bone can enlarge, making it more susceptible to arthritis, hearing loss,
fractures and discomfort. Given that this takes place in those over the age of 40, the
symptoms are often mistaken for changes associated with aging.

The cause of Paget's disease is unknown. It does appear to be, at least partially,
hereditary, perhaps when activated by exposure to a virus. Indicative of the hereditary
consideration: Paget's disease occurs more commonly in European populations and their
descendants. In 30% of cases, disease incidents often involve more than one member of a
family. Paget's is rarely discovered in individuals before they reach the age of 40, and the
number of people identified increases in each progressive age group. Typically, it is the
appearance of the bones on an X-ray that prompts the physician to make the diagnosis.
Blood tests taken most often will indicate an increase in serum alkaline phosphatase (SAP), which
is reflective of the rapid new bone turnover. Urine test results will also indicate the speed at
which this rebuilding is taking place. Physicians usually obtain a non-invasive bone scan to
determine the extent of bone involvement. Only if cancer is suspected will it be necessary to
do a bone biopsy to examine it under a microscope.

Treatment approaches can focus on providing physical assistance, including the addition
of wedges in the shoe, canes as walking aids and the administration of physical therapy
(this is considered to be the best). Medications that help reduce the pain associated with
Paget's include acetaminophen (e.g. Tylenol) and anti-inflammatory drugs such as
ibuprofen and naproxen. In addition, a group of medications called bisphosphonates
reduce the pain and help the body regulate the bone-building process to stimulate more
normal bone growth. Your physician may prescribe an oral medication such as
Alendronate (Fosamax) or etidronate (Didronel) to be taken orally every day for 6
months; Tiludronate (Skelid) to be taken orally every day for 3 months, Risedronate
(Actonel) to be taken orally every day for 2 months. All oral medications should be taken
with a large glass of water (6-8 oz) upon rising in the morning. Patients should remain
upright for the next 30 minutes and not eat until that time has passed. Any of these
treatments can be repeated if necessary. Side effects of these medicines may involve
heartburn and sometimes an increase in bone pain for a short period of time.
There are also injectable medications that can be given to a patient for Paget's which
include Pamidronate (Aredia), which is injected into the vein once a month or once every
few months. The injection takes a few hours. Unusually, there is inflammation of the eye
or loss of bone around the teeth (osteonecrosis); Zoledronate (Reclast), which is injected
in the vein once a year. The injection takes less than 30 minutes; Calcitonin, a hormone
that is injected under the skin several times a week. Surgery for arthritis caused by
Paget's disease is effective in reducing pain and improving function. Medical treatment is
not expected to correct some of the changes of the Paget's disease that have already
occurred, such as hearing loss, deformity or osteoarthritis.
Text 1: Questions 7-14

7. Pagets disease;
A. Affects more women than men;
B. Affects 3 men in 4.
C. Affects 3 women in 4.
D. Affects a higher number of men than women.

8. Pagets disease can lead to;


A. Bone disorder.
B. Softening of bones.
C. Arthritis in its final stage.
D. None of the above.

9. Pagets disease is a condition in which;


A. Bones become large and soft.
B. Rebuilding of bones is stopped.
C. Pelvis bends at a faster rate.
D. Rebuilding of bone is accelerated.

10. Reformation of the bones can lead to;


A. Lower back pain, loss of hearing and discomfort.
B. Arthritis, loss of hearing and softening of bone tissues.
C. Fractures and discomfort only.
D. All of the above.

11. Which of the following statements is right as per the given information in the passage?
A. If one member of the family is affected with Pagets disease then other members will also be
affected.
B. If one member of the family is known to have Pagets disease then others can also be affected.
C. Pagets disease is highly hereditary.
D. Pagets disease, is both heritable and inheritable.
12. Pick the correct statement as per the given information in the passage;
A. Those who reach the age of 40 shall undergo blood tests and urine tests for the identification of
Pagets disease.
B. Physicians should always advise the patients to go for a non-invasive bone scan.
C. Increase is indicative of the development of the bone at a rapid speed.
D. It is necessary to do a biopsy of the bone to understand the nature of the disease.

13. What is right about oral medications?


A. May increase bone pain.
B. Should be taken only during the morning.
C. May disturb physiological function.
D. All of the above.

14. Which of the following statements is incorrect as per the given information in the passage?
A. Pagets disease is a condition which can lead to arthritis.
B. Surgery can get rid of Pagets disease.
C. Surgery of arthritis which occurred due to Pagets disease can only reduce the pain associated with it.
D. Medical treatment will not correct a loss in hearing or deformity or osteoarthritis which resulted
from it.
Text 2: Pancreatic Cancer

Cancer is a class of diseases characterized by out-of-control cell growth, and pancreatic


cancer occurs when this uncontrolled cell growth begins in the pancreas. Rather than
developing into healthy, normal pancreas tissue, these abnormal cells continue dividing
and form lumps or masses of tissue called tumors. Tumors then interfere with the main
functions of the pancreas. If a tumor stays in one spot and demonstrates limited growth, it
is generally considered to be benign. More dangerous, or malignant, tumors form when
the cancer cells migrate to other parts of the body through the blood or lymph systems.
When a tumor successfully spreads to other parts of the body and grows, invading and
destroying other healthy tissues, it is said to have metastasized. This process itself is
called metastasis, and the result is a more serious condition that is very difficult to treat.
In the United States each year, over 30,000 people are diagnosed with pancreatic cancer.
Europe sees more than 60,000 diagnosed each year. In Asian countries, numbers can be
even higher than that.
Pancreatic cancer is categorized depending on whether it affects the exocrine or
endocrine functions of the pancreas. There is an important distinction between the two
broad types of pancreatic cancer because they have different risk factors, causes,
symptoms, diagnostic tests, treatments, and prognosis. Tumors that affect the exocrine
functions are the most common type of pancreatic cancer. Sometimes these tumors or
cysts are benign, called cystadenomas. However, it is more likely to find malignant
tumors called adenocarcinomas, which account for 95% of exocrine pancreatic cancers.
Adenocarcinomas typically start in gland cells in the ducts of the pancreas, but they can
also arise from pancreatic enzyme cells (acinar cell carcinoma). Other types of pancreatic
cancers that are associated with exocrine functions include adenosquamous carcinomas,
squamous cell carcinomas, and giant cell carcinomas, named for their appearances
underneath a microscope. There is also a disease called ampullary cancer (carcinoma of
the ampulla of Vater) that starts where the bile duct and pancreatic duct meet the
duodenum of the small intestine.
Cancer is ultimately the result of cells that grow uncontrollably and do not die. Normal
cells in the body follow an orderly path of growth, division, and death. Programmed cell
death is called apoptosis, and when this process breaks down, cancer occurs. Pancreatic
cancer cells do not experience programmatic death, but instead, continue to grow and
divide. Although scientists do not know exactly what causes these cells to behave in this
way, they have identified several potential risk factors. Cancer can be the result of a
genetic predisposition that is inherited from family members. It is possible to be born
with certain genetic mutations or a fault in a gene that makes one statistically more likely
to develop cancer later in life.
About 10% of pancreatic cancers are thought to be caused by inherited gene mutations.
Genetic syndromes that are associated with pancreatic cancer include hereditary breast
and ovarian cancer syndrome, melanoma, pancreatitis, and non-polyposis colorectal cancer
(Lynch syndrome).

Carcinogens are a class of substances that are directly responsible for damaging DNA,
promoting or aiding cancer. Certain pesticides (dyes may also be included in this list
here), and chemicals used in purification of the metal are thought to be carcinogenic,
increasing the risk of developing pancreatic cancer. When our bodies are exposed to
carcinogens, free radicals have formed that try to steal electrons from other molecules in
the body. These free radicals damage cells, affecting their ability to function normally,
and the result can be cancerous growths. As we age, there is an increase in the number of
possible cancer-causing mutations in our DNA. This makes age an important risk factor
for pancreatic cancer, especially for those over the age of 60. There are several other
diseases that have been associated with an increased risk of cancer of the pancreas.
These include cirrhosis or scarring of the liver, Helicobacter pylori infection (infection of
the stomach with the ulcer-causing bacteria H. pylori), diabetes mellitus, chronic
pancreatitis (inflammation of the pancreas), and gingivitis or periodontal disease.
In order to detect pancreatic cancer, physicians will request a complete physical
examination as well as personal and family medical histories. The way in which cancer
presents itself will differ depending on whether the tumor is in the head or the tail of the
pancreas. Tail tumors present with pain and weight loss while head tumors present with
steatorrhea, weight loss, and jaundice. Doctors also look for recent onset of atypical
diabetes mellitus, Trousseau's sign, and recent pancreatitis. In general, when making a
pancreatic cancer diagnosis, physicians pay special attention to common symptoms
such as abdominal or back pain, weight loss, poor appetite, tiredness, irritability,
digestive problems, gallbladder enlargement, blood clots (deep venous thrombosis
(DVT) or pulmonary embolism), fatty tissue abnormalities, diabetes, swelling of lymph
nodes, diarrhea, steatorrhea, and jaundice.
Text 2: Questions 15-22

15. Pancreatic tumors can be;


A. Benign
B. Malignant
C. Benign and malignant
D. None

16. Metastasized is a condition


A. When pancreatic tumors spread to other parts of the body.
B. When malignant tumors form in the pancreas.
C. When harmful tumors invade and destroy other healthy tissues of the body.
D. When tumors are grown automatically in other parts of the body.

17. Pancreatic cancer is most commonly associated with.


A. Endocrine gland
B. Exocrine gland
C. Cystadenomas
D. Adenocarcinomas

18. Ampullary cancer is associated with.


A. Endocrine gland
B. Exocrine gland
C. Pancreatic duct
D. None

19. Failure of apoptosis results in;


A. Pancreatic cell growth
B. Enlargement of the pancreatic duct
C. Cancerous tissues in the pancreas
D. B & C
20. A cancerous growth in the pancreas is a result of;
A. Specific carcinogens
B. Family Genes
C. Genetic mutations
D. None

21. The risk of pancreatic cancer is associated with these carcinogens;


A. Pesticides and dyes
B. Pesticides, dyes and chemicals used for refining metals
C. Only dyes
D. None

22. One of these is not a symptom associated with PC;


A. Back pain and problems with digestion
B. Digestive problems and blood clotting
C. Pulmonary edema and enlargement of the gallbladder
D. Jaundice and modification in the lymph

END OF READING TEST, THIS BOOKLET WILL BE COLLECTED


Reading test 5 : Answer Key

Part A - Answer key 1 – 7


1 .C
2 .D
3 .A
4 .B
5 .A
6 .C
7 .B

Part A - Answer key 8 – 14


8 .CXCL9
9 .Th17 cytokines
10 .variation in anti-SSA prevalence
11 .Anti SSA
12 .CXCL10
13 .serum chemokines
14 .relation between RF seropositivity & CXCL10 levels

Part A - Answer key 15 – 20


15.Serum Chemokine
16 .3-16%
17.Correct Answer Is: Seropositivity
18 .Cxcl10 And Cxcl13
19 . Anti- Ssa In Aa
20 .Cxcl10
Reading test - Part B : Answer Key
1 .Steps to improving interpersonal communication with patients.
2 .is associated with various anomalies.
3 .Can have a detrimental effect on elderly people.
4 .Future course of action.
5 .The majority of patients remained hospitalized for 5 days or more
6 .The majority of the patients were females.

Reading test - Part C : Answer Key

Text 1 - Answer key 7 – 14


7. Affects a higher number of men than women.
8. Arthritis in its final stage.
9. Rebuilding of bone is accelerated.
10. Lower back pain, loss of hearing and discomfort.
11. Correct Answer Is: Pagets disease, is both heritable and inheritable.
12. Increase is indicative of the development of the bone at a rapid speed.
13. Should be taken only during the morning.
14. Correct Answer Is: Surgery can get rid of Pagets disease.

Text 2 - Answer key 15 – 22


15. Benign and malignant
16. When harmful tumors invade and destroy other healthy tissues of the body.
17. Exocrine gland
18. Exocrine gland
19. Cancerous tissues in the pancreas
20. Genetic mutations
21. Pesticides, dyes and chemicals used for refining metals
22. Pulmonary edema and enlargement of the gallbladder
READING TEST 6

READING SUB-TEST : PART A


 Look at the four texts, A-D, in the separate Text Booklet.

 For each question, 1-20, look through the texts, A-D, to find the relevant information.

 Write your answers on the spaces provided in this Question Paper.


 Answer all the questions within the 15-minute time limit.
 Your answers should be correctly spelt.

TEXT BOOKLET - SYSTEMIC SCLEROSIS (SSC)

Text A
Systemic sclerosis (SSc)
Systemic sclerosis (SSc) is a disorder of the connective tissue characterized by fibrosis of
the skin, vascular abnormalities, and presence of autoantibodies. It is characterized by
excessive deposition of extracellular matrix. Therefore, there is significant heterogeneity in
organ progression and prognosis. Interstitial lung disease (ILD) is a heterogeneous group of
parenchymal lung disorders that share common radiologic, pathologic, and clinical
manifestations. It is characterized by lung parenchyma damage, accompanied by
inflammation and fibrosis, and fibrosis is often incurable. The fibrosing forms of ILD are
often incurable, and are associated with significant morbidity and mortality. SSc is often
accompanied by ILD. The incidence of SSc-ILD in the relevant literature ranges from 45%
to 90%. A recent European League Against Rheumatism Scleroderma Trials and Research
analysis revealed in a cohort of 3,656 SSc patients that ILD was present in 53% of cases
with diffuse cutaneous SSc and in 35% of cases with limited cutaneous SSc.
Text B

NSIP is the more common subtype of inflammation in ILD. 77% of SSc-ILD is NSIP. A
large number of clinical and pathological studies have confirmed that a high-resolution CT
(HRCT) pattern in patients is correlated with pathologic NSIP and pathologic UIP. NSIP
pattern is associated with better patient outcome than UIP pattern. It includes reticular,
frosted glass shadows, hollow, thickened honeycomb lung nodules, emphysema, bronchial
vascular bundles, bullae, traction bronchiectasis, cobblestone-likeappearance, bronchial tree,
bronchiectasis and so on. The most common manifestation of NSIP is lobular reticular
formation in the pleural and basal regions. UIP is mainly represented by grid or honeycomb
shadow. Different patterns in HRCT can reflect NSIP and UIP. The extent of ILD lesions can be
graded according to HRCT. At present, the commonly used methods for clinical detection of ILD
are HRCT, pulmonary function tests (PFTs) (react as per sensitivity), bronchoalveolar lavage
fluid (BALF), lung biopsy. HRCT has now become the most common and sensitive imaging
method for diagnosing ILD as it offers the most detailed images of the lungs.
Text C
KL-6
Krebs von den Lungen-6 (KL-6) is an important serum marker for ILD. It is a high molecular
weight, mucin-like glycoprotein secreted by type-II alveolar pneumocytes and bronchial
epithelial cells in response to cellular damage and regeneration in patients with ILD. KL-6 is a
mucin-associated glycoprotein, which may be a trigger for TGF-β signaling and fibrosis. The
level of KL-6 as a predictive factor could be used to identify the clinical development of ILD.
Hideaki et al retrospectively analyzed the medical records of 29 patients with SSc-ILD. They
found serum KL-6 correlated positively with diffusing capacity of the lung for carbon monoxide
(DLCO)(% predicted) and disease extent on HRCT, and the changes in serum levels of KL-6 were
significantly related to the changes in forced vital capacity (FVC) in SSc -associated ILD. Their
study suggests KL-6 can be a useful monitoring tool of SSc-ILD activity.

Text D
SP-D
Surfactant, a lipoprotein complex, was originally described for its essential role in reducing
surface tension at the air-liquid interface of the lung. However, it is now recognized as being a
critical component in lung immune host defense. They include SP-B and SP-C and hydrophilic
proteins SP-A and SP-D . SP-D levels are more sensitive than SP-A in detecting ILD as defined by
CT. The sensitivities and specificities for detecting CT-positive ILD in 42 patients with SSc were
33% and 100% for SP-A and 77% and 83% for SP-D, respectively. In a small but prospective
study of 35 patients withSSc-ILD followed over 1-10 years, SP-D levels were seen to definitely
increase over time in 9 out of the 10 patients with worsening ILD, as defined by changes in
symptoms, lung function, and imaging, compared to mild increases in only 3 out of 25 patients
with stable or improving SSc-ILD. Therefore, SP-D is closely related to SSc-ILD. In addition,
Takahashi H et al. found a less-invasive and lung-specific clinical biomarker. They found the
levels of SP-D in sera were significantly higher in the CT-positive ILD group than in the CT-
negative ILD group.
PART A -QUESTIONS AND ANSWER SHEET

Questions 1-7
For each of the questions, 1-7, decide which text (A, B, C or D) the information comes from.
You may use any letter more than once.
In which text can you find information about;
1. One of the major aspects of assessment of the how ILD may develop.________________
2. Detection or assessment of the ILD lesions._________________________________
3. Talk of naturally occurring molecule, gene, or characteristic by which a particular pathological or
physiological process, disease, etc. can be identified.____________
4.Common features of the disease._________________________
5. Production of the protein in response to damage to cell______
6. SSc more commonly occurs with the ILD______.
7. A more common form of interstitial lung disease_____________________________

Questions 8-14
Answer each of the questions, 8-14, with a word or short phrase from one of the texts.
Each answer may include words, numbers or both. Your answers should be correctly spelt.

8. What are known to be more sensitive to change?________________________


9. What is known to be closely related to SSc-ILD?______________________
10. Which is an important serum marker for interstitial lung disease? __________________
11. What are the major characteristics of UIP? _______________
12. What is known to play a major role with respect to lung immune host defense? ___________
13. What can be considered a perfect monitoring tool of SSc-ILD activity? _____________
14. Which lipoprotein complex plays an essential role in reducing surface tension at the air-liquid
interface of the lung? Answer ___________
Questions 15-20

Complete each of the sentences, 15-20, with a word or short phrase from one of the texts. Each
answer may include words, numbers or both. Your answers should be correctly spelt.

Each answer may include words, numbers or both.

15_.__________ is a form of lung disease characterized by progressive scarring of both lungs

16._________ are recorded to have shown a remarkable increase with more deteriorating ILD

conditions.

17. As per HRCT, it can be easy to analyze the extent of the ___________

18. The sensitivities in detecting CT-positive ILD was _____________ SP-D.

19 .The surfactant may include__________ and hydrophilic proteins.

20 .There are four major methods for clinical detection of __________

END OF PART A, THIS QUESTIONS PAPER WILL BE COLLECTED


READING SUB-TEST : PART B
In this part of the test, there are six short extracts relating to the work of health professionals.
For questions 1-6, choose answer (A, B or C) which you think fits best according to the text.
Write your answers on the separate Answer Sheet.

Questions 1-6

1. What does this notice explain?

A. Effects of obesity.
B. Effects of weight loss.
C. How weight loss is correlated to other diseases

1. Obesity - Weight Loss


The relationship between obesity, weight loss, and disease control has been investigated
among patients already diagnosed with chronic diseases. Health benefits of weight loss in chronic
diseases include improvements in cholesterol levels, decreased risk of cardiovascular events, and
improved quality of life. Among patients with hypertension, patients that lost ≥5 kg had improved
blood pressure. Weight loss of ≥5% was associated with improved glycemic control in patients with
diabetes mellitus. In patients with osteoarthritis, weight loss of ≥5% was associated with
improvement in joint symptoms. Weight loss of ≥5% was also associated with low/remission
disease activity in patients with psoriatic arthritis.
2. Hemophagocytic lymphohistiocytosis.
A. Is a rare disease.
B. Is potentially fatal.
C. Occurs in all ages.

Hemophagocytic lymphohistiocytosis
Hemophagocytic lymphohistiocytosis (HLH) is the clinical manifestation of a wide array of
different entities, which include primary or familial hemophagocytic lymphohistiocytosis (FHLH)
and secondary forms and can lead to deteriorating conditions and eventually loss of proper body
mechanism. The hallmark is hemophagocytosis, appearance of activated macrophages that have
engulfed other haematopoietic elements. FHLH, mainly documented in early infancy, is related to
familiar inheritance or genetic causes. Secondary forms (SHLH), also called reactive HLH, are
frequently diagnosed in adults and refer to cases with underlying infection, malignancy or
autoimmune disease. Over the last decade immunosuppression, immunodeficiency,
autoinflammatory diseases and inborn errors of metabolism have been also described as
triggering diseases. Macrophage activation syndrome (MAS) is a secondary form, recently
reported in patients with autoimmune or autoinflammatory diseases, especially patients with
systemic juvenile idiopathic arthritis (SJIA).
3. Granulomatousa Lymphocytic Interstitial Lung Disease;
A. Complication of common variable immunodeficiency disorders.
B. Can effectively be cured by using drugs that can bring in great change in immune response.
C. Appears majorly due to activity of T and B lymphocytes

Granulomatous– Lymphocytic Interstitial Lung Disease (GLILD)

Granulomatous– Lymphocytic Interstitial Lung Disease (GLILD) is an inflammatory pulmonary


complication of common variable immunodeficiency (CVID) with distinctive patterns in the
biopsy; granulomatous disease, lymphocytic interstitial disease, follicular bronchiolitis and areas
of organizing pneumonia. The immunological data of the pathophysiology suggests that it is
initiated by an infiltration of T as well as B lymphocytes and macrophages, that will further lead
the progression of the inflammatory process to fibrosis. Regarding the treatment of this disease
there are many immunomodulatory treatments with few standardized protocols, but recent
studies suggest that the combination of Rituximab and Azathioprine could be effective for
preserving the pulmonary function .
4. What is correct about MCP?
A. MCP-1 may play an important role in the development of pulmonary fibrosis in SSc.
B. The abnormal accumulation of macrophages will lead to the production of MCP-1
C. There is not much evidence to show that ILD is directly interlinked to collagen.CCL2 is known
as monocyte c

hemoattractant protein-1 (MCP-1). MCP-1 is a member of the C-C chemokines. In vivo studies
suggest that MCP-1 recruits monocytes/ macrophages to sites of inflammation in a wide
variety of pathological conditions, including ILD. The plasma level of CCL2 is correlated with
FVC value in SSc. However, there was no correlation between ILD severity and primary fibrotic
genes such as collagen. That might be because skin fibrosis peaks early during the course of SSc
and improves later, while fibrosis in pulmonary tissue continues to progress even at later
stages of disease. A study examined serum levels, spontaneous production by peripheral blood
mononuclear cells (PBMC), and histological distribution in the affected skin, of MCP-1. Elevated
serum levels of MCP-1 significantly correlated with the presence of pulmonary fibrosis. MCP-1
was expressed in mononuclear cells or vascular endothelial cells in 41% (9/22) of SSc patients.
The frequency of infiltrating mononuclear cells and endothelial cells that produced MCP-1 was
significantly higher in SSc patients with early onset than in SSc patients with late onset.
5.What information does this table provide?
A. Shows the clinical decision outcomes of the FRAX 10-year Hip and Major Osteoporotic
fracture risk score thresholds.
B. The thresholds based on the FRAX MOF risk score with DXA.
C. The thresholds based on the FRAX HF risk score with or without DXA.
6. The table clearly shows that;
A. there are high differences in serum CXCL10 concentration between SSA positive and SSA
negative subjects.
B. The RF-positive group had significantly elevated score.
C. The RF-positive group had an average CXCL10 concentration for the RF-negative group.
READING SUB-TEST : PART C

In this part of the test, there are two texts about different aspects of healthcare.
For questions 7-22, choose the answer (A, B, C or D) which you think fits best according to the text.
Write your answers on the separate Answer Sheet.

Text 1: Classification of Seizures


In order to communicate about types of seizures, epilepsy specialists have developed a
classification system for seizures. This system is not based on any fundamental property of
seizures, but rather on committee-generated conventions of terminology.
Classification is as follows: partial seizures and generalized seizures.
Partial seizures are further divided into simple partial seizures with no alteration of consciousness
or memory, or complex partial seizures with alteration of consciousness or memory.
Simple partial seizures can be motor seizures with twitching, abnormal sensations, abnormal
visions, sounds or smells, and distortions of perception. Seizure activity can spread to the
autonomic nervous system, resulting in flushing, tingling, or nausea. If the patient becomes
confused or cannot remember what is happening during the seizure, then the seizure is classified
as a complex partial seizure. Previously, they were called “psychomotor seizures”, “temporal lobe
seizures” or “limbic seizures”. During the complex partial seizure, patients may fumble or perform
automatic fragments of activity such as lip smacking, picking at their clothes, walking around
aimlessly, or saying nonsense phrases over and over again; these purposeless activities are called
automatisms. About 75% of people with complex partial seizures have automatisms; those who do
not simply stop stare and blank out for a few seconds or minutes.

Generalized seizures are divided into absence seizures and tonic-clonic seizures. Absence seizures
were previously called petit mal seizures and usually have onset in childhood, but they can persist
into adulthood. Absence seizures present with staring spells lasting several seconds, sometimes in
conjunction with eyelid fluttering or head nodding. These seizures can be difficult to distinguish
from complex partial seizures that may also result in staring. Usually, absence seizures are briefer
and permit quicker recovery. Generalized tonic-clonic seizures were previously called grand mal
seizures; these seizures start with sudden loss of consciousness and tonic activity (stiffening)
followed by clonic activity (rhythmic jerking) of the limbs. The patient’s eyes will roll up at the
beginning of the seizure and the patient will typically emit a cry, not because of pain, but because

of contraction of the respiratory muscles against a closed throat. Generalized tonicclonic seizures
usually last one to three minutes.

Seizures that begin focally can spread to the entire brain, in which case a tonic-clonic seizure
ensues. It is important, however, to distinguish those that are true grand mal, generalized from the
start, from those that start focally and secondarily generalize. Secondarily generalized seizures
arise from a part of the brain that is focally abnormal. Drugs used to treat primary and secondary
generalized tonic-clonic seizures are different: patients with secondarily generalized tonic-clonic
seizures may be candidates for curative epilepsy surgery, whereas primarily generalized tonic-
clonic seizures are not surgical candidates, because there is no seizure origin site (focus) to remove.
Atonic seizures are epileptic drop attacks. Atonic seizures typically occur in children or adults with
widespread brain injuries. People with atonic seizures suddenly become limp and may fall to the
ground and football helmets are sometimes required to protect against serious injuries. A
myoclonic seizure is a brief un-sustained jerk or series of jerks, less organized than the rhythmic
jerks seen during a generalized tonic-clonic seizure. Other specialized seizure types are occasionally
encountered. Tonic seizures involve stiffening of muscles as the primary seizure manifestation:
arms or legs may extend forward or up into the air; consciousness may or may not be lost. By
definition, the clonic (jerking) phase is absent. Classification can be difficult, because stiffening is a
feature of many complex partial seizures. Tonic seizures, however, are much less common than
complex partial or tonic-clonic seizures. Patients can have more than one seizure type. One seizure
type may progress into another as the electrical activity spreads throughout the brain. A typical
progression is from a simple partial seizure, to a complex partial seizure (when the patient
becomes confused), to a secondarily generalized tonic-clonic seizure (when the electrical activity
has spread throughout the entire brain). The brain has control mechanisms to keep seizures
localized. Antiepileptic medications enhance the ability of the brain to limit the spread of a seizure.
Part C -Text 1: Questions 7-14

7. Motor seizures are;


A. Simple partial seizures
B. Partial seizures
C. Complex seizures
D. Complex partial seizures

8. In which type of seizure does the patient generally not remember what is happening around
them?
A. Simple partial
B. Complex partial
C. Complex partial seizures
D. Partial temporal lobe seizures

9. Which one of these activities are related to automatism?


A. Fumbling
B. lib smacking
C. speaking leisurely
D. None

10. 10 Which seizures last for one to three minutes?


A. Simple partial seizures
B. Tonic-clonic seizures
C. Absence seizures
D. None
11. Which type of seizure occurs in childhood and may persist into adulthood?
A. Grand mal seizures
B. Petit mal seizures
C. Both A and B
D. None

12. Which seizures arise from a focally abnormal part of the brain?
A. Petit mal seizures
B. Grand mal seizures
C. Secondarily generalized seizures
D. Both B and C

13. As per the given information, who may undergo surgery?


A. Patients with grand mal seizures
B. Patients with secondarily generalized seizures
C. Patients with primarily generalized tonic-clonic seizures
D. Both B and C

14. Which one of the following statements correctly describes tonic seizures?
A. Rhythmic jerking
B. Stiffening of muscles
C. Loss of consciousness
D. None
Part C -Text 2: Fascioliasis Infection

Fascioliasis is a parasitic infection typically caused by Fasciola hepatica, which is also known as "the
common liver fluke" or "the sheep liver fluke." A related parasite, Fasciola gigantica, can also infect
people. Fascioliasis is found in all 5 continents, in over 50 countries, especially where sheep or
cattle are reared. People usually become infected by eating raw watercress or other water-based
plants contaminated with immature parasite larvae. The immature larval flukes migrate through
the intestinal wall, the abdominal cavity, and the liver tissue, into the bile ducts, where they
develop into mature adult flukes, which produce eggs. Typically, the pathology is most pronounced
in the bile ducts and liver. A Fasciola infection is both treatable and preventable.

The standard way to be sure a person is infected with Fasciola is by seeing the parasite - this is
usually done by finding Fasciola eggs in stool (fecal) specimens examined under a microscope.
More than one specimen may need to be examined to find the parasite.
Sometimes eggs are found by examining duodenal contents or bile. Infected people don't start
passing eggs until they have been infected for several months; people don't pass eggs during the
acute phase of the infection. Therefore, early on, the infection has to be diagnosed in other ways
than by examining stool. Even during the chronic phase of infection, it can be difficult to find eggs
in stool specimens from people who have light infections.

Fasciola parasites develop into adult flukes in the bile ducts of infected mammals, which pass
immature Fasciola eggs in their feces. The next part of the life cycle occurs in freshwater. After
several weeks, the eggs hatch, producing a parasite form known as the miracidium, which then
infects a snail host. Under optimal conditions, the development process in the snail may be
completed in 5 to 7 weeks; cercariae are then shed in the water around the snail. The cercariae
lose their tails when they encyst as metacercariae (infective larvae) on water plants. In contrast to
cercariae, metacercariae have a hard outer cyst wall and can survive for prolonged periods in wet
environments.
Immature Fasciola eggs are discharged in the biliary ducts and in the stool. Eggs become
embryonated in water; eggs release miracidia, which invade a suitable snail intermediate host,
including the genera Galba, fossaria and pseudosuccinea. In the snail the parasites undergo several
developmental stages: sporocysts, rediae, and cercariae. The cercariae are released from the snail
and encyst as metacercariae on aquatic vegetation or othersurfaces. Mammals acquire the
infection by eating vegetation containing metacercariae whereas humans can become infected by
ingesting metacercariae-containing freshwater plants, especially watercress. After ingestion, the
metacercariaeexcyst in the duodenum and migrate through the intestinal wall, the peritoneal
cavity, and the liver parenchyma into the biliary ducts, where they develop into adult flukes.

No vaccine is available to protect people against Fasciola infection. In some areas of the world
where Fascioliasis is found (endemic), special control programs are in place or are planned. The
types of control measures depend on the setting (such as epidemiologic, ecologic, and cultural
factors). Strict control of the growth and sale of watercress and other edible water plants is
important. Individual people can protect themselves by not eating raw watercress and other water
plants, especially from endemic grazing areas. As always, travelers to areas with poor sanitation
should avoid food and water that might be contaminated (tainted). Vegetables grown in fields that
might have been irrigated with polluted water should be thoroughly cooked, as should viscera from
potentially infected animals

In the early (acute) phase, symptoms can occur as a result of the parasite's migration from the
intestine to and through the liver. Symptoms can include gastrointestinal problems such as nausea,
vomiting, and abdominal pain/tenderness. In addition, fever, rashes and difficulty breathing may
occur. During the chronic phase (after the parasite settles in the bile ducts), the clinical
manifestations may be similar or more discrete, reflecting inflammation and blockage of bile ducts,
which can be intermittent. Inflammation of the liver, gallbladder and pancreas can also occur.
Part C -Text 2: Questions 15 – 22

15. Which one of the following statements is correct?


A. Infection caused by Fasciola spreads faster than any other types of infections
B. Infection by Fasciola is deadly
C. Infection by Fasciola is treatable
D. Infection by Fasciola is very common

16. In which phase is it not easy to find the eggs in the stool?
A. Chronic phase
B. Infective phase
C. Acute phase
D. A and B

17. Paragraph 3 talks about which of the following;


A. Biology of Fasciola hepatica
B. Time period in a snail
C. Initial stages of the development of the parasite
D. Complete life cycle

18. Which of these forms survives for a longer period of time?


A. Cercariae
B. Metacercariae
C. Miracidia
D. Fasciola eggs

19. Which of these topics does paragraph 4 talk about?


A. How infection occurs in humans
B. How animals get infected
C. How plants get infected
D. All of the above
20. Excystation occurs in which of these?
A. AIntestinal wall
B. Duodenum
C. Peritoneal cavity
D. Liver

21. Paragraph 5 talks about which of these topics?


A. Prevention and control
B. Availability of the treatment for the infection
C. Drugs to be used
D. A and C

22 .Which of these topics does paragraph 6 talk about?


A. How infection is controlled
B. How infection spreads through bile ducts and liver
C. How infection is prevented from spreading to different parts
D. Possibility of infection spreading to other parts of the body

END OF READING TEST, THIS BOOKLET WILL BE COLLECTED


Reading test 6 : Answer Key

Part A - Answer key 1 – 7


1. C
2.B
3. D
4. A
5. C
6. A
7. B

Part A - Answer key 8 – 14


8. HRCT and PFTs
9. Sp-d
10. KL-6
11. grid or honeycomb shadow
12. Surfactant
13. KL-6
14. Surfactant

Part A - Answer key 15 – 20


15. Usual Interstitial Pneumonia (uip)
16. Sp-d Levels
17. Ild Lesions
18. 83%
19. Sp-b And Sp-c
20. Ild
Reading test - Part B: Answer Key

1. 1.How weight loss is correlated to other diseases.


2. Is potentially fatal.
3. 3.Can effectively be cured by using drugs that can bring in great change in immune response.
4. MCP-1 may play an important role in the development of pulmonary fibrosis in SSc
5. Shows the clinical decision outcomes of the FRAX 10-year Hip and Major Osteoporotic fracture
risk score thresholds.
6. 6.The RF-positive group had an average CXCL10 concentration for the RF-negative group.

Reading test - part C – answer key

Text 1 - Answer key 7 – 14


7.Simple partial seizures
8 Complex partial seizures
9.Lip smacking
10.Tonic-clonic seizures
11.Petit mal seizures
12. Secondarily generalized seizures
13. Patients with secondarily generalized seizures
14.Loss of consciousness

Text 2 - Answer key 15 – 22


15. Infection by Fasciola is treatable
16.Chronic phase
17.Initial stages of the development of the parasite
18.Metacercariae
19. How animals get infected
20 Duodenum
21.Prevention and control
22. Possibility of infection spreading to other parts of the body
READING TES 7
READING SUB-TEST : PART A
 Look at the four texts, A-D, in the separate Text Booklet.
 For each question, 1-20, look through the texts, A-D, to find the relevant information.
 Write your answers on the spaces provided in this Question Paper.
 Answer all the questions within the 15-minute time limit.
 Your answers should be correctly spelt.
TEXT BOOKLET - GALECTIN-3

Text A

Galectin-3
It is well known that there is a close relation between obesity-induced insulin
resistance, immune cells accumulation in white adipose tissue (WAT) and
inflammation. Indeed, in obesity WAT is characterized by an increased production
and secretion of a wide range of inflammatory cytokines including TNF-alpha and
interleukin (IL)- 6, which may have local effects on endothelial, vasculature and
target adipose tissues. Therefore, activated macrophages and other antigen
presenting cells that are accumulated in elevated number in fat tissue in both types
of obese actively secrete a broad spectrum of locally produced pro-inflammatory
cytokines including galectin-3 (Gal-3). Gal-3 is a beta-galactoside-binding lectin
belonging to a multifunctional protein family, which enhances chemotaxis of
immune and antigen presenting cells, reduces insulin-stimulated glucose uptake in
myocytes and adipocytes and impairs insulin-mediated suppression of glucose
output in hepatocytes. Gal-3 may bind directly to the insulin receptor (IR) and
thereby inhibit downstream insulin resistance signaling via diminishing
interleukin-1 beta production. Therefore, Gal- 3 is a modulator of apoptosis,
necrosis and fibrosis associated with extracellular remodeling.
Text B

Gal-3 is increased in obesity and mediates inflammation and fibrosis in the heart
and vessels, as well as in the WAT. The most preclinical and clinical studies
suggest that this protein protects from inflammation in obese, while there is a large
body of evidence regarding the ability of Gal-3 to deteriorate glucose homeostasis,
modulate cell adhesion and induce pro-oxidant pathways. Interestingly, the low
serum Gal-3 concentrations are closely associated with insulin resistance in
patients with type 2 diabetes mellitus. In contrast, an inverse correlation between
serum Gal-3 and glycosylated hemoglobin in type 2 diabetes mellitus was
found. In clinical settings Gal-3 strongly independently predicts all-cause
mortality and CV mortality in the general population and in patients with known
CV disease. In fact, in cross-sectional analyses of 2946 Framingham Heart Study
participants circulating Gal-3 was associated well with abdominal adiposity,
dyslipidemia, and hypertension, but Gal-3 did not predict incident CV and
metabolic diseases after adjusting for cardiometabolic risk factors. Whether Gal-3
could be a predictive marker of the metabolically unhealthy obese is not clear,
although Gal-3 deserves further large clinical trials to understand its role in
different obese phenotypes’ development.
Text C

Natriuretic peptides (NPs) are “cardiometabolic” hormones with well-established


cardiovascular, renal, and endocrine abilities affecting sodium reabsorption and
blood pressure regulation. Although, NP levels are markers of biomechanical
cardiac stress, their role in the nature of the evolution of obese is not fully
understood. These controversies affect the clearance of NPs in obese and
pathophysiological mechanisms controlling the synthesis of them. While, NP
secretion result in a stretch of the cardiac wall and volume overload of cardiac
cavities. On the other hand, recent epidemiological and preclinical/ clinical studies
have shown that the NP system acts as a deficiency in obese patients is due to
worse clearance of NP receptors and neutral endopeptidases. Consequently, NP
system in obese is not able to mediate a wide spectrum of cardiovascular and
metabolic protective effects (i.e., vasodilation, natriuresis, diuresis, lipolysis, weight loss,
lusitropy, lipid peroxidation, and also improve mitochondrial respiration and insulin
sensitivity).

Text D
Brutsaert et al. have reported that higher levels of brain NP have associated with
decreased risk of diabetes in middle-aged adults and that the interrelation has
remained after adjustment for waist circumference, low physical activity, estimated
glomerular filtration rate and high sensitive C-reactive protein level. In contrast, it
is suggested that the low brain NP levels observed in obesity could causally
associated with the incidence of diabetes in obese individuals. The effect of brain
NPs might relate to an ability of natriuretic peptides to activate a thermogenic
program in brown and white fat tissues, increase energy expenditure and inhibit
food intake. Thus, NPs might play several metabolic roles in the development of
different phenotypes of obesity, but their predictive role in CV disease
development in obese patients is uncertain.
PART A -QUESTIONS AND ANSWER SHEET

Questions 1-7
For each question, 1-7, decide which text (A, B, C or D) the information comes from.
You may use any letter more than once.

1. Role of this metabolic hormones is not yet certain. Answer _____________

2 .Perform its role when there is a deficiency. Answer___________

3. Higher and lower level of the metabolic hormones. Answer ____________

4. Known to affect the interior surface of blood vessels and lymphatic vessels. Answer _______

5. Worsening relative sugar equilibrium. Answer _____________

6 .Introduction to carbohydrate-binding lectin whose expression is associated with inflammatory


cells. Answer ____________

7 .May have a direct impact on hemoglobin to which glucose is bound. Answer _____________

Questions 8-14
Answer each of the questions, 8-14, with a word or short phrase from one of the texts.
Each answer may include words, numbers or both. Your answers should be correctly spelt.

8. What defines the rate of myocardial relaxation? Answer___________

9. What are more associated with insulin resistance? Answer ___________

10. What can have the potential to have direct impact on insulin resistance? Answer ____

11. What defines an abnormal amount of lipids? Answer _____________

12. What are the markers of biomechanical cardiac stress? Answer______________

13 .What is associated with programmed cell death? Answer___________

14. What causes the cardiac wall to stretch and volume overload of cardiac cavities? Answer__
Questions 15-20

Complete each of the sentences, 15-20, with a word or short phrase from one of the texts.
Each answer may include words, numbers or both. Your answers should be correctly spelt.

15. ___________ can be associated with a significantly increased risk for coronary heart disease.

16 .higher levels of brain NP can be connected with the decreased risk of___________

17 .The term_____________ is referred to the thickening and scarring of connective tissue, usually as
a result of an injury.

18. It is still not clear that Gal-3 is a ______________ of the metabolically unhealthy obese

19. Excretion of sodium in the urine is a common condition and is called ______________

20. NPs may not mediate __________ of lipids.

END OF PART A, THIS QUESTIONS PAPER WILL BE COLLECTED


READING SUB-TEST : PART B
In this part of the test, there are six short extracts relating to the work of health professionals .
For questions 1-6, choose the answer (A, B or C) which you think fits best according to the text.
Write your answers on the separate Answer Sheet
Questions 1-6

1. What is correct about liver toxicity from sulfasalazine?


A. Hepatotoxicity can arise either from direct toxicity of the drug or its metabolites.

B. In rare cases, it can be immune mediated.

C. Immune-mediated toxicity is believed to be due to an idiosyncratic delayed type


hypersensitivity reaction.

Liver toxicity from sulfasalazine

Liver toxicity from sulfasalazine is a rare but serious side effect. It can range from
mild elevation in LFTs to hepatic failure and cirrhosis. The occurrence of severe
liver toxicity such as acute hepatitis as seen in our patient is <1%. After reaching
the gut, sulfasalazine is broken down by the colonic bacteria into its metabolites,
i.e., sulfapyridine and 5-aminosalicylic acid. Sulfapyridine is absorbed in the gut
and eliminated after acetylation by enzyme N-acetyltransferase which can have
variable activity based on the patient's genotype. Patients who have genotypes for
slow acetylation are found to be more predisposed to sulfasalazine-induced liver toxicity.
Injury can be hepatocellular which presents with disproportionate elevation in serum
aminotransferases or cholestasis which presents with disproportionate elevation in
alkaline phosphatase. Both patterns of liver injury can have elevation in bilirubin and
abnormal tests for liver synthetic function.
2. Adiponectin;
A. Is a protein hormone which is involved in regulating glucose levels.

B. Can have positive impact on CAD.

C. Plays a role in the development of insulin resistance.

Adiponectin

Adiponectin accounts for 0.01% of plasma protein with a half-life of 2.5 h; normal
adiponectin plasma level is 5-10 µg/mL with higher levels in females than males
due to sexual dimorphism. Adiponectin plasma forms are of two types, highmolecular-
weight and low-molecular-weight. In addition, high-molecular-weight adiponectin levels
are positively associated with CAD and negatively associated with risk of type 2 diabetes
mellitus (DM), but this is not true to the lowmolecular-weight adiponectin. Adiponectin
serum levels are inversely correlated with body mass index (BMI), visceral obesity, and
insulin resistance (IR); thus, it is regarded as an indicator and predictor of noninsulin
dependent DM, insulin resistant, and overt hyperglycemia.
3. NF induced pulmonary toxicity;
A. Is more common among women.

B. Is rare, but a serious toxic side effect may occur.

C. Can have the potential to negatively affect lungs.

NF induced pulmonary toxicity

NF induced pulmonary toxicity can be seen in three different forms, that is, due
to acute, subacute or chronic reaction. The acute form is the most common. Side
effects occurring up to the 1st month of receiving the first treatment are classified
as the acute form. Acute form develops secondary to hypersensitivity reaction
with peripheral demonstrating eosinophilia and thoracic CT showing ground glass opacity.
Chronic NF induced lung disease is seen predominantly in older women who present with
respiratory symptoms after a year or more of NF therapy. Characteristic pathologic finding
in chronic NF pulmonary toxicity is diffuse interstitial fibrosis, vascular sclerosis, fibrosis,
and thickening of the alveolar septa, interstitial inflammation, and bronchiolitis obliterans
with organising pneumonia.
4. Case Study gives information about;
A. A rare case of bilateral lower extremity edema in a young patient.

B. Talks of the effects of the low dose gabapentin therapy.

C. Worsening condition and management of the disease.

Case Study

A 46-year-old male with a past medical history of schizoaffective disorder and


chronic lower back pain, was admitted for management of worsening depression
and anxiety. He was started on gabapentin, 300mg twice daily for his back pain
and anxiety symptoms. His only other medication was hydrocodone. Over the next
few days, he started developing worsening bilateral lower extremity edema. He did
not have any cardiovascular-related symptoms. Physical exam was only significant
for 3+ pitting edema with all laboratory values and imaging being unremarkable.
Gabapentin was discontinued and his lower extremity swelling improved over
subsequent days. The incidence of pedal edema with gabapentin use is
approximately 7 to 7.5% with all studies being in elderly patients receiving doses
above 1200 mg/day. This case illustrates that lower doses of gabapentin can also
cause this adverse effect. It is important to recognize this adverse effect because
gabapentin is used in conditions like diabetic neuropathy, which is associated with
multiple comorbidities that can give rise to bilateral leg swelling. Presence of
gabapentin-induced leg swelling can thus confound the clinical picture.
5. What is correct about the survey?
A. Males were accounted for 53% of total participants

B. 43% participants had a bachelor degree of pharmacy

C. 71% were practicing as pharmacists


6. The data clearly explains that
A. The majority of the patients were between the age bracket of 29 days to 24 months

B. Over half of the patients were female

C. Male patients outnumbered female patients by 10%


READING SUB-TEST : PART C
In this part of the test, there are two texts about different aspects of healthcare.
For questions 7-22, choose the answer (A, B, C or D) which you think fits best according to the text.
Write your answers on the separate Answer Sheet

Text 1: Occupational Lung Diseases

Occupational lung diseases are a group of illnesses that are caused by either repeated,
extended exposure or a single, severe exposure to irritating or toxic substances that leads
to acute or chronic respiratory ailments. The rate of occupational lung conditions was
highest for education and health service workers in the private sector and local government
workers at 3 .8 and 5 .9 per 10,000 fulltime workers, respectively. There are two broad
categories of occupational lung diseases: (i) Diseases that are not occupation-specific, but
are aggravated at work, such as occupational asthma (ii) Diseases related to a specific
occupation, such as asbestosis, coal worker’s pneumoconiosis (black lung), berylliosis
(brown lung), and farmer’s lung. Common occupational lung diseases include
mesothelioma, occupational asthma, silicosis, asbestosis, and sick building syndrome.
Adult-onset asthma can be triggered by occupational exposures.

The estimated yearly cost of occupational injuries and illnesses is between $128
and $150 billion. Although, occupational lung diseases are often incurable, they are always
preventable. Improving ventilation, wearing protective equipment, changing work
procedures, and educating workers are key factors for prevention.
Occupational Asthma (OA) is the most common form of occupational lung disease.
Occupational asthma (also known as work-related asthma) is asthma that is caused by or
made worse by exposures in the workplace. Estimates suggest that 15 to 23 percent of new
asthma cases in adults are work related. Four states (California, New Jersey, Massachusetts,
and Michigan) tracked cases of occupational asthma over a seven-year period. During this
time, the occupations with the highest percentage of asthma cases were operators,
fabricators, and laborers (32.9%); managerial and professional specialty (20.2%), and
technical, sales, and administrative support jobs (19.2%). The four most common agents
associated with occupational asthma were miscellaneous chemicals (19.7%), cleaning
materials (11.6%), mineral and inorganic dust (11.1%), and indoor air pollutants (9.9%).
Malignant mesothelioma is a fatal type of cancer caused by exposure to asbestos.
Millions of construction and general industry workers have been exposed to
asbestos while on the job. Occupations associated with significantly higher
mesothelioma deaths include plumbers, pipefitters, and steamfitters; mechanical
engineers; electricians; and elementary school teachers. In the U.S., asbestos use
peaked in 1973 but had declined by 99.8 percent in 2007. Because mesothelioma
usually does not show up until 20 to 40 years after exposure, most of the deaths
from the disease are the result of exposures that occurred decades ago. This long
lag time means that mesothelioma deaths are expected to peak around 2010,
despite the much lower current use of asbestos. From 1999 to 2005, 18,068
malignant mesothelioma deaths were reported in the U.S. Men (81%) and
Caucasians (95%) accounted for the majority of these cases.

Silicosis is a disabling, dust-related disease and is one of the oldest occupational lung
diseases in the world. Silicosis is caused by exposure to and inhalation of airborne
crystalline silica. Dust particles from silica can penetrate the respiratory system and land
on alveoli (air sacs). This causes scar tissue to develop in the lungs and impair the
exchange of oxygen and carbon dioxide in the blood. Though symptoms of silicosis rarely
develop in less than five years, progression of the disease can lead to extreme shortness
of breath, loss of appetite, chest pains, and respiratory failure, which can cause death.
Silicosis also makes a person more susceptible to infectious diseases of the lungs, such as
tuberculosis. The death rate is generally low, but still too high considering that every one
of these deaths could have been prevented. Because of the low number of overall deaths
due to silicosis, multiple years of data are combined to provide a more accurate estimate
of the burden of this disease.
Text 1: Questions 7-14
7. The rate of OLC is reported to be higher in;
A. Healthcare service providers (private sectors)
B. Professionals in the field of education.
C. Government officials.
D. Local government workers and healthcare professionals in private industries.

8. One of these groups of diseases doesn’t come under OLD;

A. Mesothelioma and occupational asthma.


B. Occupational asthma and silicosis, asbestosis.
C. Asbestosis and mesothelioma.
D. Asbestosis and silicosis.

9. According to paragraph 2, OLDs are;

A. Incurable
B. Curable
C. Preventable but not curable
D. Curable and preventable

10. According to paragraph 2, work-related asthma means;

A. A disease which occurs due to more work


B. A disease which occurs due to less work
C. A disease which occurs due to exposure to work
D. None of the above

11. Common agents which are associated with OA in the lowest percentage are;

A. Air pollutants
B. Mineral and inorganic dust
C. Cleaning materials
D. Miscellaneous chemicals
12. The root cause of malignant mesothelioma is associated with the;

A. Use of the asbestos in the construction field


B. Exposure to asbestos on a regular basis
C. Low-quality asbestos
D. None of the above

13. The use of the asbestos was almost next to naught in the year;

A. 1997
B. 1973
C. 2007
D. 2010

14. Silicosis is;

A. More dangerous than occupational asthma.


B. More dangerous than mesothelioma.
C. Not as fatal as occupational asthma and mesothelioma.
D. Not very fatal.
Text 2: Immune System – Notes
Since inflammation in the body can lead to inflammation in the brain, we first need to
understand what inflammation is. Inflammation is part of the immune system’s response to
defend you against microbial infections. It is the body’s first line of defense against invasion
by microorganisms such as bacteria and viruses, and it is activated rapidly after infection.
The microbes are detected as foreign to the body by immune cells such as macrophages
(literally “big eater”). When macrophages encounter and recognize a foreign microorganism
they engulf the microorganism and, in addition, release a variety of cellular products into
the space around them that start and regulate further defenses that include inflammation.
Two classes of these products, known as cytokines and chemokines, leading to
inflammation. Cytokines are chemical messengers that travel away from the cells that
release them which causes alterations to the function of other cells. Chemokines also leave
the cell and attract other cells into the region. Together, they alter the blood vessels
near the site of infection, causing increased blood flow to the area and the entry of
immune system cells.

Inflammation—swelling, redness and heat—is part of the immune system’s first response to
microbial infections, but this defensive response is not limited to the bodily site of infection.
Soon after infection, a pattern develops that includes what is called the “acute phase
response (APR)” and “sickness behavior.” Fever is the most prominent feature of the APR
and for good reason: many microorganisms reproduce best at humans’ normal core body
temperature, and many of the immune system’s agents for killing them are bolstered by
elevated temperature. Sickness behaviors are well known to anyone who has had the flu.
They include reductions in activity, food intake, social interaction, mood sags; difficulty in
forming new memories; sleep changes; and sensitivity to pain increases (just think of how
even a light touch hurts when you have the flu). These changes also reduce the energetic
costs of behavior to free available energy stores to fight the infection. Fever, for example, is
quite energy intensive, requiring an extra 10 to 12 percent in energy for each degree rise. It
is obvious how all the sickness behaviors, with the exception of memory disruption, fit the
scheme of keeping us away from our usual activities. Memory disruption serves a different
purpose.
We now understand that all of the changes described above are accomplished
through the CNS. Fever, for example, occurs because the set point of temperature sensitive
cells in the hypothalamus is increased. Of course, behavior, mood, and pain are all products
of the CNS. This raises two issues: a) How does the CNS “know” what is going on in the
peripheral immune system, and b) What kinds of changes are produced in the CNS that
mediate fever and sickness behaviors? The same cytokines that participate in producing the
inflammatory response in the body also initiate the communication process to the CNS.
They accumulate in the bloodstream and thereby travel to the brain, where, although they
are large proteins and cannot readily cross the blood-brain barrier, these chemical signals
are carried across the barrier by active transport. They cross into the brain in regions where
the barrier is weak, and they bind to receptors on the insides of the cerebral vascular
blood vessels, thereby inducing the production of soluble mediators within the epithelial
cells that can cross into the brain.

The cytokine interleukin-1 beta is released in response to pathogen recognition, and a)


activates vagal fibers, b) diffuses into the brain where the barrier is weak,and c) are actively
transported across. In the brain, they act on microglia, which then produce and secrete
further cytokines that can act on neuronal cells, thereby producing sickness behaviors.

Often, a set of mechanisms that evolve to handle acute emergencies lead to outcomes that
nature did not intend if they are engaged too long. During a normal infection,
neuroinflammation and the resulting adaptive sickness behaviors persist only for several
days. However, if these responses become exaggerated or prolonged, the outcomes may
well become established, leading to cognitive impairment instead of brief memory
disruption, depression instead of reduced mood, fatigue instead of inactivity, and chronic
pain instead of acute pain. That is, physiology can become pathology when a set of
processes designed to be relatively brief becomes prolonged.
Text 2: Questions 15-22
15. The first line of defense implies;
A. Macrophages
B. Immune system
C. Inflammation
D. All of the above

16. Chemical messengers have;

A. The ability to change the functioning of other cells.


B. A greater level of mobility.
C. No capacity to move further away from cells.
D. None

17. APR develops;

A. At the time of infection.


B. After the infection.
C. After curing of the infection.
D. Before or after infection.

18. Fever occurs due to;

A. The increase in the growth of the microbes.


B. A powerless immune system.
C. The multiplication of the microbes, supported by the temperature of the human body.
D. none

19. One of the following is not associated with sickness behaviors;

A. Loss of appetite and reduction in social interaction.


B. Reduction in social interaction and sleeping hours.
C. Change in mood and body language.
D. Body language and sleeping disorder.
20. Cytokines are described as;

A. Messenger cells.
B. Protein bodies and messenger cells.
C. Immune response bodies.
D. None

21. Cytokines cross the barrier;

A. Through active transport.


B. By bridging the blood-brain barrier.
C. By penetrating into the blood vessels.
D. Through the bloodstream and lymph vessels.

22. What happens if the responses become exaggerated?

A. Loss of memory may take place.


B. Reduction in normal mood may be noted.
C. Cognitive impairment occurs.
D. Acute pain may begin.

END OF READING TEST, THIS BOOKLET WILL BE COLLECTED


Reading test 7 : Answer Key

Part A - Answer key 1 – 7


1: D
2: C
3: D
4: A
5: B
6: A
7: B

Part A - Answer key 8 – 14


8: Lusitropy
9: Low serum Gal-3 concentrations
10: Gal-3
11: Dyslipidemia
12: NP levels
13: Gal- 3
14: NP secretion

Part A - Answer key 15 – 20


15: Interleukin 1 Beta
16: Diabetes
17: Fibrosis
18: Predictive Marker
19: Natriuresis
20 : Oxidative Degradation
Reading test - part B – answer key

1: Hepatotoxicity can arise either from direct toxicity of the drug or its metabolites.
2: Plays a role in the development of insulin resistance.
3: Can have the potential to negatively affect lungs.
4: Talks of the effects of the low dose gabapentin therapy.
5: 71% were practicing as pharmacists
6: The majority of the patients were between the age bracket of 29 days to 24 months and

Reading test - part C – answer key

Text 1 - Answer key 7 – 14


7: Local government workers and healthcare professionals in private industries.
8: Asbestosis and mesothelioma.
9: Preventable but not curable
10: None of the above
11 Is: Air pollutants
12: Exposure to asbestos on a regular basis
13: 2007
14: Not very fatal.

Text 2 - Answer key 15 – 22


15: Inflammation
16: The ability to change the functioning of other cells.
17: After the infection.
18: The multiplication of the microbes, supported by the temperature of the human body.
19: Body language and sleeping disorder.
20: None
21: Through active transport.
22: Cognitive impairment occurs.
READING TEST 8
READING SUB-TEST : PART A
 Look at the four texts, A-D, in the separate Text Booklet.
 For each question, 1-20, look through the texts, A-D, to find the relevant information.
 Write your answers on the spaces provided in this Question Paper.
 Answer all the questions within the 15-minute time limit.
 Your answers should be correctly spelt.
TEXT BOOKLET - BATTENS DISEASE

Text A

Battens Disease

The Neuronal Ceroid Lipofuscinoses (NCL's), also known as Battens disease, are a
collection of congenital neurodegenerative conditions that span from prenatal life
to late adulthood with an incidence of 1:12,500. They comprise of at least 8
autosomal recessive disorders defined by having a mutation in a CLN gene, either
coding for an enzyme (CLN1 and CLN2) or a transmembrane protein (CLN3,
CLN5, CLN6 and CLN8) with all disorders having common clinical features,
including progressive visual loss to blindness, seizures, speech disturbances, motor
degeneration and intellectual decline, leading to early death LINCL has an
incidence of 0.36-0.46 per 100,000 with an age of onset of between 2-4 years, and
death commonly anticipated in the early teenage years. LINCL is caused by a
mutation of the CLN2 gene on chromosome 11p15, of which 98 mutations are
known, three of which account for the majority of cases.
Text B

The normal product of CLN2 is tripeptidyl peptidase I (TPP-1), which functions


within the lysosome to degrade N-terminal tripeptides from their substrates.
Therefore deficiencies lead to an accumulation, in particular of subunit C of
mitochondrial ATP synthase causing subsequent neuronal and retinal cell death.
This accumulation is seen on UV imaging as autofluorescent storage and on
electron microscopy as curvilinear bodies. Gene therapy is an attractive prospect
for long-term therapy in LINCL because it is a monogenic disorder. The process
involves introducing CLN2 human complementary DNA (cDNA) into the central
nervous system under the control of a promoter and in a suitable vector.
Adenoassociated viruses are considered the ideal vector as they can transduce
Postmitotic cells, mediate long-term gene expression and have an excellent safety record.
Therefore, these vectors are considered potential delivery vectors for the treatment
of LINCL. The challenge is whether using a suitable AAV vector allows sufficient
activity, expression and distribution of TPP-1 to destroy existing lysosomal storage
protein, prevent its on-going formation and consequently halt the progression of
the disease. One way of maximizing enzyme dispensation is through crosscorrection
using mannose-6-phosphate pathway, this means that CLN2 cDNA does
not need not be introduced to the whole of the CNS. The target for gene expression
is 1-5% normal expression of the gene product which is sufficient to reverse
pathology in other conditions.
Text C

One human trial evaluated the use of AAV2 vector to transfer human cCLN2
cDNA to the CNS of 10 children with LINCL aged between 3 and 10 with five
different mutation types. The study was an 18-month follow-up to vector
administration with a primary outcome measure being neurological assessment of
disease status using the modified Hamburg LINCL scale. Secondary measures
were quantitative CNS Magnetic Resonance Imaging assessment of the brain
including grey matter and ventricular volume. Control comparisons were made
with data from 4 independent untreated LINCL children who had been assessed
twice at 1-year intervals. Participants received an average dose of 2.5 × 1012
particles (1.8-3.2 × 1012) of the AAV2Hcln2 vector. The dose was shared equally
through 6 burr holes (3 in each hemisphere) of the cranial vault, and 12 cortical
locations were targeted. Assessments were made on days 7 and 14 and at 1, 6, 12
and 18 months after therapy. Adverse effects were assessed at 2 and 3 months.
Text D

While surgery itself recorded no evidence of adverse effects, post-therapy


highlighted 60 serious and 94 non-serious complications, the majority occurring
within the first 2 weeks after therapy. Serious events, including seizures and
myoclonus were considered to be consequences of drug administration, whereas
vomiting and thrombocytosis (non-serious examples) were not considered to be
caused by the operation or by tolerance to the therapy. Out of the 10 patients, 2
died, one of unknown reasons and the other during the study period from status
epilepticus, a known complication of late LINCL. Primary assessments
demonstrated that gene therapy had an effect on the progression of disease, with
an improved modified Hamburg scale score in treated subjects monitored for >6
months, and a disease progression rate which was significantly slower than in
controls. This difference was shown by the mean rate of change of the modified
Hamburg scale in the treated and untreated groups (P<0.05). Secondary measures
showed that the MRI parameters indicated a decreased rate of decline, comparable
with the primary outcomes, however, there was no statistical significance between
groups including in the grey matter volume as a percentage of total brain volume
with -2.6%/year loss of volume compared to -2.84%/year in controls (p=0.8).
PART A -QUESTIONS AND ANSWER SHEET

Questions 1-7

For each question, 1-7, decide which text (A, B, C or D) the information comes from.
You may use any letter more than once.

In which text can you find information about;

1. It is not currently known to cause disease, but causes a very mild immune response. Answer___
2. A disorder in which nerve cell activity in the brain is disturbed, causing seizures. Answer ___
3. Not so prevalent disorder that primarily affects the nervous system. Answer ___
4. Degeneration of the neurons. Answer _____________
5. Recent studies conducted on evaluation of the disease. Answer ______________
6 .After effects of the treatment. Answer _____________
7. Affected person may not be in a state to use mental power effectively. Answer ____________

Questions 8-14

Answer each of the questions, 8-14, with a word or short phrase from one of the
texts. Each answer may include words, numbers or both. Your answers should be correctly spelt.

8 What is the common reason for Battens Disease? Answer ______________


9 Which can transduce postmitotic cells? Answer_______________
10 Which is a known complication of late LINCL? Answer_______________
11 What are the two conditions which are known to arise not out of therapy ? Answer____
12 What is used to maximize the enzyme dispensation through cross-correction? Answer____
13 Which is the effective long-term therapy in LINCL? Answer____
14 What dosage of the AAV2Hcln2 vector did the participants receive? Answer___
Questions 15-20

Complete each of the sentences, 15-20, with a word or short phrase from one of the texts.
Each answer may include words, numbers or both. Your answers should be correctly spelt.

15.____________ can be described as spasmodic jerky contraction of groups of muscles.


16 . The ____________ gene provides instructions for making an enzyme called tripeptidyl peptidase 1.
17. It is always not required to introduce ____________ to CNS.
18.____________ made it all clear that the therapy can have more positive impact on the disease.
19. Out of all the mutations, _____________ are considered to be the major causes for the disease to
appear in infants and adults.
20 .The____ gene provides instructions for making a protein whose function is not well understood

END OF PART A, THIS QUESTIONS PAPER WILL BE COLLECTED


READING SUB-TEST : PART B
In this part of the test, there are six short extracts relating to the work of health professionals .
For questions 1-6, choose the answer (A, B or C) which you think fits best according to the text.
Write your answers on the separate Answer Sheet
Questions 1-6

1 .What does the notice indicates?


A. Angina pectoris is common among people.
B. There is a direct relationship between the functioning of liver and Qi blood of the
heart.
C. Various ailments can lead to angina pectoris.

Angina pectoris

Angina pectoris, whose common type is Qi-blood stagnation type is one of chest
congestion in TCM. The main cause of this type of angina is damage due to an
excess of seven emotions. The heart governs the mind and blood circulation, liver
stores of blood and controls catharsis. There is close relationship between the two.
Only if liver function is normal can Qi-blood of the heart can be calm. For elderly
individuals, who lack Qi-blood, yin and yang of the heart, spleen and kidney, are
of cold invasion, consume an inappropriate diet, have emotional disorders and
excessive fatigue, among others, can also lead to blood stasis stagnation, which can
consequently cause coronary angina pectoris of Qi-blood stagnation. The
‘Suwen Yujizhen zang’ theory has described “blockage of vessel, inability of QI to
flow freely”, which illustrates that blood stagnation can lead to Qi depression.
Therefore, accelerating blood circulation, removing blood stasis and promoting the
circulation of Qi are the main therapies.
2. The notice gives information about;
A. two different technologies used in treatment.

B. ineffectiveness of the SCS.

C. DRG and SCS comparison.

Spinal cord stimulation

For many years, Spinal cord stimulation has been used as a salvage treatment for
intractable CRPS even though many studies have not proven long-term benefit. In
multiple studies published by a European neurosurgical group, there has been great
benefit from this technology in the first year but the vast majority of patients experience a
return of symptoms by year six. Newer dorsal root ganglion (DRG) stimulation technology
may be more promising. In the most recent DRG stimulation trial, patients with CRPS have
been shown to decrease pain by fifty percent or greater in 93% of patients with chronic
intractable pain at three-month follow-up, versus 72% of patients with an SCS implant.
Unfortunately, this product is currently only FDA approved for treatment in the lower
extremity.
3 .SGB is known;
A. to be more effective in treatment.

B. to provide temporary relief from pain.

C. to be a proven medicine in certain instances.

Stellate ganglion blocks (SGB)

Stellate ganglion blocks (SGB) are the most commonly performed interventional
procedure for patients with upper extremity CRPS. The stellate ganglion is located
anterior to the 7th cervical transverse process on the anterior surface of the longus
coli muscle. It lies medial to the vertebral artery and anterolateral to the ipsilateral
common carotid artery. This ganglion can be accessed either through fluoroscopy,
CT, or ultrasound guidance. In a study published in 2006 by Ackerman and Zhang,
25 subjects underwent SGB at weekly intervals for 3 weeks. At 6 months, 40% of
patients had complete symptom relief while 24% of patients had no pain relief. In a
second study, published in 2009, three weekly blockades were completed at weekly
intervals in 22 patients with CRPS type I of the hand. Pain intensity and range of motion
were assessed two weeks after treatment. In this study patients had statistically significant
improvement in wrist ROM (P>0.001) and an overall decrease in VAS values from 8 to 1.
While most physicians do not believe that SGB alone is effective in curing the disease, we
do know that stellate ganglion blocks at least offer temporary benefits that last well
beyond the effects of local anesthetic.
4 .What is correct about metabonomics?
A. High-throughput metabolomic approach revealed the acupuncture exerting
intervention effects.

B. Metabolomics has seen a surge in popularity in recent scientific research.

C. The ultimate aim of metabonomics is to detect every small-molecule metabolite.

Metabonomics

Metabonomics as an advanced technology of systems biology has made considerable


contributions to early diagnosis of serious illness, toxicology and pharmacology. Early,
accurate diagnosis of tumours, such as liver cancer and ovarian cancer, is a distinguishing
feature and advantage of metabonomics. Valine, saturated lipids, glycine, lactate, inositol,
nucleotides, polyunsaturated fatty acids, taurine and other tumour-related metabolic
markers can be identified by metabolomics, which makes metabonomics increasingly
applicable to diagnosis, therapy and prognostic evaluation. Acupuncture has a therapeutic
effect for tumours: it can improve symptoms, prolong the lifespan of patients, improve the
quality of life, relieve cancer pain syndrome and, especially, improve the side effects of
radiotherapy and chemotherapy, eg, myelosuppression, gastrointestinal reaction, fever
etc. Acupuncture treatment of tumour possesses many features and advantages. Applying
metabonomics to research investigating acupuncture treatment of tumours both gives
play to the sensitive detection advantage of metabonomics for tumour diagnosis,
treatment assessment and exploring the mechanism of acupuncture treatment at deeper,
more extensive and more dynamic levels. In addition, there are some reports investigating
Alzheimer diseases at home and abroad, however, few in research of venereal disease.
5 .What does this notice indicate?
A. The curative effect of improving disease condition in treatment group is superior to
the control group.

B. two groups differ only by minor values.

C. total effectiveness of the control group is lesser than treatment group.

Treatment Groups - Recent Analysis


C Total
excellen effect Ineffec exacerb
Groups as effective
t ive tive ated
es rate, %
4 70.0
Control 19 9 10 2
0
Treatme 4 90.2*
28 9 4 0
nt 1
6 .What does the result indicate?
A. The results indicate that subjects in the two groups improved after treatment.
B. The laboratory indexes of the treatment group significantly declined.
C. After treatment, the laboratory indexes of the observation group were also
more declined than the control group.

Disease Treatment Results - Before and After


READING SUB-TEST : PART C
In this part of the test, there are two texts about different aspects of healthcare.
For questions 7-22, choose the answer (A, B, C or D) which you think fits best according to the text.
Write your answers on the separate Answer Sheet

Text 1: Hormonal Disorder - Adrenal Insufficiency

Adrenal insufficiency is an endocrine or hormonal disorder that occurs when the


adrenal glands do not produce enough of certain hormones. The adrenal glands are
located just above the kidneys. Adrenal insufficiency can be primary or secondary.
Primary adrenal insufficiency, also called Addison’s disease, occurs when the
adrenal glands are damaged and cannot produce enough of the hormone cortisol,
and often the hormone aldosterone. Addison’s disease affects one to four of every
100,000 people, in all age groups and both sexes. Secondary adrenal insufficiency
occurs when the pituitary gland, a bean-sized organ in the brain, fails to produce
enough adrenocorticotropin (ACTH), a hormone that stimulates the adrenal glands
to produce cortisol. If ACTH output is too low, cortisol production drops.
Eventually, the adrenal glands can shrink due to lack of ACTH stimulation.
Secondary adrenal insufficiency is much more common than Addison’s disease.

Addison's disease symptoms usually develop slowly, often over several months,
and may include muscle weakness and fatigue, weight loss and decreased appetite,
darkening of skin (hyperpigmentation), low blood pressure (even fainting), salt
cravings, low blood sugar (hypoglycemia), nausea, diarrhea or vomiting, muscle or
joint pains etc. Sometimes, however, the signs and symptoms of Addison's disease
may appear suddenly. In acute adrenal failure (Addisonian crisis), the signs and symptoms
may also include pain in your lower back, abdomen or legs, severe vomiting and diarrhea,
leading to dehydration, low blood pressure and loss of consciousness.
Your adrenal glands are composed of two sections: the interior (medulla) produces
adrenaline-like hormones; the outer layer (cortex) produces a group of hormones
called corticosteroids, which include glucocorticoids, mineralocorticoids and male
sex hormones (androgens). Glucocorticoids: These hormones, which include cortisol,
influence your body's ability to convert food fuels into energy, play a role in your immune
system's inflammatory response and help your body respond to stress. Mineralocorticoids:
These hormones, which include aldosterone, maintain your body's balance of sodium and
potassium to keep your blood pressure normal. Androgens, male sex hormones, are
produced in small amounts by the adrenal glands in both men and women. They cause
sexual development in men and influence muscle mass, libido and a sense of well-being in
men and women.

Addison's disease occurs when the cortex is damaged and doesn't produce its hormones in
adequate quantities. Doctors refer to the condition involving damageto the adrenal glands
as primary adrenal insufficiency. The failure of your adrenal glands to produce
adrenocortical hormones is most commonly the result of the body attacking itself
(autoimmune disease). For unknown reasons, your immune system views the adrenal
cortex as foreign, something to attack and destroy. Other causes of adrenal gland failure
may include tuberculosis, other infections of the adrenal glands, spreading of cancer to the
adrenal glands and bleeding into the adrenal glands. Adrenal insufficiency can also occur if
your pituitary gland is diseased. The pituitary gland produces a hormone called
adrenocorticotropic hormone (ACTH), which stimulates the adrenal cortex to produce its
hormones. Inadequate production of ACTH can lead to insufficient production of
hormones normally produced by your adrenal glands, even though your adrenal glands
aren't damaged. Doctors call this condition secondary adrenal insufficiency. Another more
common cause of secondary adrenal insufficiency occurs when people who take
corticosteroids for treatment of chronic conditions, such as asthma or arthritis, abruptly
stop taking the corticosteroids. If you have untreated Addison's disease, an Addisonian
crisis may be provoked by physical stress (such as an injury), infection or illness. All
treatment for Addison's disease involves hormone replacement therapy to correct the
levels of steroid hormones your body isn't producing. Some options for treatment include
oral corticosteroids, corticosteroid injections and androgen replacement therapy.
Text 1: Questions 7-14

7. Addisons disease effects;


A. Four of every 1,000 people.
B. One of every 1,000 people, including all age-groups.
C. One to four of every 1,000 people, including all age groups.
D. One to four of every 100,000 people.

8. Secondary adrenal insufficiency occurs due to one of these reasons;


A. More production of adrenocorticotropin (ACTH).
B. More production of cortisol.
C. Low production of aldosterone.
D. Low production of ACTH.

9. According to the passage, symptoms of Addisons disease occur;


A. Slowly
B. Suddenly
C. After a month
D. Slowly after many months

10. Cortex produces;


A. Androgens
B. Glucocorticoids
C. Mineralocorticoids
D. All of the above
11. Aldosterone maintains;
A. Body balance
B. Balance of sodium and potassium
C. High BP
D. Low BP

12. According to the information given in the passage, the statement that Androgen
influences muscle mass and physical and mental nature of men and women is;
A. True
B. False
C. Can say
D. Not given in the passage(s)

13. Doctors refer to the damage to the cortex of the kidney as;
A. Primary adrenal insufficiency.
B. Secondary adrenal insufficiency.
C. Other fatal infections.
D. Not given

14 .Secondary adrenal insufficiency also occurs because of;


A. Asthma
B. Arthritis
C. Both asthma and arthritis.
D. Stopping the use of corticosteroids.
Text 2: Carpal Tunnel Syndrome

Carpal tunnel syndrome is a condition that may be caused by repeatedly performing


stressful motions with your hand or holding your hand in the same position for long
periods of time. CTS is classified as a cumulative trauma disorder, an ailment that
attacks the body’s musculoskeletal system. The musculoskeletal system is made up of
muscles that pull on tendons and move the bones at joints. The joints are held
together by ligaments. Carpal tunnel syndrome specifically affects the sensitive nerves
of - and the blood supply that feeds – the hands and wrists.

Carpal tunnel syndrome has been around for a long time; meatpackers began
complaining of pain and loss of hand function in the 1860s. Back then, these
complaints were largely attributed to poor circulation. The nature of work has
changed over the years; today, more jobs are highly specialized and require use of
only a small number of muscles repeatedly. With the growing numbers of people
using computers and keyboards, plus the focus on better healthcare for workers,
carpal tunnel syndrome is of real concern to both employers and health-care
professionals. Recent studies have shown that carpal tunnel syndrome, like all
other cumulative trauma disorders, is on the rise while other workplace injuries
have leveled off. Many companies are turning to physical therapists for help with
designing and implementing health promotion and injury prevention programs to
protect their employees from CTS.
People with CTS usually experience feelings of numbness, weakness, tingling, and
burning in their fingers and hands. If not treated, the symptoms may escalate into
acute, persistent pain. CTS can become so crippling that people can no longer do
their work or even perform simple tasks at home. At its most extreme, carpal
tunnel syndrome forces people to undergo surgery and miss many days of work, or
prevents them from working at all because their hand functions are permanently
impaired. Carpal tunnel syndrome occurs in men and women of all ages, and is
often found in workers whose tasks require repeating the same motion in the
fingers and hand for long periods of time. CTS has surfaced among meat packers,
assembly line workers, jackhammer operators, and employees who spend hours
working at a computer or typewriter. Carpal tunnel syndrome shows up in athletes
as well as homemakers.

The U.S. Department of Labor has cited carpal tunnel syndrome, as well as other
cumulative trauma disorders, as the cause of 48 percent of all industrial workplace
illnesses. The disease affects more than five million Americans. CTS’s impact on
American businesses is devastating. It shows up in the workplace in the form of
fatigue, poor work performance, discomfort and pain, and poor
employer/employee relations. The high cost of treatment for an employee with
CTS, plus the lost productivity when that employee is absent for a long period of
time, strains the company’s ability to operate efficiently and can lead to morale
problems when other employees have to take over the absent workers’
responsibilities. Physical therapists with specialized training in cumulative trauma
disorders have been working in industrial and corporate settings for many years to
meet the health-care needs of America’s workforce. They work closely with
employers to educate employees about CTS—what causes it and how to avoid it
through proper use of the musculoskeletal system.
Physical therapists can target and correct poor work habits and improper work
designs, such as tools, furniture, equipment, and workspace. They can also assess
the potential risks of an individual and determine if that person is physically
unsuited to a particular job. Among their many responsibilities, physical therapists
teach health awareness and job safety. A typical education program includes
exercises employees can do at work and at home, adjustments to the overall work
environment and individual workstations, plus early detection of symptoms to
avoid painful and costly surgery. Physical therapists also work with employers and
their engineering departments to design and modify the work environment, helping
to remove the causal factors of CTS. If anyone has symptoms of carpal tunnel
syndrome, then consulting a physical therapist or other qualified healthcare
practitioners for an evaluation and individualized treatment is always
recommended.
Text 2: Questions 15-22

15. According to the passage, CTS;


A. Is a cumulative trauma disorder.
B. Is caused due to weakness in musculoskeletal system.
C. Occurs due to weakness in ligaments between joints.
D. All of the above

16. According to the information given, CTS is;


A. On the rise.
B. On the rise without any other cumulative trauma disorders.
C. One of the common cumulative trauma disorders that are increasing.
D. On the rise with many other cumulative trauma disorders.

17. In CTS, _____ may become permanently impaired.


A. Carpels
B. Hands
C. Feet
D. Wrists

18. CTS often occurs in;


A. Men
B. Women
C. Men and women of all ages
D. Only men of all ages

19.____ complained of pain and loss of hand function in the 1860s;


A. Meat packers
B. Assembly line workers
C. Jackhammer operators
D. Employees who spend hours at a computer

20. CTS accounts for _____.of all industrial workplace illnesses;


A. 50%
B. 48%
C. 84%
D. 60%

21. Physical therapists teach employees about;


A. Proper use of the musculoskeletal system.
B. How to avoid CTS.
C. How to increase efficiency at work.
D. Health-care needs.

22. CTS educational program includes;


A. Only detection of CTS symptoms in employees.
B. Explanation to employees on how to avoid CTS.
C. Exercises that employees can do to avoid CTS.
D. Discussion of cost involved in surgery.

END OF READING TEST, THIS BOOKLET WILL BE COLLECTED


Reading test 8: Answer Key
Part A - Answer key 1 – 7
1 :B
2 :D
3A
4:B
5:C
6:D
7 :A

Part A - Answer key 8 – 14


8: mutation of the CLN2 gene
9: Adeno-associated viruses
10: epilepticus
11 : vomiting and thrombocytosis
12: mannose-6-phosphate pathway
13: Gene therapy
14: 2.5 × 1012 particles

Part A - Answer key 15 – 20


Complete each of the sentences, 15-20, with a word or short phrase from one of
the texts. Each answer may include words, numbers or both.
15: Myoclonus
16 : Tpp1
17: Cln2 Cdna
18 : Primary Assessments
19: Three
20 :Cln6
Reading test - part B – answer key
1: Various ailments can lead to angina pectoris.
2: ineffectiveness of the SCS.
3: to provide temporary relief from pain.
4: Metabolomics has seen a surge in popularity in recent scientific research.
5: The curative effect of improving disease condition in treatment group is
superior to the control group.
6: The results indicate that subjects in the two groups improved after treatment.

Reading test - part C – answer key

Text 1 - Answer key 7 – 14


7: One to four of every 1,000 people, including all age groups.
8: Low production of ACTH.
9: Slowly after many months
10: Androgens
11: Balance of sodium and potassium
12: False
13: Primary adrenal insufficiency.
14: Stopping the use of corticosteroids.

Text 2 - Answer key 15 – 22


15: Is a cumulative trauma disorder.
16: On the rise with many other cumulative trauma disorders.
17: Wrists
18: Men and women of all ages
19: Meat packers
20: 48%
21: How to avoid CTS.
22: Exercises that employees can do to avoid CTS.
READING TEST 9
READING SUB-TEST : PART A
 Look at the four texts, A-D, in the separate Text Booklet.
 For each question, 1-20, look through the texts, A-D, to find the relevant information.
 Write your answers on the spaces provided in this Question Paper.
 Answer all the questions within the 15-minute time limit.
 Your answers should be correctly spelt.
TEXT BOOKLET- AUTISM SPECTRUM DISORDER

Text A

Autism Spectrum Disorder


Autism Spectrum Disorder (ASD) develops in early childhood. Recent population
analysis indicates that the number of cases of ASD is increasing in many countries,
particularly in technologically developed countries. The U.S. Center for Disease
Control research claims that, in some states, one of every 68 children (one of 42
boys) has a diagnosis of the ASD, a 30% increase from 2012 (IACC Strategic Plan
for Autism Spectrum Disorder Research, 2013.
Multiple dysfunctional reflex patterns are characteristic in two separate groups of
children diagnosed with autism: 1) those whose patterns were immature or
pathological and severely dysfunctional from birth, and 2) those that developed
normally but regressed into autism at age 2 or 3 unexpectedly. Reflexes of these
children may have been delayed and immature, but not noted by specialists or
parents. Their nerve system, possibly, was not resilient enough to cope with the
stress that they experienced. Alternatively, their reflexes might not have matured
and have caused the asynchronicity in their brain function development on both
cortical and extrapyramidal levels resulting in neurodevelopmental disorders
beginning around 2 years of age. An initially mild unrecognized problem can lead
to more complicated deficits with age.
Text B

Individuals diagnosed with ASD show a chronic lack of sensory motor integration
and delay of skills concerning the early motor milestones. They show a wide range
of immature reflex patterns such as Hands Pulling, Hands Supporting, Hands Grasp,
Crawling, Asymmetrical Tonic Neck Reflex, Symmetrical Tonic Neck Reflex, Babkin
Palmomental, Ocular-Vestibular, and other patterns. The MNRI program utilizes
non-invasive intervention to support the development of the neuro-sensory-motor
aspects of those reflex patterns through specific techniques and procedures that
allow restoration of links between reflex circuit components and the protection
function of a reflex to normalize their over-freezing and fight or flight reactions
seen, for example, in tactile defensiveness or deprivation. Thus, the MNRI program
works particularly with the autonomic nervous system – its sympathetic and
parasympathetic processes.
Text C

Disharmony in muscular system development and lack of regulation for muscle


tone beginning in children with ASD in their infancy results in impulsive reactions
that often turns into permanent physical characteristics and behaviors as they
grow older. For example, impulsivity may lead to poor ability for goal setting, poor
focus and following instructions, deficient inner control, hyperactivity,
disorganized and chaotic behavior, and irritability and impatience. Lack of muscle
tone regulation may later result in challenges in motor programming and control,
planning, and thus lead to poor motor-cognitive- behavior coordination. This poor
regulation is caused by a lack of balance in the excitation and inhibition processes
in the reflex circuits, including improper connectivity between alpha and gamma
motor neurons. Clinical observations show that the disharmony and lack of proper
regulation in muscle tone in children with ASD are seen mainly in: Hypertonic
muscles in the posterior dorsal plane of the body (along the spinal column -
sacrospinalis, thoracic longus, trapezius) and with the opposite hypotonic
abdominal muscles and diaphragm negatively affects development of postural
control. The child with ASD, in an attempt to release tension caused by this
disproportion of muscle tone in the back and front of their body, may often display
reactivity in behavior and impulsive movements triggering balance/equilibrium
mechanisms (balancing reflex pattern), resulting in a state of being overstimulated.
Text D

Problems in visual and auditory perception systems:


The eyes of children diagnosed with ASD show a restless state or lack of mobility
and dilated pupils. They usually have a limited, narrow visual span, poor visual
attention and focus, and hyperactive peripheral vision. Their eye movements
appear to freeze or jump rapidly in saccades. Many children with ASD demonstrate
an addictive tendency for computers and cell phones with compulsive repetition of
the same image, object, or program, often watching it at a very close range. The
child with ASD becomes over-focused, which over-stimulates their vestibular
system and static balance. The Pupillary Reflex in these children may become
hypersensitive, overstimulating the sympathetic system, with either over-reactive
or hypoactive motor activity. The visual system of children diagnosed with ASD
copes poorly with this visual chaos which leads to a visual processing disorder. A
Bonding response in infants is seen from their first months after birth. Bonding as a
behavior trait matures during their first years of life. Almost every child with ASD
assessed presented signs of inadequate bonding - lack of attachment, tactile and
interactivity defensiveness, a tendency for self-isolation, a poor imitation, and poor
learning of verbal communication. When bonding is immature, there are problems
with visual contact, focusing on the face/eyes of their mother and other adults and
poor emotional communication, inability to adequately smile, and poor labeling of
the objects in their environment.
PART A -QUESTIONS AND ANSWER SHEET

Questions 1-7
For each question, 1-7, decide which text (A, B, C or D) the information comes from.
You may use any letter more than once.
In which text can you find information about;
1. Represents the resistance to passive movement of a joint. Answer _______________
2. Associated with pupillary function. Answer ________________
3. Utilization of information and clinical experience from neurodevelopment in different ways.
Answer_____________
4. Development of autism. Answer______________
5. Possibility of development of strange characteristics as one grows. Answer ______________
6. Not existing or occurring at the same time with respect to movements or reactions. Answer____
7. Primitive reflex that normally emerges during the first year of an infant's life. Answer __

Questions 8-14
Answer each of the questions, 8-14, with a word or short phrase from one of the texts.
Each answer may include words, numbers or both. Your answers should be correctly spelt.
8. What MNRI will operate with? Answer _____________
9. How eye movements may appear? Answer ___________
10. What is the term which defines activation of the vestibular system which causes eye movement?
Answer ____________
11 ..What is related to tendency to act on a whim, displaying behaviour characterized by little or no
forethought, reflection? Answer ____________
12. What is the term used to define healthy stress? Answer__________
13. What is known to be activated as a result of turning the head to one side? Answer______
14. What is the impact visual chaos of the children with ASD? Answer ___________
Questions 15-20

Complete each of the sentences, 15-20, with a word or short phrase from one of the texts.
Each answer may include words, numbers or both. Your answers should be correctly spelt.

15. The term _______ is used to describe a rapid movement of the eye between fixation points.

16. Generally,____________ in newly born babies will be at very early stages.

17. What synchronizes hands, neck, and jaw is _____________

18. In many of the cases, reflex in affected children may turn out to be more __________ .

19. Almost all of the children with ASD show various signs of ____________ .

20 .The complexities in _______can be the result of the regulation changes in muscle tone.

END OF PART A, THIS QUESTIONS PAPER WILL BE COLLECTED


READING SUB-TEST : PART B
In this part of the test, there are six short extracts relating to the work of health professionals .
For questions 1-6, choose the answer (A, B or C) which you think fits best according to the text.
Write your answers on the separate Answer Sheet

Questions 1-6

1. What this notice talks about?


A. AIDS-related deaths dropped by more than 50%

B. AIDS-related deaths increased more significantly

C. Efforts that can lead to curbing AIDS in various countries.

UNAIDS Programme Coordinating Board

The 31st UNAIDS Programme Coordinating Board (PCB) meeting took place in Geneva
from 11-13 June 2018. There were more than 700 000 less new HIV infections estimated
globally in 2016 than in 2001. The road from 2.5 million new HIV infections in 2011 to zero
new HIV infections is a long one and significant efforts are required to accelerate HIV
prevention programmes. Sustained investments for access to antiretroviral therapy by
donors and national governments have led to record numbers of lives being saved in the
past six years.
In 2011 more than half a million fewer people died from AIDS-related illnesses than six
years earlier. It’s a dramatic turning point. Numbers can quantify, but alone cannot
express the impact of each averted death on the whole community, including its children.
The number of people dying from AIDS-related causes began to decline in the mid-2000s
because of scaled up antiretroviral therapy and the steady decline in HIV incidence since
the peak of the epidemic in 1997. In 2011, this decline continued, with evidence showing
that the drop in the number of people dying from AIDS-related causes is accelerating in
several countries.
2. The given notice explains the procedure of;
A. Use of radix Sophorae samples.
B. Ultrasonic treatment - obtaining radix.
C. Preparing radix Sophorae tonkinensis samples.

Radix Sophorae tonkinensis


Radix Sophorae tonkinensis was crushed and screened, then taking screened powder
(approximately 0.5 g) gain a respective weight, denoted M. Trichloromethane-methanol-
ammonia (40:10:1) was used to dispose of the radix Sophorae tonkinensis samples for 30
min, and all samples were subsequently obtained from organic solvent extraction with 30
min ultrasonic treatment. All disposed of samples were filtered, then 10 mL of filtrate was
measured to recover solvents to dry under decompression at 38°C to obtain the residue, the
residue was diluted by methanol, then transferred to a 10 mL volumetric flask. After mixing
and filtering with 0.45 µm filter membrane, Radix Sophorae samples were obtained. The
blank groups were treated as the samples but without radix Sophorae tonkinensis.

3. What is correct about Torcetrapib?


A. It is known to enhance health.
B. Trials performed produced negative results.
C. The drug did not get approved completely as the project was dropped in the middle.
Short note on Torcetrapib
Torcetrapib, which has been in development since the early 1990s, was supposed to raise
so-called good cholesterol, and cardiologists had hoped it would reduce the buildup of
plaques in blood vessels that can cause heart attacks. This drug actually caused an increase
in deaths and heart problems. Eighty-two people had died so far in a clinical trial, versus
51 people in the same trial who had not taken it. The GABR company gave up the project
incurring a claimed loss of $1 billion investment and not much hue and cry was raised on
the deaths of the study subjects as this was the doing of a billionaire giant manufacturer.
In case of trials or experiments, if a single death is noticed with the use of UD, a big hue
and cry would be raised.
4. The table shows
A. Comparison of Type A, B, C and G and H drugs and their resistance quality.
B. Type A is known to produce more adverse reactions than all the others present in the
table.
C. A total of 200 cases have been reported with respect to adverse reaction.

Drug Adverse Reactions


Type of ADR Number ADRS Percentage (%)
Type-A 96 103.7
Type-B 69 56.79
Type-C 23 28.39
Type G 8 7.4
Type H 4 3.7
Total 200 200

5. The notice clearly explains


A. Clinical trials using Vitamin B12.
B. Advantages of B12.
C. Study performed with focus on B12 usage.

Vitamin B12
Vitamin B12 is produced by the liver and is involved in several biochemical metabolic
reactions. It promotes the repair of damaged skin mucous membranes and vascular
endothelial cells, reduces spasm and occlusion of blood vessels, improves local blood flow
and prevents the deterioration of wound infection. In addition, it reduces the excitability of
pain fibers C and AG, leading to an analgesic effect. Vitamin B12 injections to the skin in the
radiation field benefit the wound by reducing irritation and pain, preventing rupture and
enhancing new epithelial resistance to radiation, thereby promoting healing of the skin.
Chen et al used a vitamin B12 solution to treat radiation-induced moist dermatitis. The cure
rate at 10 days was 100%, which was significantly different from the control group.
6. What is correct about the given table?
A. The age wise male patients population ranges from 4.

B. 18.18 patients were in the age group of 50-70 years.

C. 32.72 patients were in the age group of 50-60 years.

Age wise distribution of male patients showing percentage of distribution.


Age in years Male patients Age in years Female patients
20-30 4 (7.27) 20-30 0 (0)
30-40 54 (98.18) 30-40 32 (71.11)
40-50 24 (43.63) 40-50 38
50-60 18 50-60 10
60-70 10 60-70 6 (13.32)
Total 110 (110) 70-80 4 (8.88)
Total 90 (9
READING SUB-TEST : PART C
In this part of the test, there are two texts about different aspects of healthcare.
For questions 7-22, choose the answer (A, B, C or D) which you think fits best according to the text.
Write your answers on the separate Answer Sheet

Text 1: Ebola Virus and Marburg Virus

The Ebola virus and Marburg virus are related viruses that cause hemorrhagic fevers; illnesses
marked by severe bleeding (hemorrhage), organ failure and, in many cases, death. Both the
Ebola virus and Marburg virus are native to Africa, where sporadic outbreaks have occurred
for decades. The Ebola virus and Marburg virus both live in animal hosts, and humans can
contract the viruses from infected animals. After the initial transmission, the viruses can
spread from person to person through contact with bodily fluids or contaminated needles.

No drug has been approved to treat the Ebola virus or Marburg virus. People diagnosed with
the Ebola or Marburg virus receive supportive care and treatment for complications. Scientists
are coming closer to developing vaccines for these deadly diseases. In both the Ebola virus and
Marburg virus, signs and symptoms typically begin abruptly within the first five to 10 days of
infection. Early signs and symptoms include fever, severe headaches, joint and muscle aches,
chills, sore throat and weakness. Over time, symptoms become increasingly severe and may
include nausea and vomiting, diarrhea (may be bloody), red eyes, raised rash, chest pain and
coughing, stomach pain, severe weight loss, bleeding from the nose, mouth, rectum, eyes and
ears.

The Ebola virus has been found in African monkeys, chimps and other nonhuman primates. A
milder strain of Ebola has been discovered in monkeys and pigs in the Philippines. The
Marburg virus has been found in monkeys, chimps and fruit bats in Africa. The virus can be
transmitted to humans by exposure to an infected animal's bodily fluids, including blood.
Butchering or eating infected animals can spread the viruses; scientists who have operated on
infected animals as part of their research have also contracted the virus. Infected people
typically don't become contagious until they develop symptoms. Family members are often
infected as they care for sick relatives or prepare the dead for burial.
Medical personnel can be infected if they don't use protective gear such as surgical masks and
latex gloves. Medical centers in Africa are often so poor that they must reuse needles and
syringes and some of the worst Ebola epidemics have occurred because contaminated
injection equipment wasn't sterilized between uses. There's no evidence that the Ebola virus
or Marburg virus can be spread via insect bites.
Ebola and Marburg hemorrhagic fevers are difficult to diagnose because many of the early
signs and symptoms resemble those of other infectious diseases, such as typhoid and malaria.
But if doctors suspect that you have been exposed to the Ebola virus or Marburg virus, they
use laboratory tests that can identify the viruses within a few days.

Most people with Ebola or Marburg hemorrhagic fever have high concentrations of the virus
in their blood. Blood tests known as enzyme-linked immunosorbent assay (ELISA) and reverse
transcriptase polymerase chain reaction (PCR) can detect specific genes or the virus or
antibodies to them. No antiviral medications have proved effective in treating the Ebola virus
or Marburg virus infections. As a result, treatment consists of supportive hospital care; this
includes providing fluids, maintaining adequate blood pressure, replacing blood loss and
treating any other infections that develop.

As with other infectious diseases, one of the most important preventive measures for the Ebola
virus and Marburg virus is frequent hand-washing. Use soap and water, or use alcohol-based
hand rubs containing at least 60 percent alcohol when soap and water aren't available. In
developing countries, wild animals, including nonhuman primates, are sold in local markets;
avoid buying or eating any of these animals. In particular, caregivers should avoid contact with
the person's bodily fluids and tissues, including blood, semen, vaginal secretions and saliva.
People with Ebola or Marburg are most contagious during the later stages of the disease. If
you're a healthcare worker, wear protective clothing — such as gloves, masks, gowns and eye
shields - keep infected people isolated from others. Carefully disinfect and dispose of needles
and other instruments; injection needles and syringes should not be reused. Scientists are
working on a variety of vaccines that would protect people from Ebola or Marburg viruses.
Some of the results have been promising, but further testing is needed
Text 1: Questions 7-14
7. The Ebola and Marburg Viruses are native to;

A. America
B. Japan
C. Africa
D. China

8. What is right about Ebola and Marburg viruses?

A. Spread from person to person only.


B. Spread from animals to humans.
C. Spread from animals to animals.
D. Spread person to person after initial transmission from the infected animals.

9. Symptoms are typically seen within;

A. Five days
B. Ten days
C. Five to seven days
D. Five to ten days

10. In the Philippines, Ebola was discovered in;

A. Chimpanzees
B. Human primates
C. Non-human primates
D. Monkeys
11. Most known Ebola diseases occur due to;

A. Contamination
B. Bodily fluids
C. Contaminated needles and syringes
D. None

12. People with hemorrhagic fever show;

A. High number of viruses in their blood


B. Low concentrations of virus
C. High concentrations of antibodies
D. Low concentrations of antibodies

13. Pick one of the best preventive measures stated in the passage here;

A. Hand cleaning with medicinal soap.


B. Use of alcohol-based hand rubs, containing at least 60% alcohol, in absence of water &
soap.
C. Only use of soap.
D. Avoiding direct contact with patients is a necessity.

14. As a healthcare worker, you should;

A. Keep infected people totally isolated from others.


B. Not reuse needles and syringes for the second time.
C. Wear clothing such as gowns and eye shields.
D. none of the above
Text 2: A Chronic Disease - Atopic Dermatitis

Atopic dermatitis is a common chronic skin disease. It is also called atopic eczema.
Atopic is a term used to describe allergic conditions such as asthma and hay fever.
Both dermatitis and eczema mean inflammation of the skin. People with atopic
dermatitis tend to have dry, itchy and easily irritated skin. They may have times
when their skin is clear and other times when they have rash. In infants and small
children, the rash is often present on the skin around the knees and elbows and the
cheeks. In teenagers and adults, the rash is often present in the creases of the
wrists, elbows, knees or ankles, and on the face or neck.

Atopic dermatitis usually begins and ends during childhood, but some people
continue to have the disease into adulthood. If you have ever had atopic dermatitis,
you may have trouble with one or more of these: dry, sensitive skin, hand
dermatitis and skin infections. The exact cause of atopic dermatitis is unknown.
Research suggests that atopic dermatitis and other atopic diseases are genetically
determined; this means that you are more likely to have atopic dermatitis, food
allergies, asthma and/or hay fever if your parents or other family members have ever
had atopic dermatitis. These diseases may develop one after another over a period of
years. This is called the “atopic march”.

Knowing that a child with a slight wheeze has had a history of atopic dermatitis,
for example makes it easier to diagnose the subtle onset of asthma. There are many
things that make the itching and rash of atopic dermatitis worse. When you learn
more about atopic dermatitis and how to avoid things that make it worse, you may
be able to lead a healthier life.

If you have a reaction to something you touch, breathe or eat, you might have an
allergy. Allergies can trigger or worsen your atopic dermatitis symptoms. Common
causes of allergy are: dust mites, furry and feathered animals, cockroaches, pollen,
mold, foods, chemicals. Your healthcare provider may recommend allergy testing
and food challenges to see if allergies worsen itching or rashes. Allergy testing
may include skin testing, blood tests or patch tests. Many measures can be taken to
avoid things to which you are allergic. Although many of the measures can be done
for the entire home, the bedroom is the most important room to make skin friendly.
Talking with healthcare provider about what measures you can take to avoid your
allergens can be very beneficial.

Food allergies may be the cause of itching or rashes that occur immediately after
eating, especially in children. Some common food allergens include milk, eggs,
peanuts, wheat, nuts, soy and seafood. Most people are allergic to only one, two or
at the most three foods. Be aware that diet restrictions can lead to poor nutrition and
growth delay in babies and children. Talk with your healthcare provider about
maintaining a well-balanced diet.

Emotions and stress do not cause atopic dermatitis, but they may bring on itching
and scratching. Anger, frustration and embarrassment can cause flushing and
itching. Day to day stresses as well as major stressful events can lead to or worsen
the itch-scratch cycle.The medications used in atopic dermatitis include topical
steroids, topical immunomodulators, tar products, anti-infectives and
antihistamines. Steroid medicines that are applied to the skin are called topical
steroids. Topical steroids are drugs that fight inflammation; they are very helpful
when a rash is not well controlled. Topical steroids are available in many forms
such as ointments, creams, lotions and gels. It is important to know that topical
steroids are made in low to super potent strengths. Steroid pills or liquids, like
prednisone, should be avoided because of side effects and because the rash often
comes back after they are stopped.
Text 2: Questions 15-22
15. People with atopic dermatitis suffer from;

A. Hay fever
B. Asthma
C. Dry, itchy and irritated skin
D. Rashes

16. In small children, a rash is seen;

A. Around elbows
B. On the face
C. On the neck
D. Around the knees

17. People with atopic dermatitis have;

A. Dry skin
B. Skin infections
C. Hand dermatitis
D. All of the above

18. The term atopic refers to;

A. Allergic diseases
B. Asthma and hay fever.
C. Allergic conditions like hay fever.
D. Allergic conditions like asthma.

19. _____ can worsen dermatitis symptoms;

A. Allergies
B. Pollen
C. Dust
D. Mold
20. According to the information given in the passage, avoiding allergens is;

A. Easy
B. Difficult
C. Sometimes easy and sometimes difficult
D. Can say

21. Allergic conditions like asthma in patients who have had a history of atopic
dermatitis can be easily diagnosed by health professionals, this statement is;

A. Out of the paragraphs given


B. False
C. True
D. Can be true or can be false

22. Topical steroids are available in these forms:

A. Gel tubes
B. Ointments
C. Lotions
D. Ointments, creams, lotions and gels.

END OF READING TEST, THIS BOOKLET WILL BE COLLECTED


Reading test 9 : Answer Key

Part A - Answer key 1 – 7


1: C
2: D
3: A
4: A
5: C
6: A
7: B

Part A - Answer key 8 – 14


8: autonomic nervous system
9: freeze
10: Ocular-Vestibular
11: impulsivity
12: Eustress
13: asymmetrical tonic neck reflex
14: visual processing disorder

Part A - Answer key 15 – 20


15: Saccades
16: Bonding Response
17: Babkin Palmomental
18: Hypersensitive
19: Inadequate Bonding
20: Motor Programming
Reading test - part B – answer key

1: AIDS-related deaths increased more significantly


2: Preparing radix Sophorae tonkinensis samples.
3: Trials performed produced negative results.
4: Type A is known to produce more adverse reactions than all the others
present in the table.
5: Study performed with focus on B12 usage.
6: 32.72 patients were in the age group of 50-60 years.

Reading test - part C – answer key

Text 1 - Answer key 7 – 14


7: Africa
8: Spread person to person after initial transmission from the infected animals.
9: Five to ten days
10: Monkeys
11: None
12: High number of viruses in their blood
13 : Use of alcohol-based hand rubs, containing at least 60% alcohol, in absence of
water & soap.
14: Not reuse needles and syringes for the second time.

Text 2 - Answer key 15 – 22


15: Dry, itchy and irritated skin
16: Around the knees
17: All of the above
18: Asthma and hay fever.
19: Allergies
20: Easy
21: True
22: Ointments, creams, lotions and gels.
READING TEST 10
READING SUB-TEST : PART A
 Look at the four texts, A-D, in the separate Text Booklet.
 For each question, 1-20, look through the texts, A-D, to find the relevant information.
 Write your answers on the spaces provided in this Question Paper.
 Answer all the questions within the 15-minute time limit.
 Your answers should be correctly spelt.
TEXT BOOKLET- BRAIN IMAGING IN CHILDREN

Text A

Brain Imaging In Children

A headache is a result of disorders that affect pain-sensitive sites, such as meninges,


blood vessels, paranasal sinuses and muscles; it is also one of the most common causes of
pain in children. The prevalence of severe and frequent headaches is 25.25 per thousand in
children. A large number of physicians and parents of sick children are concerned that inter-
carcinoma lesions may be due to headaches. The most important means to diagnose the cause
of headaches is to take precise medical history and para-clinical measures.
Brain imaging method, including CT scan and MRI, is one of the diagnostic methods for
children with headaches. The main reason for performing MRI and CT scans in patients with a
headache is the diagnosis of curable lesions, which can increase the patient’s longevity or
improve her quality of life. Cases such as brain tumors, hydrocephalus vascular malformations,
and subdural hematoma are in this category. Another important cause for conducting MRI in
headache patients is a way to alleviate their anxiety due to the presence of a brain tumor or
intracranial disease.
Text B

Cases in which imaging should be done include important changes in the type of
headache, headache worsening, sudden development of headache or when it is
stimulated by awakening of sleep, and when it is associated with a neurological
symptom. It has been reported that there are abnormal findings in the imaging of
patients, who are suspected of having a headache based on the findings of the
study. Secondary pathologic factors are more common than those in the general
population of headache patients. Parents’ anxiety and their concern about
headaches in their children and the availability of imaging measures have caused
most children to experience CT scan and radiation due to the importance of
headaches in children and due to limited studies in under developing countries.
Patients below 12 years of age were asked for, imaging actions, such as CT scan
and MRI, were the main inclusion criteria. The exclusion criteria included being
older than 12 years of age, lack of consent to enter the study, and defects in the
medical records. Subsequently, the CT and MRI reports of these patients were
reviewed and the findings were recorded. Abnormal findings in the CT scan include mass,
cysts, infarcts, hydrocephalus, calcification, hematoma, ventricular dilation and edema.
Abnormal findings in MRI include sinusitis,retinal cysts, masses, cysts, atrophy, ventricular
dilatation, age variations, hydrocephalus, hematoma, demyelinating disease, mastoiditis,
encephalomalacia, schizophyllum and hypoplasia of corpus callosum, the prevalence of which
were measured after collecting reports.
Text C

All patient information, including demographic factors and para-clinical


symptoms, were recorded in a researcher-made check list and entered into SPSS.
In the descriptive part, abnormal CT scan and MRI findings are presented as the
main variable in different groups. All of the demographic and clinical
characteristics of patients were also reported based on descriptive criteria. In the
analytical section, based on statistical assumptions, parametric and nonparametric
proportional tests were used. CHI-SQUARE test was used to analyze the
qualitative findings and independent T-test was used to compare quantitative
data; non-parametric Mann-Whitney was used if the initial assumptions were not
as normal as they were supposed to. All tests were examined at a 5% error level.
Text D

Total amount of 353 people were included in the study, of which 7 were excluded
during the study. In the first group, CT scan was performed on 217 patients, of
which 85 subjects were girls and 132 were boys and 11.1% were abnormal. In the
second group, 136 people were subjected to MRI, of which 56 (41.1%) were
female and 80 (58.8%) were male, and 24.3% were abnormal. Also, according to
gender segregation, abnormal findings in CT scan were significantly higher in
boys (63% boys and 37% girls) (P = 0.03), and it was also found that MRI findings
were also significantly more common in boys in comparison to girls (66% vs.
34%) (P = 0.04). The results of the study showed that the most common CT scan
abnormal finding was mass (21%) and hematoma (21%). Then, cysts (14%) and
Ventriculomegaly (14%) were the most frequent forms of abnormality. Prevalently
found abnormality was cysts in MRI (30%). Atrophy (12%) and Ventriculomegaly
had the second and third frequencies (15%) (P> 0.05).
PART A -QUESTIONS AND ANSWER SHEET
Questions 1-7
For each question, 1-7, decide which text (A, B, C or D) the information comes from.
You may use any letter more than once.
In which text can you find information about;

1. Research methodology. Answer________________

2. Patients who underwent study. Answer ________________

3. Importance of detection of the causes in early stages. Answer ____________________

4. Reasons why imaging shall be done. Answer _______________________

5. Data segregation. Answer __________________

6. Condition in which there is an accumulation of cerebrospinal fluid (CSF) within the brain. Answer
__

7. A small localized area of dead tissue resulting from failure of blood supply. Answer ________

Questions 8-14
Answer each of the questions, 8-14, with a word or short phrase from one of the texts.
Each answer may include words, numbers or both. Your answers should be correctly spelt.

8. Which age group patients were included in the study? Answer ______________

9. What is the reason presented for taking MRI in children with headache? Answer _______________

10. What is define as inflammation of the air cavities within the passages of the nose? Answer __

11. Was any defect in medical records an inclusion criteria? Answer _________________

12. Who shows more peculiarities in CT imaging? Answer ________________

13. What method was followed if the initial assumptions were abnormal? Answer ______________

14. What is known to be more common with respect to CT scanning? Answer ______________
Questions 15-20

Complete each of the sentences, 15-20, with a word or short phrase from one of the texts.
Each answer may include words, numbers or both. Your answers should be correctly spelt.

15. __________ is referred to as a solid swelling of clotted blood within the tissues.

16. The __________ were presented in two sections of descriptive and analytical.

17. The percentage of boys who underwent CT scan is _________________

18. _______________ can be defined as a brain condition that occurs in the fetus when the lateral
ventricles become dilated

19. A _____ is any disease of the nervous system in which the myelin sheath of neurons is damaged.

20. ______________ can be defined as the general physiological process of reabsorption and
breakdown of tissues.

END OF PART A, THIS QUESTIONS PAPER WILL BE COLLECTED


READING SUB-TEST : PART B
In this part of the test, there are six short extracts relating to the work of health professionals .
For questions 1-6, choose the answer (A, B or C) which you think fits best according to the text.
Write your answers on the separate Answer Sheet
Questions 1-6

1. According to the notice given


A. health rate improved from 2003-2017

B. many disparities present in 2012-2017 widened over time

C. increase in physical activity led to improved health score

Nutrition
Between 2003 to 2004 and 2016 to 2017 in the United States, the mean AHA healthy diet
score improved in both children and adults. The prevalence of an ideal healthy diet score
increased from 0.2 percent to 0.6 percent in children and from 0.7 percent to 1.5 percent in
adults. These improvements were largely attributable to increased whole grain consumption
and decreased sugar-sweetened beverage consumption in both children and adults, as well
as a small, non-significant trend in increased fruit and vegetable consumption. No major
trends were evident in children or adults in progress toward the targets for consumption of
fish or sodium. Between 2012 and 2017, although AHA healthy diet scores tended to improve
in all race/ethnicity, income, and education levels, there has been constant inequalities as
well. with generally smaller improvements seen in minority groups and those with lower
income or education.
About one in every three US adults or 30.4 percent, do not engage in leisure time physical
activity. Hispanic and Non-Hispanic black adults were more likely to be inactive. Among
students in grades 9-12, only about 27.1 percent meet the American Heart Association
recommendation of 60 minutes of exercise every day. More high school boys (36 percent)
than girls (17.7 percent) reported having been physically active at least 60 minutes per day
on all 7 days.
2. The term which is more close to condition in which the child may reply
A. instantly somnolence
B. hypotonia
C. infantile reflex

Acute Bilirubin Encephalopathy (ABE)


ABE is a term to describe the variable spectrum, from subtle to advanced manifestations of
bilirubin toxicity present in the first weeks of life (American Academy of Pediatrics
Subcommittee on Hyperbilirubinemia, 2004; Van Praagh, 1961). Symptoms associated with
ABE include a range of neurological manifestations, sleeping disorder, somnolence,
hypotonia, loss of the Moro reflex, followed by a stage characterized by hypertonia of the
extensor muscle groups (backward arching of the neck and backward arching of the trunk).
Additionally, fever and/or a high pitched cry may be present. Different investigators have
used different methods to characterize and define clinical manifestations of ABE in the infant

3. The notice
A. places women at the centre of the care
B. explains what can be done in order to decrease mortality rate
C. talks of standards that ensure healthy life

Care For Women


Undoubtedly, encouraging women to give birth in health facilities, where there are skilled
birth attendants, is essential and has helped reduce global newborn and maternal mortality
rates for decades. However, there is room for improvement in the quality of care provided in
these facilities. To meet Sustainable Development Goal 3 of ensuring healthy lives and
promoting well-being for all at all ages, we cannot keep our focus solely on survival. High
quality care for all pregnant women and their newborns, throughout pregnancy, childbirth
and the postnatal periods, is essential to ensure that mothers and children both survive and
thrive.
4 .The notice clearly explains that
A. disease control is a still a big problem

B. disease has successfully been managed

C. disease is widespread and is affecting population across the globe

Wild polio
Overall the Committee was encouraged by continued progress in WPV1 eradication, with the
number of cases globally falling to an all-time low in 2017. In addition, there has been no
international spread of WPV since the fifteenth meeting in November 2017.
The Committee commended the continued high level commitment seen in sub continents
and the high degree of cooperation and coordination, particularly targeting the high risk
mobile populations that cross the international border, such as nomadic groups, local
populations straddling the border, seasonal migrant workers and their families, repatriating
refugees (official and informal), and guest children (children staying with relatives across the
border). Stopping transmission in these populations is going beyond efforts and cannot be
underestimated, underlining the critical continuing need for cross border activities in
surveillance and vaccination
5. The notice talks more about
A. how ICD works

B. ICD purpose and uses

C. actions undertaken by the ICD

ICD - Health Trends

ICD is the foundation for the identification of health trends and statistics globally, and the
international standard for reporting diseases and health conditions. It is the diagnostic
classification standard for all clinical and research purposes. ICD defines the universe of
diseases, disorders, injuries and other related health conditions, listed in a comprehensive,
hierarchical fashion that allows for:
 easy storage, retrieval and analysis of health information for evidence-based decision-making
 sharing and comparing health information between hospitals, regions, settings and
countries;
 and data comparisons in the same location across different time periods.
Uses include monitoring of the incidence and prevalence of diseases, observing
reimbursements and resource allocation trends, and keeping track of safety and quality
guidelines. They also include the counting of deaths as well as diseases, injuries, symptoms,
reasons for encounter, factors that influence health status, and external causes of disease
6. What is correct?
A. Ebola virus is common among Waganta population

B. There has been widespread prevalence of the virus in Iboko

C. Virus in Bikoro is spreading at a rate closer to Waganta

Distribution of Ebola virus disease cases by health zone in Democratic Republic of the
Congo, 1 April – 9 July 2018
READING SUB-TEST : PART C
In this part of the test, there are two texts about different aspects of healthcare.
For questions 7-22, choose the answer (A, B, C or D) which you think fits best according to the text.
Write your answers on the separate Answer Sheet

Text 1: Viral Infection – Yellow Fever

Yellow fever is a viral infection spread by a particular species of mosquito. It's


most common in the areas of Africa and South America, affecting both travellers
to and residents of those areas. In mild cases, it causes fever, headaches, nausea
and vomiting. However, it can become more serious, causing heart, liver and
kidney problems along with bleeding (haemorrhaging). Up to 50 percent of people
with the more severe form of yellow fever die of the disease. There's no specific
treatment for yellow fever, but getting a yellow fever vaccine before travelling to
an area in which the virus is known to exist can protect you from the disease.
During the first three to six days after contracting yellow fever — the incubation
period — there won’t be any signs or symptoms of the disease. After this, the
virus enters an acute phase and, in some cases, a toxic phase follows which can be
life-threatening.

Once the yellow fever virus enters the acute phase, you may experience signs and
symptoms including: fever, headaches, muscle aches - particularly in your back
and knees - nausea, vomiting or both, loss of appetite, dizziness, red eyes, face or
tongue. These signs and symptoms usually improve and disappear within several
days. Although signs and symptoms may disappear for a day or two following the
acute phase, some people with acute yellow fever then enter a toxic phase. During
the toxic phase, acute signs and symptoms return and more severe and life threatening one
also appear. These can include yellowing of the skin and the whites of the eyes
(jaundice), abdominal pain and vomiting - sometimes of blood - decreased urination,
bleeding from your nose, mouth and eyes, heart dysfunction (arrhythmia), liver and kidney
failure, and brain dysfunction, including delirium, seizures and coma.
Make an appointment to see your doctor four to six weeks before travelling to an
area in which yellow fever is known to occur. If you don't have that much time to
prepare, call your doctor anyway. Your doctor will help you determine whether
you need vaccinations and can provide general guidance on protecting your health
while abroad. Seek emergency medical care if you've recently travelled to a
region where yellow fever is known to occur and you develop severe signs or
symptoms of the disease. Even if you develop mild symptoms, call your doctor.
Yellow fever is caused by a virus that is spread by the Aedes aegypti mosquito.
These mosquitoes thrive in and near human habitations where they can breed in
even the cleanest water. Most cases of yellow fever occur in sub-Saharan Africa
and tropical South America.

Humans and monkeys are most commonly infected with the yellow fever virus;
mosquitoes transmit the virus back and forth between monkeys, humans or both.
When a mosquito bites a human or monkey infected with yellow fever, the virus
enters the mosquito's bloodstream and circulates before settling in the salivary
glands. When the infected mosquito bites another monkey or human, the virus
then enters the host's bloodstream, where it may cause the illness to develop.

You may be at risk of the disease if you travel to an area where mosquitoes continue
to carry the yellow fever virus. These areas include sub-Saharan Africa and tropical
South America. Even if there aren't current reports of infected humans in these areas,
it doesn't necessarily mean you're risk-free. It's possible that local populations have
been vaccinated and are protected from the disease, or that cases of yellow fever just
haven't been detected and officially reported. If you're planning on travelling to these
areas, you can protect yourself by getting a yellow fever vaccine at least 10 to 14
days before travelling. Anyone can be infected with the yellow fever virus, but older
adults are at greater risk of becoming seriously ill.
Diagnosing yellow fever based on signs and symptoms can be difficult because, early
in its course, the infection can be easily confused with malaria, typhoid, dengue fever
and other viral hemorrhagic fevers. To diagnose your condition, your doctor will
likely: Ask questions about your medical and travel history, collect a blood sample
for testing. If you have yellow fever, your blood may reveal the virus itself. If not, blood tests
called enzyme-linked immunosorbent assay (ELISA) and polymerase chain reaction (PCR) can
also detect antigens and antibodies specific to the virus. Results from these tests may take
several days. No antiviral medications have proved helpful in treating yellow fever and, as a
result, treatment consists primarily of supportive care in a hospital. This includes providing
fluids and oxygen, maintaining adequate blood pressure, replacing blood loss, providing
dialysis for kidney failure, and treating any other infections
that develop. Some people receive transfusions of plasma to replace blood proteins that
improve clotting. If you have yellow fever, you may also be kept away from mosquitoes, to
avoid transmitting the disease to others.
Text 1: Questions 7-14
7. Yellow fever is common in;

A. Africa
B. South America
C. Both
D. Not given

8. Which of the following is not a sign of yellow fever?

A. Back pain
B. Vomiting
C. Nausea
D. Dry tongue

9. Signs/symptoms of toxic phase;

A. Loss of appetite
B. Yellowness of eyes
C. Brain dysfunction
D. B and C

10. Seizures may occur during;

A. Acute phase
B. Toxic phase
C. Sometimes in both the phases
D. Not given
11. Yellow fever, which is a viral disease, is spread by;

A. Aedes albopictus mosquito


B. Aedes aegypti mosquito
C. Female aedes mosquito
D. Male aedes mosquito

12. Mosquitoes transmit the virus from

A. Humans to monkeys
B. Monkeys to humans
C. Human to human
D. A&B

13. Travelling to areas where the disease is common is recommended after vaccination of;

A. 10 days
B. 12 days
C. 14 days
D. 10-14 days

14. Supportive care in hospitals includes;

A. Free medical check up


B. Free BP check up
C. Effective dialysis procedures in case of kidney failure
D. B and C
Text 2: Aortic Dissection or Dissecting Aneurysm

An aortic dissection is a serious condition in which a tear develops in the inner


layer of the aorta, the large blood vessel branching off the heart. Blood surges
through this tear into the middle layer of the aorta, causing the inner and middle
layers to separate (dissect). If the blood-filled channel ruptures through the outside
aortic wall, aortic dissection can be fatal. Aortic dissection, also called a dissecting
aneurysm, is relatively uncommon. Anyone can develop the condition, but it most
frequently occurs in men between 60 and 70 years of age. Symptoms of aortic
dissection may mimic those of other diseases, often leading to delays in diagnosis.
However, when an aortic dissection is detected early and treated promptly, your
chance of survival greatly improves.

Aortic dissection symptoms may be similar to those of other heart problems, such
as a heart attack. Typical signs and symptoms include sudden severe chest or
upper back pain (often described as a tearing, ripping or tearing sensation, that
radiates to the neck or down the back), loss of consciousness (fainting), shortness
of breath, sweating, weaker pulse in one arm compared to the other etc. If you
have signs or symptoms such as severe chest pain, fainting, sudden onset of
shortness of breath or symptoms of a stroke then seeking medical assistance is
imperative. While experiencing such symptoms doesn't always mean that you
have a serious problem, it's best to get checked out quickly because early detection
and treatment may help to save your life.

An aortic dissection occurs in a weakened area of the aortic wall. Chronic high
blood pressure may stress the aortic tissue, making it more susceptible to tearing.
You can also be born with a condition associated with a weakened and enlarged
aorta, such as Marfan syndrome or bicuspid aortic valve. Rarely, aortic
dissections may be caused by traumatic injury to the chest area, such as during
motor vehicle accidents.
Aortic dissections are divided into two groups, depending on which part of the aorta is
affected: Type A: This is the more common and dangerous type of aortic dissection. It
involves a tear in the part of the aorta just where it exits the heart or a tear extending from
the upper to lower parts of the aorta, which may extend into the abdomen. Type B: This type
involves a tear in the lower aorta only, which may also extend into the abdomen. Risk factors
for aortic dissection include Uncontrolled high blood pressure (hypertension), found in at
least two thirds of all cases, Hardening of the arteries (atherosclerosis), Weakened and
bulging artery(pre-existing aortic aneurysm), aortic valve defect (bicuspid aortic valve), A
narrowing of the aorta you're born with (aortic coarctation)

People with certain genetic diseases are more likely to have an aortic dissection
than other people. These genetic diseases include Turner's syndrome, high blood
pressure, heart problems, and a number of other health conditions may be a result
of this disorder. Marfan syndrome: This is a condition in which connective tissue,
which supports various structures in the body, is weak. People with this disorder
often have a family history of aneurysms of the aorta and other blood vessels.
These weak blood vessels are prone to tears (dissection) and rupture easily.
Ehlers Danlos syndrome: This group of connective tissue disorders is characterized by
skin that bruises or tears easily, loose joints and fragile blood vessels. Loeys-Dietz
syndrome: This is a connective tissue disorder marked by twisted arteries, especially in the
neck. People who have Loeys-Dietz syndrome are thought to be at risk of developing aortic
dissections and aneurysms.

An aortic dissection can lead to death, due to severe internal bleeding, including into
the lining around the heart (pericardial sac), organ damage (such as kidney failure or
life-threatening damage to the intestines), strokes (possibly including paralysis), and
aortic valve damage, such as causing the aortic valve to leak (aortic regurgitation).
Detecting an aortic dissection can be tricky because the symptoms are similar to
those of a variety of health problems. Doctors often suspect an aortic dissection if the
following signs and symptoms are present: sudden tearing or ripping chest pain, widening of
the aorta on a chest X-ray, blood pressure difference between the right and left arms.
Text 2: Questions 15-22
15. In aortic dissection a tear develops in;
A. Outer layer of aorta.
B. Inner layer of aorta.
C. Middle aorta.
D. A blood vessel branching off the heart.

16. Dissecting aneurysm is common among;


A. Men
B. Women
C. Both
D. Children

17. Symptoms of aortic dissection include;


A. Chest pain and swelling
B. Weak pulse in both arms
C. Loss of consciousness.
D. All of the above

18. Aortic dissection can also be caused due to;


A. High BP
B. Weak aortic wall
C. Inborn symptoms
D. Traumatic injury to chest during accidents

19. The most dangerous type of aortic dissection is;


A. Type A
B. Type B
C. Aortic aneurysm
D. Aortic coarctation
20. A condition in which connective tissue is weak is called;

A. Turners syndrome
B. Loeys-Dietz syndrome
C. Ehlers-Danlos syndrome
D. Marfans syndrome

21. People with Loeys-Dietz syndrome are likely to develop;

A. Aneurysms
B. Ruptured blood vessels
C. Twisted arteries in the neck
D. Aortic complications

22. Detecting aortic dissection is;

A. Easy
B. Difficult
C. Impossible
D. Sometimes possible

END OF READING TEST, THIS BOOKLET WILL BE COLLECTED


Reading test 10 : Answer Key

Part A - Answer key 1 – 7


1: C
2: D
3: A
4: B
5: C
6: A
7: B

Part A - Answer key 8 – 14


8: below twelve years
9: diagnosis of curable lesions
10: sinusitis
11: no
12: boys
13: non-parametric Mann-Whitney
14: mass and hematoma

Part A - Answer key 15 – 20


15: Hematoma
16: Statistical Analyzes
17: 61%
18: Ventriculomegaly
19: Demyelinating Disease
20: Atrophy
Reading test - part B – answer key 1 – 6

1: many disparities present in 2012-2017 widened over time


2: infantile reflex
3: places women at the centre of the care
4: disease control is a still a big problem
5: ICD purpose and uses
6: There has been widespread prevalence of the virus in Iboko

Reading test - part C – answer key

Text 1 - Answer key 7 – 14


7: Both
8: Dry tongue
9: B and C
10: Toxic phase
11: Aedes aegypti mosquito
12: A & B
13: 10-14 days
14 : B and C

Text 2 - Answer key 15 – 22


15: Inner layer of aorta.
16: Men
17: Loss of consciousness.
18: Traumatic injury to chest during accidents
19: Type A
20: Marfans syndrome
21: Aneurysms
22: Difficult
READING TEST 11
READING SUB-TEST : PART A

 Look at the four texts, A-D, in the separate Text Booklet.


 For each question, 1-20, look through the texts, A-D, to find the relevant information.
 Write your answers on the spaces provided in this Question Paper.
 Answer all the questions within the 15-minute time limit.
 Your answers should be correctly spelt.

TEXT BOOKLET -ECZEMA / DERMATITIS]

Text A

Eczema / Dermatitis
The words `eczema` and `dermatitis` are often used synonymously to describe a
polymorphic pattern of inflammation, which in the acute phase is characterized by
erythema and vesiculation, and in the chronic phase by dryness, lichenification and
fissuring. Contact dermatitis describes these patterns of reaction in response to
external agents, which may be acting either as irritants, where the T cell-mediated
immune response is not involved, or as allergens, where cell-mediated immunity is
involved. Contact dermatitis may be classified into the following reaction types:
Subjective irritancy ± idiosyncratic stinging and smarting reactions that occur
within minutes of contact, usually on the face, in the absence of visible changes.
Cosmetic or sunscreen constituents are common agents. Acute irritant contact
dermatitis is often the result of a single overwhelming exposure or a few brief
exposures to strong irritants or caustic agents. Chronic irritant contact dermatitis
occurs following repetitive exposure to weaker irritants, which may be either `wet`,
such as detergents, organic solvents, soaps, weak acids and alkalis, or `dry`, such
as low humidity air, heat, powders and dust.
Text B

Patch testing
The mainstay of diagnosis in allergic contact dermatitis is the patch test. This test
has a sensitivity and specificity of between 70% and 80%15. Patch testing involves
the reproduction under the patch tests of allergic contact dermatitis in an individual
sensitized to a particular antigen(s). The standard method involves the application of
the antigen to the skin at standardized concentrations in an appropriate vehicle and
under occlusion. The back is most commonly used principally for convenience
because of the area available, although the limbs, in particular the outer upper arms,
are also used. Various application systems are available of which the most commonly
used are Finn chambers. With this system, the investigator adds the individual
allergens to test discs that are loaded on to adhesive tape. Available are available ±
the TRUE and the Epiquic tests. There are few comparative studies between the
different systems. Pre-prepared tests are significantly more reliable than operator-
prepared tests. There is also some evidence that larger chambers may give more
reproducible tests. However, this may only apply to some allergens.
The open patch test, not so common, is used where potential irritants or sensitizers
are being assessed. It is also useful in the investigation of contact urticaria and
protein contact dermatitis. The open patch test is usually performed on the forearm
but the upper outer arm or scapular areas may also be used. The site should be
assessed at regular intervals for the first 30±60 min and a later reading should be
carried out after 3±4 days. A repeated open application test, applying the suspect
agent on to the forearm, is also useful in the assessment of cosmetics, where
irritancy or combination effects may interfere with standard patch testing. This
usually involves the application of the product twice daily for up to a week,
stopping if a reaction develops.
Text C

Photopatch testing
Where photoallergic dermatitis is suspected, photopatch testing may be carried out.
Very briefly, the standard method of photopatch testing involves the application of
the photo allergen series and any suspected materials in duplicate on either side of
the upper back. One side is irradiated with ultraviolet (UV) after an interval (1 or 2
days) and readings are taken in parallel after a further 2 days. The exact intervals
for irradiation and the dose of UVA given vary from centre to centre. The U.K.
multicentre study into photopatch testing has now been completed and published.
It is recommended that allergens be subjected to 5 J cm2 UVA and a reading to be
taken after 2 days. The incidence of photo allergy in suspected cases was low at
below 5%; however, further readings at 3 and 4 days increased the detection rate.

Text D

There are a number of aspects, which can have their effect on the accuracy of patch
testing. Principal among these are the characteristics of the individual allergens and
the method of patch testing. Some allergens are more likely to cause irritant
reactions than others. These reactions may be difficult to interpret and are easily
misclassified as positive reactions. Nickel, cobalt, potassium dichromate and carba
mix are the notable offenders in the standard series. As indicated above, preprepared
patch tests are better standardized in terms of the amount of allergen
applied and are therefore more reproducible, but are prohibitively expensive in the
U.K. Patient characteristics are also important. It is essential that the skin on the
back is free from dermatitis and that skin disease elsewhere is as well controlled as
possible. This will help to avoid the `angry back syndrome` with numerous false
positives. However, if a patient applies topical steroids to the back up to 2 days
prior to the test being applied or is taking oral corticosteroids or
immunosuppressant drugs, then there is a significant risk of false negative results.
PART A -QUESTIONS AND ANSWER SHEET

Questions 1-7
For each question, 1-7, decide which text (A, B, C or D) the information comes from.
You may use any letter more than once.

In which text can you find information about;


1. One of the most common tests. Answer _______________
2. The blockage or closing of a blood vessel or hollow organ. Answer __________
3. Important factors that may influence the testing results. Answer ____________
4 .The condition that occurs in response to excessive itching or rubbing of the skin. Answer __
5. Two different types of tests which help in diagnosing the disease condition. Answer _
6 .The process or fact of irradiating or being irradiated. Answer ___________
7 .Splitting or cracking of the skin. Answer __________

Questions 8-14

Answer each of the questions, 8-14, with a word or short phrase from one of the texts.
Each answer may include words, numbers or both. Your answers should be correctly spelt.

8. Which test is known to be more helpful with respect to the determination of


other irritants, which make it difficult to carry out other tests? Answer __________
9. What are given orally or by injection and distribute throughout the body? Answer _____
10. In which process do we apply the product two times? Answer ______________
11. What would be the outcome of the pre-prepared patch test, if there is dermatitis on the back of
the skin? Answer ______________
12. What are the common precipitants of contact dermatitis? Answer ____________
13 .What would be the outcome of the test, if the patient applies topical steroids two days prior to
the test? Answer _____________
14 .What was the proportion of photoallergic incidents in suspected cases? Answer __________
Questions 15-20
Complete each of the sentences, 15-20, with a word or short phrase from one of the
texts. Each answer may include words, numbers or both. Your answers should be correctly spelt.

15. ___________ as the cumulative irritant is known to create more worsening conditions.

16. The open patch is carried in order to examine the potentiality of the _______ or irritants.

17 .The chronic phase of a polymorphic pattern of inflammation is characterized by __________.

18. Carba mix is one of the _____________ in the standard series.

19. The standard method of testing involves the application of the___________.

20. As per recommendations, allergens be subjected to ________________.

END OF PART A, THIS QUESTIONS PAPER WILL BE COLLECTED


READING SUB-TEST : PART B
In this part of the test, there are six short extracts relating to the work of health professionals .
For questions 1-6, choose the answer (A, B or C) which you think fits best according to the text.
Write your answers on the separate Answer Sheet

Questions 1-6

1. The notice talks about;


A. How blood clots occur.
B. Blood clot doesn`t form. (if the reaction doesn`t occur the way it should)
C. Blocking of clotting reaction.

VKCFD
Inherited combined deficiency of the vitamin K dependent clotting factors is a very
rare inherited bleeding disorder that is caused by a problem with clotting Factors
II, VII, IX, and X. VKCFD can also be acquired later in life as a result of other
disorders, or certain medications such as the blood-thinning drug Coumadin.
Acquired VKCFD is more common than the inherited form. Some newborn babies
have a temporary vitamin K deficiency, which can be treated with supplements at
birth. In order to continue the chain reaction of the coagulation cascade, these four
factors need to be activated in a chemical reaction that involves vitamin K.
VKCFD is an autosomal recessive disorder, which means that both parents must
carry the defective gene in order to pass it on to their child. It also means that the
disorder affects both males and females.
2. What is correct?
A. All types of Factor I deficiency affect both males and females.
B. The fibrogen defect in impairment leads to disorder.
C. Genes can be both recessive or dominant.

Factor I deficiency

It is an umbrella term for several related disorders in males and females, known as
congenital Fibrinogen defects. Afibrinogenemia (a lack of Fibrinogen) and
hypofibrinogenemia (low levels of Fibrinogen) are quantitative defects, meaning
the amount of Fibrinogen in the blood is abnormal. Dysfibrinogenemia is a
qualitative defect in which Fibrinogen does not work the way it should.
Hypodysfibrinogenemia is a combined defect that involves both low levels of
Fibrinogen and impaired function. Afibrinogenemia is an autosomal recessive
disorder, which means that both parents must carry the defective gene in order to
pass it on to their child. Hypofibrinogenemia, dysfibrinogenemia, and
hypodysfibrinogenemia can be either recessive (both parents carry the gene) or
dominant (only one parent carries and transmits the gene).
3. What is referred to as weak muscle?
A. Chronic fatigue
B. Brachymesophalangy
C. Hypotonia

2q37 deletion syndrome

It is a condition that can affect many parts of the body. This condition is characterized
by hypotonia in infancy, mild to severe intellectual disability and developmental delay,
behavioral problems, characteristic facial features, and other physical abnormalities.
Most babies with 2q37 deletion syndrome are born with potentially chronic fatigue,
which usually improves with age. About 25 percent of people with this condition have
autism, a developmental condition that affects communication and social interaction.
The characteristic facial features associated with 2q37 deletion syndrome include a
prominent forehead, highly arched eyebrows, deep-set eyes, a flat nasal bridge, a thin
upper lip, and minor ear abnormalities. Other features of this condition can include
short stature, obesity, unusually short fingers and brachymesophalangy, sparse hair,
heart defects, seizures, and an inflammatory skin disorder called eczema. A few people
with 2q37 deletion syndrome have a rare form of kidney cancer called Wilms tumor.
4. The notice talks about;
A. Every kind of procedure is used in the process of sterilization to keep sterile the
objects or articles that are to be introduced into a wound or body cavity or that is
to penetrate the skin;
B. General overview of sterilization techniques;
C. The practices that the nurses will have to focus on; Sterile technique

Surgical asepsis is used to maintain sterilize. Use of effective sterile technique means
that no organisms are carried to the client. Microorganisms are destroyed before they
can enter the body. Sterile technique is used when changing dressings, administering
parenteral (other than the digestive tract) medications, and performing surgical and
other procedures such as urinary catheterization. With surgical asepsis, first articles
are sterilized, and then their contact with any unsterile articles is prevented. When a
sterile article touches an unsterile article, it becomes contaminated. It is no longer
sterile.
5. It is known to prevent viral infection;
A. PEG-IFN, RBV
B. Boceprevir
C. Sofosbuvir

Treatment for HCV


Before the commencement of HCV treatment, it is necessary to genotype the virus as
different genotypes require different types and duration of treatment, and the
protease inhibitors. Current therapy for genotype 1 infection is a combination of PEG-
IFN, RBV and a PI or nucleotide polymerase inhibitor, which results in high rates of
sustained virological response (SVR; a negative HCV RNA test three or six months after
the end of treatment). Boceprevir, simeprevir and telaprevir can also be used. Dual
therapy with PEG-IFN and RBV or sofosbuvir with RBV is used for genotypes 2 and 3
infections. Patients with genotype 4 infection treated with treated with sofosbuvir,
PEG-IFN and RBV have similar response rates when compared with genotype 1-
infected individuals. Small studies of genotypes 5- and 6-infected patients have shown
similar SVR rates to genotypes 2- and 3-infected ones.
6. The notice talks about;
A. Rate of heterogeneity of genitourinary sarcomas.
B. Survival rate and diagnosis.
C. Survival rates of patients with genitourinary sarcomas.

Patients with genitourinary sarcomas

Patients with genitourinary sarcomas are relatively in a bad state, when compared
with other soft tissue regions. Prognosis is relatively poor and can be explained by
the high proportion seen in high degree tumors, a large proportion of patients with
metastatic disease, large tumor and the area affected. In addition, the rarity and
heterogeneity of genitourinary sarcomas can explain the great variability in clinical
progress in different subgroups. Dissemination of urethral cancer follows the
anatomic subdivision. The anterior urethra has a lymphatic drainage system for
superficial and deep inguinal region. Posterior urethra drains the lymphatic
ganglion of the external iliac artery, hypogastric, and internal obturator muscle.
Late diagnosis is seen in one third of patients with inguinal lymphatic ganglion
metastasis and in 20% of those with pelvic ganglion metastasis.
READING SUB-TEST : PART C
In this part of the test, there are two texts about different aspects of healthcare.
For questions 7-22, choose the answer (A, B, C or D) which you think fits best according to the text.
Write your answers on the separate Answer Sheet

Text 1: Cancer and Cervical Cancer

The body is made up of trillions of living cells. Normal body cells grow, divide,
and die in an orderly fashion. During the early years of a person`s life, normal cells
divide faster to allow the person to grow. After the person becomes an adult, most
cells divide only to replace worn-out or dying cells or to repair injuries.

Cancer begins when cells in a part of the body start to grow out of control. There are
many kinds of cancer, but they all start due to out-of-control growth of abnormal cells.
Cells become cancer cells because of damage to DNA. DNA is in every cell and directs all
its actions. In a normal cell, when DNA gets damaged the cell either repairs the damage
or the cell dies. In cancer cells, the damaged DNA is not repaired, but the cell doesn`t die
like it should. Instead, this cell goes on making new cells that the body does not need.
These new cells will all have the same damaged DNA as the first cell does.

No matter where cancer may spread, it is always named after the place where it
started. For example, breast cancer that has spread to the liver is still called breast
cancer, not liver cancer. Likewise, prostate cancer that has spread to the bone is
metastatic prostate cancer, not bone cancer. Different types of cancer can behave
very differently. For example, lung cancer and breast cancer are very different
diseases. They grow at different rates and respond to different treatments. That is
why people with cancer need the treatment that is aimed at their particular kind of
cancer. Not all tumors are cancerous. Tumors that aren`t cancer are called benign.
Benign tumors can cause problems ± they can grow very large and press on healthy
organs and tissues. But they cannot grow into (invade) other tissues. Because they
can`t invade, they also can`t spread to other parts of the body (metastasize). These
tumors are almost never life threatening.
The cervix is the lower part of the uterus (womb). It is sometimes called the uterine
cervix. The body of the uterus (the upper part) is where a baby grows. The cervix
connects the body of the uterus to the vagina (birth canal). The part of the cervix
closest to the body of the uterus is called the endocervix. The part next to the vagina is
the exocervix (or ectocervix). The 2 main types of cells covering the cervix are squamous
cells (on the exocervix) and glandular cells (on the endocervix). The place where these 2
cell types meet is called the transformation zone. Most cervical cancers start in the
transformation zone and in the cells lining the cervix. These cells do not suddenly change
into cancer. Instead, the normal cells of the cervix first gradually develop pre-cancerous
changes that turn into cancer. Doctors use several terms to describe these pre-cancerous
changes, including cervical intraepithelial neoplasia (CIN), squamous intraepithelial lesion
(SIL), and dysplasia. These changes can be detected by the Pap test and treated to
prevent the development of cancer.

Cervical cancers and cervical pre-cancers are classified by how they look under a
microscope. There are 2 main types of cervical cancers: squamous cell carcinoma and
adenocarcinoma. About 80% to 90% of cervical cancers are squamous cell carcinomas.
These cancers are from the squamous cells that cover the surface of the exocervix. Under
the microscope, this type of cancer is made up of cells that are like squamous cells.
Squamous cell carcinomas most commonly begin where the exocervix joins the endocervix.

Most of the other cervical cancers are adenocarcinomas. Cervical adenocarcinomas


seem to have become more common in the past 20 to 30 years. Cervical
adenocarcinoma develops from the mucus-producing gland cells of the
endocervix. Less commonly, cervical cancers have features of both squamous cell
carcinomas and adenocarcinomas. These are called adenosquamous carcinomas or
mixed carcinomas. Although cervical cancers start from cells with pre-cancerous
changes (pre-cancers), only some of the women with pre-cancers of the cervix will
develop cancer. The change from cervical pre-cancer to cervical cancer usually
takes several years, but it can happen in less than a year. For most women, precancerous
cells will go away without any treatment. Still, in some women pre cancers turn into true
(invasive) cancers. Treating all pre-cancers can prevent almost all true cancers.
Text 1: Questions 7-14
7. When a person becomes an adult, the cells divide only to;
A Replace old cells
B Replace dying cells
C Repair injuries
D All of the above

8. The DNA in cancer cells is;


A Damaged, but can be repaired.
B Not totally damaged.
C Permanently damaged, but the cell doesn`t die.
D Damaged but creates new cells with damaged DNA.

9. Cancer in liver having its origin in breast is a;


A Liver cancer
B Breast cancer
C Prostate cancer
D Bone cancer

10. Benign tumors;


A Can grow and invade the tissues.
B Can grow and press on healthy organs and tissues.
C Are cancerous.
D Are highly fatal.

11. Most cervical cancers start in;


A Squamous cells;
B Glandular cells;
C The transformation zone;
D All the above;
12. There are............ main types of cervical cancers
A two
B three
C four
D five

13. About 80 to 90% of cervical cancers are;


A Adenocarcinoma
B Adenosquamous carcinomas
C Mixed carcinomas
D Squamous cell carcinomas

14. Pre-cancerous cells in women can go away;


A With growth in more number of new cells.
B With medications from doctors.
C Without treatment
D Can`t say
Text 2: Hepatitis - Viral Liver Infection

Hepatitis (A, B, or C) can be caused by a virus (Viral Hepatitis), drugs, alcohol,


medications, and blood transfusions. Scientists estimate that between 3.5 and 5.3
million people in the USA are living with Hepatitis. A blood test is required to
diagnose Hepatitis infection. Hepatitis A is a viral liver infection. In most cases the
body easily defeats the virus (much like the flu, which is what you may feel like
you have). Because of this it does not lead to long term liver challenges. Hepatitis
A is the most common form of Hepatitis. It is spread through the feces of a
contaminated person. This can easily be prevented by thoroughly washing hands
after using the restroom, before eating, and after changing a diaper.

Eating raw oysters and undercooked clams can increase your chances of
contracting the virus. If you are traveling in a country where Hepatitis is common
make sure you wash your hands often and well, eat cooked oysters and clams, and
use an antiviral essential oil such as Lemon to help protect yourself. Hepatitis B is
a viral liver infection. Again, most adult bodies are able to fight off the virus. In
this case, it is referred to as Acute (something that does not last long) Hepatitis B.
Hepatitis B is spread through contact with blood or bodily fluids of an infected
person. This can include unprotected sexual intercourse, sharing drug needles,
getting a tattoo with instruments that were not properly cleaned, or by sharing a
personal item such as a razor or toothbrush with an infected person.

A mother who is infected can pass the virus on to her baby during delivery. Again,
the symptoms are flu-like in nature, so it often goes undiagnosed. A person who
has Chronic (lasting three months or more) Hepatitis B may show no symptoms
until liver damage has occurred. Hepatitis B can lead to liver damage or cancer; your
doctor may want to do a biopsy to determine the amount of damage your liver has
experienced. Hepatitis C is also a viral liver infection. A few people will contract Hepatitis C
and get better. This is called Acute Hepatitis C. Most, however, will develop Chronic
Hepatitis C and go on to deal with liver damage,
cirrhosis of the liver, liver cancer, and possibly liver failure. Hepatitis C is the
number one reason for liver transplants in the USA.

Hepatitis C is spread through contact with contaminated blood. This can occur by
sharing a needle, receiving a blood transfusion or organ transplant (blood and organs have
been screened for Hepatitis in the USA since 1992), getting a tattoo with equipment that
has not been properly cleaned, and, in rare cases, a mother can pass the virus on to her
baby during birth. Scientists are not sure, but think there may be a slim possibility that the
virus may be passed through unprotected sexual intercourse.

Symptoms generally do not occur until the virus is causing damage. Again, the
symptoms are flu-like; you may also experience jaundice (yellowish eyes and skin)
after the flu-like symptoms go away. Most people discover they are infected by
having routine tests done or by donating blood or organs and the standard tests
show the infection. There is also a home test you can purchase and do if you
suspect you are infected.

If you are infected with a Hepatitis virus, or if you have been in the past, one of the
most important things that you can do is strengthen your liver. The easiest way to
do this is the Be Young Liver Cleanse: in the morning, take 1 drop of Be Young
Lemon essential oil, 1 drop of Be Young Peppermint essential oil, and 1 teaspoon
to 1 tablespoon of fresh lemon juice, followed by a glass of water. “Be Young essential
oils” are absolutely 100% pure, EOBBD tested and guaranteed to be free of synthetics and
extenders. Do not try this with an essential oil that you are not certain has been properly
cared for and tested as you do not want to increase challenges to your liver. When
properly supported, the liver has a remarkable capacity for regeneration.
Text 2: Questions 15-22

15. Hepatitis is caused by;


A Virus
B Alcohol consumption
C Medications
D All of the above

16. Which of the following spreads through feces of a contaminated person;


A Hepatitis A
B Hepatitis B
C Both
D Can`t say

17. Most adult bodies are able to fight off this virus;
A Hepatitis A virus
B Hepatitis B virus
C Both
D Can`t say

18. Hepatitis can lead to;


A Cancer
B Severe damage to the liver cells.
C Cancerous growth in the liver.
D Not given

19. In the USA people go for liver transplantation due to;


A Hepatitis A
B Hepatitis B
C Hepatitis C
D All of the above
20. Hepatitis C spreads through;
A Sharing needles
B Blood transfusion
C Organ transplantation
D All of the above

21. A patient may experience jaundice when;


A The flu-like symptoms appear.
B The flu-like symptoms disappear.
C Eyes become yellow.
D All of the above.

22. Be Young essential oils are;


A Free from extenders
B EOBBD
C Full of synthetics
D Not given

END OF READING TEST, THIS BOOKLET WILL BE COLLECTED


Reading test 11 : Answer Key

Part A - Answer key 1 – 7


1: C
2: B
3: D
4: A
5: B
6: C
7: A

Part A - Answer key 8 – 14


8: repeated open application test
9: Corticosteroids
10: repeated open application test
11: false positive result
12: cosmetic and sunscreen constituents
13: false negative result
14: below 5%

Part A - Answer key 15 – 20


Complete each of the sentences, 15-20, with a word or short phrase from one of
the texts. Each answer may include words, numbers or both.
15: Chronic irritant contact dermatitis
16: sensitizers
17: dryness, lichenification and fissuring
18: notable offenders
19: photo allergen series
20: 5 J cm2 UVA
Reading test - part B – answer key
1: B Blood clot doesn`t form. (if the reaction doesn`t occur the way it should)
2: A All types of Factor I deficiency affect both males and females.
3: C Hypotonia
4: A Every kind of procedure is used in the process of sterilization to keep sterile the
objects or articles that are to be introduced into a wound or body cavity or that is to
penetrate the skin;
5: B Boceprevir
6: C Survival rates of patients with genitourinary sarcomas.

Reading test - part C – answer key

Text 1 - Answer key 7 – 14


7: D All of the above
8: D Damaged but creates new cells with damaged DNA.
9: B Breast cancer
10: B Can grow and press on healthy organs and tissues.
11: C The transformation zone;
12: A two
13: D Squamous cell carcinomas
14: B With medications from doctors.

Text 2 - Answer key 15 – 22


15: D All of the above
16: A Hepatitis A
17: B Hepatitis B virus
18: A Cancer
19: C Hepatitis C
20: A Sharing Needles
21: B The flu-like symptoms disappear.
22: A Free from extenders
READING TEST 12
READING SUB-TEST : PART A
 Look at the four texts, A-D, in the separate Text Booklet.

 For each question, 1-20, look through the texts, A-D, to find the relevant information.

 Write your answers on the spaces provided in this Question Paper.


 Answer all the questions within the 15-minute time limit.
 Your answers should be correctly spelt.

TEXT BOOKLET - ATRIAL FIBRILLATION (AF)

Text A

Atrial fibrillation (AF)

Atrial fibrillation (AF) is the most common cardiac arrhythmia. It affects >33 million
individuals worldwide, and its prevalence is projected to double by 2050. AF is
associated with a 5- and 2-fold increased risk of stroke and mortality, respectively.
Furthermore, AF-related strokes are associated with higher morbidity, mortality, and
health care costs compared with non-cardioembolic strokes.
The mainstay of stroke prevention remains oral anticoagulation (OAC), with vitamin K
antagonists and more recently, direct oral anticoagulants (DOAC), reducing the risk of
ischemic stroke and all-cause mortality in patients with AF. However, more than onethird
of AF patients at high risk for stroke still fail to receive effective stroke prophylaxis
in contemporary practice. Although the introduction of DOAC has overcome some of the
limitations of warfarin therapy, persistent barriers including costs, ongoing bleeding risks
with no reversal agent for most DOACs, noncompliance and high discontinuation rates
may preclude a broader use of DOAC in clinical practice.
Text B
The left atrial appendage (LAA) is a remnant of the embryonic left atrium and is
considered the main reservoir for left atrial thrombi in >90% of patients with non valvular
AF. In recent years, percutaneous LAA closure (LAAC) has rapidly grown worldwide as
an appealing alternative for the prevention of thromboembolism in patients at high risk
for stroke, with a specific focus on patients ineligible for OAC. While no specific
recommendation on LAAC was given in the 2014 American guidelines, the 2016
European guidelines for the management of AF provided a class IIb recommendation for
percutaneous LAAC in patients with AF and contraindications for long-term OAC, based
on data from the PROTECT-AF and PREVAIL trials, the only LAAC randomized trials
to date. Although none of these studies included patients ineligible for OAC, most of the
real-world registries conducted to date have focused on this target population, which
currently represents the majority of LAAC recipients. This review provides an updated
overview of current transcatheter LAAC devices and reviews the main clinical data from
LAAC randomized trials and registries, focusing on procedural and late outcomes, as
well as on future directions.

Text C
WATCHMAN and WATCHMAN FLX
The WATCHMAN device (Boston Scientific, Natick, MA) was the second dedicated
LAAC device and remains the only device studied in randomized clinical trials to date. It
consists of a self-expanding nitinol 10-strut frame, with a 160 µm permeable
polyethylene terephthalate membrane fabric cap facing the left atrium. The open distal
end is fixed by 10 active fixation anchors in 1 row. The tool will be of different
dimensions 21, 24, 27, 30, and 33 mm. The transseptal access sheath will have crucial
specifications, and may show 14F with respect to width or breadth and is available in 3
different preformed curve shapes: anterior curve, double curve (used in >90% of
procedures), and single curve. Three proximal radio-opaque markers correspond to the
approximate level of deployment for 21, 27, and 33 device sizes, respectively.
Text D

The transseptal puncture is performed under fluoroscopic and preferably


transesophageal echocardiography (TEE; bicaval view followed by short-axis view)
guidance in the inferoposterior portion of the fossa ovalis. After transseptal puncture, a
long extra-stiff J tipped 0.035-inch wire is advanced into the left upper pulmonary vein
and the transseptal sheath is exchanged over the wire for the access sheath. After
removal of dilator and guidewire, a 5F to 6F pigtail catheter is advanced through the
access sheath into the left upper pulmonary vein. By using TEE and fluoroscopic
guidance, it will be easy to create some access. In the stage pertaining to it, access sheath
and pigtail are adjusted as required. Appropriate WATCHMAN device sizing is determined
by the maximum LAA ostium diameter (measured from the circumflex artery to 1-2 cm
within the pulmonary vein ridge at 0°, 45°, 90°, and 135°) and depth (from ostium to the
tip of LAA). An oversizing of the device by 10 to 20 percent (corresponding to 2±4 mm) is
generally recommended. A fluoroscopic right (20°±30°) and caudal (20°±30°) projection,
which usually opens the mid-distal portion of the LAA is the preferred one for the
deployment of the WATCHMAN device. After accurate LAA assessment, the delivery
system is advanced into the access sheath until the distal markers of the delivery
catheter and the access sheath align. The device is then deployed with a slow
unsheathing movement.
PART A -QUESTIONS AND ANSWER SHEET
Questions 1-7
For each question, 1-7, decide which text (A, B, C or D) the information comes from.
You may use any letter more than once. Your answers should be correctly spelt.

In which text can you find information about;


1. Used for accessing the left side of the heart. Answer _____________
2. Functioning of the device. Answer ______________
3. A comparison with another disease. Answer ______________
4. A drug that helps with blocking and managing the disease condition. Answer _____________
5. Structure of the device. Answer _____________
6. Made, done, or effected through the skin. Answer _____________
7. Details about LAA imaging and LAAC closure techniques are simply beyond the study performed.
Answer _______________

Questions 8-14
Answer each of the questions, 8-14, with a word or short phrase from one of the texts.
Each answer may include words, numbers or both.
8. What is generally recommended with respect to device sizing? Answer ___________

9. When will the catheter be introduced into the left upper pulmonary vein? Answer ____

10. In how many sizes, the WATCHMAN device is available? Answer ___________

11. What will be the outer diameter of the transseptal access sheath? Answer _______________

12. What is recommended for the effective use of the Watchman Device? Answer ____

13. What is taken into account to measure the size of the device? Answer _______________

14. What is the study more focused on? Answer ______________


Questions 15-20

Complete each of the sentences, 15-20, with a word or short phrase from one of the texts.
Each answer may include words, numbers or both. Your answers should be correctly spelt.

15. The best defending procedure is more connected to the use of the ___________.

16. All three ____________ are needed to be adjusted as per the levels of the device deployed.

17. With_________ access sheath is rightly repositioned into the LAA.

18. With DOAC being introduced, the scope for the __________ enhanced more.

19. The recommended oversizing of the device is in range of___________.

20. In its final stage, the device will be deployed with_________.

END OF PART A, THIS QUESTIONS PAPER WILL BE COLLECTED


READING SUB-TEST : PART B
In this part of the test, there are six short extracts relating to the work of health professionals .
For questions 1-6, choose the answer (A, B or C) which you think fits best according to the text.
Write your answers on the separate Answer Sheet
Questions 1-6

1. As per the given notice, one of the major duties include;


A. Care professional will have to look for ways to stop bleeding.
B. It is important to maintain hemodynamic stability.
C. Support cardiac function and circulation.

Nursing care of the heart transplant patient

Nursing care of the heart transplant patient is similar to the care of any cardiac surgery
patient. Bleeding is a major concern in the early postoperative period. Chest tube
drainage is frequently monitored (initially every 15 minutes), as are the cardiac output,
pulmonary artery pressures, and CVP. Cardiac tamponade can develop, presenting as
either a sudden event or a gradual process. Chest tubes are gently milked (not stripped)
as needed to maintain patency. Atrial dysrhythmias are relatively common following
cardiac transplant. Temporary pacing wires are placed during surgery because surgical
manipulation or postoperative swelling may disrupt the conduction system.
Hypothermia is induced during surgery; postoperatively, the patient is gradually re-
warmed over a 1- to 2-hour period. Cardiac function is impaired in up to 50% of
transplanted hearts during the early postoperative period. Inotropic agents such as low-
dose dopamine, dobutamine, or milrinone may be required to bring more stability.
2. The given notice explains;
A. Why medical professionals shall join the conference?
B. What medical professionals will learn?
C. How this is different from others.

Cardiology Conference
With people from around the world focused on getting some answers concerning
Cardiology, this is your single most obvious opportunity to accomplish the greatest
accumulation of individuals from the mending focuses, Universities, bunch, etc. This
Cardiology conference rather European Cardiology Congress in 2017 will coordinate
appears, disperse information, meet with recurring pattern and potential investigators
and get name affirmation at this 3-day event. Broadly acclaimed speakers, the most
recent frameworks, methodologies, and the most current updates in Cardiology field
are indications of this conference. This World Cardiology Congress will help in
frameworks organization, B2B uniting amidst specialists and academicians.

3. The major result of the changes mention is

A. inability to see / blindness


B. ineffective accommodation and depth perception
C. extra sensitivity to glare and dry eyes

With Age Comes Weakness


The eyelids become lax with the rotation of lid margins as result flow of tear may be
disrupted. The production of tear from the lachrymal glands may decline to make the
eyes dry and burning. There may be an accumulation of fluid in the cornea turning it
hazy. The muscles of iris are weak and the lens capsule becomes stiff making it difficult
to see small objects and depth perception accurately. The lens protein gets denatured and
solidifies. The anterior aspect of the uveal tract degenerates along with degeneration of
important areas in the retina.
4. The given notice talks about;
A. Complications involved in DVT.
B. Symptoms that lead to major problems.
C. Result of the disease conditions.

DVT
The problem occurs when a part of the clot breaks off and travels through the
bloodstream to the lungs, causing a blockage called pulmonary embolism (PE). If the clot
is small, and with appropriate treatment, people can recover from PE. However, there
could be some damage to the lungs. If the clot is large, it can stop blood from reaching
the lungs and is fatal. In addition, nearly one-third of people who have a DVT will have
long-term problems caused by the damage the clot does to the valves in the vein called
post-thrombotic syndrome (PTS). People with PTS have symptoms such as swelling,
pain, discoloration, and in severe cases, scaling or ulcers in the affected part of the
body. In some cases, the symptoms can be so severe that a person becomes disabled.

5. According to the information given, what is correct?

A. Only 40% of the household could have afforded the medications.


B. 33% of households in lower middle-income countries managed to have used the mediations.
C. 25% of households in upper middle-income countries got access to medications.
Medication Survey
In a survey of 598 communities in 18 countries, the availability of aspirin, β-blockers,
ACEIs, and statins varied widely In the low-income countries, only 1 of 30 rural and
only 25 of 32 urban communities had all 4 medications available. The 4 medications
were potentially unaffordable for 60% of households in low-income countries.
Availability rose to 37% (rural) and 62% (urban) in lower middle-income countries and
73% (rural) and 80% (urban) in upper middle-income countries. Effective strategies for
delivering medications are being developed for LMIC where health system infrastructure is
underdeveloped.
6. The given notice talks about;
A. Bidil dosage use.
B. Effectiveness of BiDil in treating heart failure.
C. how it helped the community.

BiDil - Support Blacks


BiDil for in African Americans BiDil, a fixed-dose combination of two vasodilators
(hydralazine and isosorbide), is indicated as an adjunctive treatment in African
Americans with heart failure. It has been shown to reduce symptoms, decrease the
number of hospitalizations, and prolong life in Blacks. The recommended dose is one to
two tablets three times per day, although the dose may be as low as 1/2 tablet three times a
day if side effects are intolerable. The approval of this combination drug has raised the
ethical issue of race-specific FDA approval.
READING SUB-TEST : PART C
In this part of the test, there are two texts about different aspects of healthcare.
For questions 7-22, choose the answer (A, B, C or D) which you think fits best according to the text.
Write your answers on the separate Answer Sheet

Text 1: What is Zika?

The Zika virus disease is caused by the Zika virus, which is spread to people primarily
through the bite of an infected mosquito (aedes aegypti and aedes albopictus). The illness
is usually mild with symptoms lasting up to a week, and many people do not have
symptoms or will have only mild symptoms. However, a Zika virus infection during
pregnancy can cause a serious birth defect called microcephaly and other severe brain defects.

Zika is spread to people primarily through the bite of an infected aedes species mosquito
(aedes aegypti and aedes albopictus). A pregnant woman can pass Zika to her fetus
during pregnancy or around the time of birth. Also, a person with Zika can pass it to his
or her sexual partners. We encourage people, who have traveled to or live in places with
Zika to protect themselves by preventing mosquito bites and sexual transmission of Zika.

Many people infected with Zika will have no symptoms or mild symptoms that last
several days to a week. However, a Zika infection during pregnancy can cause a serious
birth defect called microcephaly and other severe fetal brain defects. Current research
suggests that Guillain-Barre syndrome (GBS), an uncommon sickness of the nervous
system, is strongly associated with Zika; however, only a small proportion of people with a
recent Zika virus infection get GBS. Once someone has been infected with Zika, it`s very
likely they`ll be protected from future infections. There is no evidence that past Zika
infection poses an increased risk of birth defects in future pregnancies.

Going to places where Zika is common is often not recommended. Travellers who go to
places with outbreaks of Zika may or may not get infected with Zika. Moreover, in
pregnant women, if the virus is caught from such places, it can cause microcephaly and
other severe fetal brain defects.
Any pregnant women who have recently travelled to an area with Zika should talk to
their doctor about their travel, even if they don`t feel sick. Pregnant women should see a
doctor if they have any Zika symptoms during their trip or within 2 weeks after
travelling. All pregnant women can protect themselves by using plenty of prevention
measures, which include but are not limited to: avoiding travel to an area with Zika;
preventing mosquito bites; and following recommended precautions against getting Zika.
The most common symptoms of the Zika virus disease are fever, rashes, joint pain, and
red eyes. Other symptoms include muscle pain and headaches. Many people infected with
Zika won`t have symptoms or will have mild symptoms, which can last for several days
to a week.

Currently, there is no evidence that a woman who has recovered from the Zika virus
infection (the virus has cleared her body) will have Zika-related pregnancy complications
in the future. Based on information about similar infections, once a person has been
infected with the Zika virus and has cleared the virus, he or she is likely to be protected
from future Zika infections. If you`re thinking about having a baby in the near future and
you or your partner live in or travelled to an area with Zika, talk with your doctor or
another healthcare provider. Men who have travelled to any areas with Zika or who have
had a Zika infection should wait at least 6 months after travel (or 6 months after
symptoms started if they get sick) before trying to conceive with their partner. Women
should wait at least 8 weeks after travel (or 8 weeks after symptoms started if they get
sick) before trying to get pregnant.
Text 1: Questions 7-14

7. According to paragraph 1, the Zika virus disease is;


A. Common
B. Uncommon
C. Severe disease
D. Without any symptoms

8. What does paragraph 2 indicate?


A. How does Zika spread from person to person
B. Modes of transmission of Zika
C. How do people get infected with Zika?
D. A & C

9. What is mentioned in paragraph 3?


A. How do people get infected with Zika?
B. What health problems can result from getting Zika?
C. How microcephaly occurs.
D. Microcephaly and birth defects.

10. According to paragraph 5, what is recommended for pregnant women?


A. They should not travel to places where the Zika virus is common.
B. They should talk to their partner before going to Zika-infected places.
C. They should inform their doctor before travelling.
D. Not given

11. In paragraph 5, how many methods of prevention from Zika are described?
A. Plenty of prevention measures
B. 2
C. 3
D. 4
12 Can a person who is completely recovered from Zika virus infection, get Zika infection again?
A. Yes
B. No
C. Depends from person to person
D. Not given

13. According to paragraph 6, which one of the following statements is true?


A. A woman`s future pregnancies will be at risk if she is infected with the Zika virus.
B. There is no evidence that infected women will not face any complications in future pregnancies.
C. Any woman who is infected with Zika is advised to wait for six months before getting pregnant.
D. B & C

14. According to paragraph 6, which of the following is not true?


A. A woman should wait for six months before they get pregnant if they have traveled to any places
with the Zika virus.
B. If the woman has been infected with the Zika virus and has cleared the virus, she is likely to be
protected from future Zika infections.
C. In most of the cases, Zika complications may occur in future pregnancies.
D. All above statements are true.
Text 2: Avian Influenza - A Virus Infections in Humans

Although avian influenza A viruses usually do not infect humans, rare cases of human
infection with these viruses have been reported. Infected birds shed the avian influenza
virus in their saliva, mucus and feces. Human infections with bird flu viruses can happen
when enough of the virus gets into a person`s eyes, nose or mouth, or is inhaled. This can
happen when the virus is in the air (in droplets or possibly dust) and a person breathes it
in, or when a person touches something that has the virus on it and then touches their
mouth, eyes or nose. Rare human infections with some avian viruses have occurred most
often after unprotected contact with infected birds or surfaces contaminated with avian
influenza viruses. However, some infections have been identified where direct contact
was not known to have occurred.

The reported signs and symptoms of low pathogenic avian influenza (LPAI) A virus
infections in humans have ranged from conjunctivitis to influenza-like illness (e.g., fever,
cough, sore throat, muscle aches) to lower respiratory disease (pneumonia) requiring
hospitalization. Highly pathogenic avian influenza (HPAI) LPAI H7N9 and HPAI Asian
H5N1 have been responsible for most human illness worldwide to date, including the
most serious illnesses and deaths. A virus infections in people have been associated with
a wide range of illness from conjunctivitis only to severe respiratory illness (e.g.
shortness of breath, difficulty breathing, pneumonia, acute respiratory distress, viral
pneumonia, respiratory failure) with multi-organ disease, sometimes accompanied by
nausea, abdominal pain, diarrhea, vomiting and sometimes neurologic changes (altered
mental status, seizures).

Detection of the avian influenza A virus infection in humans is a formidable process. It


may be difficult to assess the presence of the virus by clinical signs and symptoms alone;
laboratory testing is required. The avian influenza A virus infection is usually diagnosed
by collecting a swab from the nose or throat of the sick person during the first few days
of illness. This specimen is sent to a lab; the laboratory looks for the avian influenza A
virus either by using a molecular test, by trying to grow the virus, or both. (growing avian
influenza A viruses should only be done in laboratories with high levels of protection).
For critically ill patients, collection and testing of lower respiratory tract specimens may
lead to a diagnosis of avian influenza virus infection. For some patients who are no
longer very sick or who have fully recovered, it may be difficult to find the avian
influenza A virus in the specimen, using these methods. It is often termed a problem of
gargantuan proportion. Sometimes it may still be possible to diagnose the avian influenza
A virus infection by looking for evidence of the body`s immune response to the virus
infection by detecting specific antibodies the body has produced in response to the virus.
This is not always an option because it requires two blood specimens (one taken during the
first week of the illness and another taken 3-4 weeks later).

Analyses of available avian influenza viruses circulating worldwide suggest that most
viruses are susceptible to oseltamivir, peramivir, and zanamivir. However, some evidence
of antiviral resistance has been reported in HPAI Asian H5N1 viruses and influenza A
H7N9 viruses isolated from some human cases. Monitoring for antiviral resistance among
avian influenza A viruses is crucial and ongoing. This data directly informs WHO
antiviral treatment recommendations.
The best way to prevent an infection of the avian influenza A virus is to avoid sources of
exposure. Most human infections with avian influenza A viruses have occurred following
direct or close contact with infected poultry. People who have had contact with infected
birds may be given expert-formulated influenza antiviral drugs preventatively. While
antiviral drugs are most often used to treat flu, they can also be used to prevent infection
in someone who has been exposed to influenza viruses. When used to prevent seasonal
influenza, antiviral drugs are 70% to 90% effective. A seasonal influenza vaccination will
not prevent infection with avian influenza A viruses, but can play hardball in effectively
reducing the risk of co-infection with human and avian influenza A viruses. It`s also
possible to make a vaccine intellectually that can protect people against avian influenza
viruses. For example, the United States government always have access to the stockpiled
vaccine to protect against avian influenza A H5N1 vaccine. The stockpiled vaccine could
be used if a similar H5N1 virus were to begin transmitting easily from person to person.
Creating a candidate vaccine virus is the first step in producing a vaccine.
Text 2: Questions 15-22

15. According to paragraph 1, the virus enters the human body through.
A. The nose
B. The eyes
C. The mouth
D. Contact with an infected bird

16. According to paragraph 1, human infection with avian influenza A virus is;
A. Uncommon
B. Common
C. Rare
D. Frequent in infected places

17. According to paragraph 2, people may suffer from flu if they have been infected by
A. LPAI
B. HPAI
C. H5N1
D. LPAI H7N9

18. According to paragraph 3, what is not true?


A. A molecular test is performed to see the presence of the virus in the collected specimen.
B. Lab tests are aimed at growing the virus for the right detection purpose.
C. A specimen for testing is usually collected right after the person gets the illness by infection.
D. People who have recovered from the illness should also be diagnosed for further
confirmation.

19. According to paragraph 3, the word “gargantuan― may mean;


A. Small-scale
B. Trivial
C. Very large
D. Pint-sized
20. Which central idea is presented in paragraph 4?
A. Treatment of the avian influenza disease.
B. Effective medications for the viral disease.
C. Research on effective medicine for the avian virus disease.
D. None of the above.

21. According to paragraph 5, which word may give the meaning of experience?
A. Exposure
B. Expert
C. Intellectualism
D. None of the above

22. The phrase to play hardball in paragraph 5 may mean;


A. To be able to achieve the results.
B. To do what it takes to complete the task.
C. To work effectively.
D. To be firm and determined in order to get what you want.

END OF READING TEST, THIS BOOKLET WILL BE COLLECTED


Reading test 12 : Answer Key

Part A - Answer key 1 – 7


1: D
2: D
3: A
4: A
5: C
6: B
7: C

Part A - Answer key 8 – 14


8: Oversizing by 10 to 20 percent
9: After removal of dilator and guidewire
10: Five
11: 14 F
12: Fluoroscopic right and caudal projection
13: Maximum diameter / LAA ostium diameter
14: LAAC recipients

Part A - Answer key 15 – 20


15: oral anticoagulation (OAC)
16: radio-opaque markers
17: fluoroscopic guidance
18: warfarin therapy
19: 10% to 20%
20: unsheathing movement
Reading test - part B – answer key
1. C Support cardiac function and circulation.
2. A Why medical professionals shall join the conference?
3. C extra sensitivity to glare and dry eyes
4. A Complications involved in DVT.
5. A Only 40% of the household could have afforded the medications.
6. B Effectiveness of BiDil in treating heart failur

Reading test - part C – answer key

Text 1 - Answer key 7 – 14


7: C Severe disease
8: C How do people get infected with Zika?
9: B What health problems can result from getting Zika?
10: D Not given
11: C 3
12: B. No
13: A A woman`s future pregnancies will be at risk if she is infected with the
Zika virus.
14: C In most of the cases, Zika complications may occur in future pregnancies.

Text 2 - Answer key 15 – 22


15: B The eyes
16: A Uncommon
17: B HPAI
18: D People who have recovered from the illness should also be diagnosed
for further confirmation.
19: C Very large
20: A Treatment of the avian influenza disease.
21: A Exposure
22: D To be firm and determined in order to get what you want.
READING TEST 13

READING SUB-TEST : PART A


 Look at the four texts, A-D, in the separate Text Booklet.

 For each question, 1-20, look through the texts, A-D, to find the relevant information.

 Write your answers on the spaces provided in this Question Paper.


 Answer all the questions within the 15-minute time limit.
 Your answers should be correctly spelt.

TEXT BOOKLET- POLYCYTHEMIA

Text A
The mechanism of polycythemia in primary familial and congenital polycythemia (PFCP) is due to
the truncated EpoR (genetic mutation) in which there is no inhibition of signalling pathways. In all
conditions of hypoxia HIF-1 is responsible for the polycythemia. Some patients with chronic lung
disease or congenital cyanotic heart disease do not develop polycythemia in spite of hypoxia, the
mechanism of which is not very clear. Polycythemia in smokers is due to increased blood carbon
monoxide (CO). CO displaces one molecule of O2 from hemoglobin and converts it to
carboxyhemoglobin (COHb). COHb has 200 times greater affinity than oxygen. This results in not
only occupation of one of the heme groups of haemoglobin but also increase in the oxygen affinity
of the remaining heme group resulting in tissue hypoxia. Polycythemia accompanying kidney and
liver diseases and neoplastic disorders, is usually associated with increased EPO production. In
tumours EPO production is shown to be autonomous of hypoxic stimulation .production is shown
to be autonomous of hypoxic stimuli.
Text B

The molecular basis of post-transplantation erythrocytosis (PTE) remains unclear. It is found in 5-


10% of renal allograft recipients developing within 8-24 months following a successful renal
transplantation. It resolves spontaneously within 2 years in about 25% of patients. In congenital
secondary polycythemia, mutations in the haemoglobin can lead to increased oxygen affinity
leading to decreased oxygen delivery and compensatory polycythemia. A rare mechanism in this
group is 2, 3 BPG deficiency. This compound is synthesised in red blood cell and binds to
haemoglobin reducing its affinity for oxygen. Its absence leads to increased affinity of haemoglobin
for oxygen resulting in a lifelong hypoxic stimulus and erythrocytosis. The fetal haemoglobin has
high oxygen affinity and many of the neonates may have markedly elevated hematocrits.
Polycythemia vera rises from the transformation of a single hematopoietic stem cell with a
selective growth advantage that gradually becomes the predominant myeloid progenitor. Recently
a somatic mutation is detected in a gene on chromosome 9p in a majority of
polycythemia vera patients. This gene encodes for tyrosine kinase JAK. This somatic mutation
transforms this kinase into a constitutively active form and seems to be responsible for the
uncontrolled proliferation of the erythroid cells.
Text C

Clinical Approach
Symptoms of polycythemia are very nonspecific like a headache, weakness, pruritus, dizziness,
sweating and visual disturbances. Some of the patients are seen initially with complications of
polycythemia like thrombosis (cerebral,peripheral) and haemorrhage. Thrombosis may occur at
unusual sites like hepatic vein. Polycythemia may be diagnosed when Budd Chiari syndrome is
being investigated. Hematocrit values above 51% in males and over 48% in females requires further
evaluation.
Diagnostic criteria laid down by PVSG and WHO require demonstration of an elevated red cell mass
as a must. This is practically not possible in most centres. So, WHO has revised the criteria (2008)
for the diagnosis of PV6 .
Accordingly, there are 2 major and 3 minor criteria.
Major criteria
1. Hemoglobin level above 18.5g/dl for men and 16.5g/dl for females OR Hemoglobin or
hematocrit > 99th percentile of reference range for age, sex, or altitude of residence OR elevated
red cell mass >25% above mean normal predicted value.
2. Presence of JAK2 gene mutation (V617F) or other functionally similar.
Minor criteria
1. Bone marrow showing hypercellularity for age and trilineage growth (panmyelosis)
2. Subnormal Epo level
3. EEC (endogenous erythroid colonies)
Diagnostic combinations - Major criteria + one minor criterion and first major criterion + 2 minor
criteria
Text D
Recommendations;
Low dose aspirin 75- 150 mg is recommended in all PV patients without history of major bleeding
or gastric intolerance, based on the results of the ECLAP study.
Patients with PV should be properly hydrated when they develop gastrointestinal disorders. The
spent phase occurs after about 15-20 years, when the phlebotomy requirement decreases and the
patient develops anaemia. The marrow fibrosis increases and spleen becomes greatly enlarged.
The treatment during this phase is purely symptomatic including blood transfusions. Other
treatment modalities tried are splenectomy, thalidomide and marrow transplantation in younger
patients. In the future we may have new JAK2 targeted inhibitors to treat PV. Some patients may
get transformed into acute leukaemia Any form of treatment during this phase is not at all
satisfactory. Currently, management of PV depends on the risk stratification
Age >60yrs or history Cardiovascular risk
Risk category
of thrombosis factors
Low No No
Intermediate No Yes
High Yes
Phlebotomy is the cornerstone of low-risk patients aimed at reaching and maintaining a target
hematocrit of 45% in males and 42% in females. Low dose aspirin may be added to the treatment.
High-risk patients should receive myelosuppressive treatment in addition to phlebotomy. The drug
of choice is hydroxyurea.PV may infrequently occur during childbearing years. There is an
increased incidence of abortion in about 30% of cases. Pre-eclampsia is also common. It is very
interesting that some of the women may even reduce their hematocrit. Their phlebotomy
requirement is also found to be decreased. The possible explanations are the erythropoietic
suppressive effect of the high estrogen levels, expansion of the plasma volume and nutritional
deficiencies. If needed, the patient should be treated with phlebotomy, low dose aspirin or
interferon. After delivery the blood count will drift back to the original polycythemic level.
PART A -QUESTIONS AND ANSWER SHEET
Questions 1-7
For each of the questions, 1-7, decide which text (A, B, C or D) the information comes from.
You may use any letter more than once.

In which text can you find information about;


1. Severe itching of the skin, as a symptom of various ailments. Answer __________
2. Maintaining the ratio of the volume of red blood cells to the total volume of blood around five
percent and 8 percent lesser than 50% in males and females. Answer _________________
3 .Take place during pregnancy. Answer _____________
4 .A condition arising due to shortening of the genes. Answer______________
5 .The presence of an abnormal excess of cells. Answer _________________
6 .Bluish cast to the skin and mucous membrane. Answer ________________
7. An enzyme that can transfer a phosphate group from ATP to a protein in a cell. Answer ______

Questions 8-14
Answer each of the questions, 8-14, with a word or short phrase from one of the texts. Each answer
may include words, numbers or both. Your answers should be correctly spelt.

8 .What is the term which refers to the use of the drug in the treatment of certain cancers?__
9 .What is found in a gene on chromosome 9p in patients? Answer ___________________
10 .When does a patient develop anaemia? Answer ______________________
11. What is the condition in which bone marrow activity is decreased, resulting in fewer red blood
cells, white blood cells, and platelets? Answer __________________
12. What leads to the formation by peripheral blood mononuclear cells from patients with
polycythemia vera? Answer _____________
13 .What is the hormone produced by the kidney that promotes the formation of red blood cells by the
bone marrow? Answer ______________
14. What is the recommended for treatment? Answer __________________________
Questions 15-20

Complete each of the sentences, 15-20, with a word or short phrase from one of the texts. Each
answer may include words, numbers or both. Your answers should be correctly spelt.

15. _________ is used with other medications or radiation therapy to treat some blood disorders.

16 . ____________mutations were found to have a high correlation with abnormal heart defects.

17 .________ cells can undergo rapid proliferation before differentiating into maturation stages.

18._____________________ are seen in the great majority of cases of polycythaemia.

19 .Polycythemia vera shows stable growth, majorly come into being from a single _____stem cell.

20. In various cancerous conditions, production of______ is recorded to be independent of hypoxic

stimuli.

END OF PART A, THIS QUESTIONS PAPER WILL BE COLLECTED


READING SUB-TEST : PART B
In this part of the test, there are six short extracts relating to the work of health professionals .
For questions 1-6, choose the answer (A, B or C) which you think fits best according to the text. Write
your answers on the separate Answer Sheet
Questions 1-6

1. As per the following notice, what is correct?


A. Dyslipidemia in children is common.
B. Young committee mostly gets affected by the disease.
C. Statistics showing the curtailment ratio of the affected people.

The American Academy of Pediatrics recommends screening for dyslipidemia in children and
adolescents who have a family history of dyslipidemia or premature CVD, those whose family
history is unknown, and those youths with risk factors for CVD, such as being overweight or obese,
having hypertension or DM, or being a smoker 1 In 2011, the NHBLI Expert Panel recommended
universal dyslipidemia screening for all children between 9 and 11 years of age and again
between 17 and 21 years of age 23.Analysis of data from NHANES 1999 to 2006 showed that the
overall prevalence of abnormal lipid levels among youths 12 to 19 years of age was 20 3%. From
2005 to 2010, among adults with high LDL- C, age adjusted control of LDL-C increased from 22 3%
to 29 5% 25 The prevalence of LDL-C control was lowest among people who reported receiving
medical care less than twice in the previous year (11 7%), being uninsured (13 5%), being Mexican
American (20 3%), or having income below the poverty level (21 9%) 2.
2. Notice on debatable concepts gives information about;
A. Concepts which may have direct impact on splenectomy.
B. Conditions which might arise after splenectomy.
C. Situations requiring great effort.

DEBATABLE Concepts of Laparoscopic splenectomy


 Malignant hematologic diseases

 Huge splenomegaly (> 25 cm)

 Malignant splenic tumor

 Pericapsular inflammation

 Large lymph nodes at the splenic hilum

 PHT and cirrhosis

 Difficulties
• Technical challenge
• Splenic mobilization
• Safe access to the splenic hilum
3.The manual gives information about
A. Rescue Protocol
B. Emergency Care Plan
C. Adjustable Properties of Power heart AED G3

The AED protocol is consistent with the guidelines recommended by the American
Heart Association (Guidelines 2005 for Cardiopulmonary Resuscitation and Emergency Cardiac Care
American Heart Association; Circulation vol 112, Issue 24 Suppl. Dec. 13, 2005) and the
International Liaison Committee on Resuscitation (ILCOR)). Upon detecting a shockable cardiac
rhythm, the AED advises the operator to press the SHOCK button (9390E only) to deliver a
defibrillation shock followed by performing 2 minutes of CPR. For the Powerheart AED G3
Automatic, upon detecting a shockable rhythm, the AED will automatically deliver defibrillation
shocks followed by performing 2 minutes of CPR.

Note: In alignment with the 2005 Guidelines, the default setting for the CPR time has been set to
allow for 5 cycles of 30 compressions and 2 breaths.
Increasing or decreasing the CPR time setting may increase or decrease the number of actual cycles
allowed during the CPR timeout period.
4. As per the given notice, GM levels of triglycerides is;
A. Higher among people who are 20+ years.
B. Common among men.
C. Common among women.
The geometric mean level of triglycerides for American adults ≥20 years of age was 103 5 mg/ dL in
NHANES 2011 to 2014. Approximately 24 2% of adults had high triglyceride levels (≥150 mg/dL) in
NHANES 2011 to 2014. Among males, the age adjusted geometric mean triglyceride level was 111 6
mg/dL. in NHANES 2011 to 2014, with the following racial/ethnic breakdown: — 113 2 mg/dL for
non-Hispanic white males — 86 7 mg/dL for non-Hispanic black males — 124 1 mg/dL for Hispanic
males — 115 3 mg/dL for non-Hispanic Asian males

5. Pick the one that is incorrect


A. GGTP (cut-off): GGTP <100; 100 % ¤ GGTP % ¤ 200; GGTP >200; for scores 1, 2, 3 respectively.
B. Bilirubin (cut-off): Bil <1.5; 1.5% ¤ Bil % ¤ 2.5; Bil >2.5; for scores 1, 2,3 respectively.
C. Albumin (cut-off): Alb >3.5; 2.5 % ¤ Alb< 3.5; Alb < 2.5; for scores 1, 2, 3 respectively.
Relationship of a liver index and its parameters to HCC aggressiveness
β Se(β) p. 95% C.I
(A)
Liver Index score 0.2462 0.0247 <0.001 0.1978 to 0.2945
(B)
GGTP (IU/ml) 0.0013 0.0003 <0.001 0.0007 to 0.0020
Total Bilirubin (mg/dl) 0.0585 0.0140 <0.001 0.0311 to 0.0859
Albumin (g/dl) -0.3821 0.0554 <0.001 -0.4908 to - 0.2733
Platelets (x109/L) 0.0031 0.0005 <0.001 0.0021 to 0.0041
6. The given notice talks about
A. Functioning of the ultrasound.
B. Detection of fetus.
C. Examination of the fetus.

Ultrasound is done during pregnancy to track the development of the fetus in the mother’s womb.
It is not only helpful in tracking down the development but also helps to find out any fetal
anomalies. Ultrasound reveals the heartbeat of the fetus, the radius of the head, the length of the
hands and feet and also his/her height and weight. There are various kinds of ultrasound which can
be done during pregnancy namely Transvaginal Ultrasound, 3-D Ultrasound, 4-D Ultrasound and
Fetal Echocardiography. While the Sonography reports in the first trimester provides
information about the fetal heartbeat, it also examines the placenta, uterus, ovaries, cervix, checks
for multiple pregnancies, the sonography done in the second and third trimester reveals much
important criteria like placental abruption, placenta previa, characteristics of Down’s syndrome if
there are any possibilities. The ultrasound in this stage also determines whether the fetus is
carrying any form of congenital disease whether hereditary or non-hereditary or not.
READING SUB-TEST : PART C ]

In this part of the test, there are two texts about different aspects of healthcare.
For questions 7-22, choose the answer (A, B, C or D) which you think fits best according to the text.
Write your answers on the separate Answer Sheet

Text 1: Anaplasmosis
Anaplasmosis is a tick-borne disease caused by the bacterium anaplasmaphagocytophilum. It was
previously known as human granulocytic ehrlichiosis (HGE) and has more recently been called
human granulocytic anaplasmosis (HGA). Anaplasmosis is transmitted to humans by tick bites,
primarily from the black-legged tick (Ixodesscapularis) and the western blacklegged tick
(Ixodespacificus). Of the four distinct phases in the tick life-cycle (egg,larva, nymph, adult), nymphal
and adult ticks are most frequently associated with transmission of anaplasmosis to humans. Typical
symptoms include fever, headache, chills, and muscle aches. Usually, these symptoms which occur
within 1-2 weeks of a tick bite can’t be known and in many cases can’t even be averted.
Anaplasmosis, which often can’t be thwarted, is initially diagnosed based on
symptoms and clinical presentation, and later confirmed by the use of specialized laboratory tests.
The first line treatment for adults and children of all ages is doxycycline. Anaplasmosis and other
tick-borne diseases can be obviated.

Anaplasmosis is caused by the bacterium anaplasmaphagocytophilum. This organism used to be


known by other names, including Ehrlichiaequi and Ehrlichiaphagocytophilum, and the disease
caused by this organism has been previously described as human granulocytic ehrlichiosis (HGE).
However, a taxonomic change in 2001 identified that this organism belonged to the genus
anaplasma, and resulted in a change in the name of the disease to anaplasmosis. Anaplasmosis was
first recognized as a disease of humans in the United States in the mid-1990’s, but did not become a
reportable disease until 1999. The number of anaplasmosis cases reported has increased steadily
since the disease became reportable, from 348 cases in 2000, to 1761 cases in 2010. The incidence
(the number of cases for every million persons) of anaplasmosis has also increased, from 1.4 cases
per million persons in 2000 to 6.1 cases per million persons in

2010. The case fatality rate (i.e. the proportion of anaplasmosis patients that reportedly died as a
result of infection) has remained low, at less than 1%.

The bacterium anaplasmaphagocytophilum is transmitted to humans by the bite of an infected tick.


The black-legged tick (Ixodesscapularis) is the vector of A. phagocytophilum in the northeast and
upper mid western United States. The western black-legged tick (Ixodespacificus) is the primary
vector in Northern California. The first symptoms of anaplasmosis typically begin within 1-2 weeks
after the bite of an infected tick. A tick bite is usually painless, and some patients who develop
anaplasmosis do not remember being bitten. It can be a serious illness that can be fatal if not
treated correctly, even in previously healthy people.

The severity of anaplasmosis may depend in part on the immune status of the patient. Persons with
compromised immunity caused by immunosuppressive therapies (e.g., corticosteroids, cancer
chemotherapy, or long-term immunosuppressive therapy following an organ transplant), HIV
infection, or splenectomy appear to develop a more severe disease, and case-fatality rates for
these individuals are characteristically higher than case-fatality rates reported for the general
population.

Because A. phagocytophilum infects the white blood cells and circulates in the bloodstream, this
pathogen may pose a risk to be transmitted through blood transfusions.
Anaplasmaphagocytophilum has been shown to survive for more than a week in refrigerated blood.
Several cases of anaplasmosis have been reported associated with the transfusion of packed red
blood cells donated from asymptomatic or acutely infected donors. Patients who develop
anaplasmosis within a month of receiving a blood transfusion or solid organ transplant should be
reported to state health officials for prompt investigation.

There are several aspects of anaplasmosis that make it challenging for healthcare providers to
diagnose and treat. The symptoms vary from patient to patient and can be difficult to distinguish
from other diseases. Treatment is more likely to be effective if started early in the course of the
disease. Diagnostic tests based on the detection of antibodies will frequently appear negative in the
first 7-10 days of illness. For this reason, healthcare providers must use their judgment to treat
patients based on clinical suspicion alone. Healthcare providers may find important information in
the patient’s history and physical examination that may aid clinical diagnosis. Information such as

recent tick bites, exposure to areas where ticks are likely to be found, or history of recent travel to
areas where anaplasmosis is endemic can be helpful in making the diagnosis.
Part C -Text 1: Questions 7-14
7. According to paragraph 1, what is not anaplasmosis?
A. A bacterial disease
B. A disease that is transmitted by tick bites
C. A disease in which people suffer from muscle pain
D. A disease that can™t be prevented

8. Which word in paragraph 1 may mean removing difficulty?


A. Averted
B. Thwarted
C. Obviated
D. None of the above.

9. Paragraph 2 deals more with the;


A .Annual cases of anaplasmosis in the US.
B. cause of the anaplasmosis.
C. prevlance of the anaplasmosis.
D. None of the above.

10. What is not true about anaplasmosis, according to paragraph 2?


A. The old name of anaplasmosis was HGE.
B. The causal agent of anaplasmosis was recorded to be ehrlichiaequi
C. HGE was renamed as anaplasmosis in the year 2001
D. Cases of anaplasmosis became known only after 2000
11. What is not true about ticks, according to paragraph 3?
A. A tick bite is painless
B. A tick bite carries bacterium anaplasmaphagocytophilum
C. Ticks are present throughout the US
D. Black-legged ticks are present across California

12. What is the central idea of paragraph 4?


A. Immune-compromised individuals
B. The effects of anaplasmosis
C. Fatality rate and anaplasmosis
D. None of the above

13. What do we find in paragraph 5?


A. Blood transfusion risks
B. Organ transplant risks
C. Blood transfusion and organ transplant risks associated with anaplasma species
D. Information about anaplasmaphagocytophilum

14. According to paragraph 6, what is not true about anaplasmosis?


A. It is difficult to diagnose and treat anaplasmosis
B. Why tests for anaplasmosis appear negative
C. Different patients may show different symptoms
D. A patients medical history is often taken into consideration
Text 2: Candidiasis
Candidiasis is a fungal infection caused by yeasts that belong to the genus Candida. There are over
20 species of Candida yeasts that can cause infection in humans, the most common of which is
Candida albicans. Candida yeasts normally reside in the intestinal tract and can be found on
mucous membranes and skin without causing infection; however, overgrowth of these organisms
can cause symptoms to develop. Symptoms of candidiasis vary depending on the area of the body
that is infected.

Candidiasis that develops in the mouth or throat is called “thrush” or oropharyngeal candidiasis.
Candidiasis in the vagina is commonly referred to as a “yeast infection.” Invasive candidiasis occurs
when Candida species enter the bloodstream and spread throughout the body. The infection is not
very common in the general population. It is estimated that between 5% and 7% of babies less than
one month old will develop oral candidiasis. The prevalence of oral candidiasis
among AIDS patients, (particularly women rather than men, although not yet an established fact) is
estimated to be between 9% and 31%, and studies have documented clinical evidence of oral
candidiasis in nearly 20% of cancer patients.
Candidiasis of the mouth and throat, also known as “thrush" or oropharyngeal candidiasis, is a
fungal infection that occurs when there is an overgrowth of a yeast called Candida. Candida yeasts
normally live on the skin or mucous membranes in small amounts. However, if the environment
inside the mouth or throat becomes imbalanced, the yeasts can multiply and cause symptoms.
Candida overgrowth can also develop in the oesophagus, and this is called Candida
esophagitis, or esophageal candidiasis.

Candida infections of the mouth and throat can manifest in a variety of ways. The most common
symptom of oral thrush is white patches or plaques on the tongue and other oral mucous
membranes. Other symptoms include redness or soreness in the affected areas; difficulty
swallowing; cracking at the corners of the mouth (angular cheilitis) etc.
Candida infections of the mouth and throat are infrequent among adults who are otherwise
healthy. Oral thrush presents itself most recurrently among babies less than one month old, the
elderly, and groups of people with weakened immune systems. Other factors associated with oral
and esophageal candidiasis include HIV/AIDS, cancer treatments, organ transplantation, diabetes
etc. Good oral hygiene practices may sporadically help to prevent oral thrush in people with
weakened immune systems. Some studies have shown that chlorhexidine (CHX) mouthwash can
help to prevent oral candidiasis in people undergoing cancer treatment. People who use inhaled
corticosteroids may be able to reduce the risk of developing thrush by washing out the mouth with
water or mouthwash after using an inhaler.

Candida infections of the mouth and throat must be treated with prescription antifungal
medication. The type and duration of treatment depends on the severity of the infection and
patient-specific factors such as age and immune status. Untreated infections can lead to a more
serious form of invasive candidiasis. Oral candidiasis usually responds to topical treatments such as
clotrimazole troches and nystatin suspension (nystatin “swish and swallow”). Systemic antifungal
medication such as fluconazole or itraconazole may be necessary for oropharyngeal infections that
do not respond to these treatments. Candida esophagitis is typically treated with oral or
intravenous fluconazole or oral itraconazole. For severe or azole-resistant esophageal candidiasis,
treatment with amphotericin B may be necessary. For healthcare providers: the most up-to-date
clinical practice guidelines for the treatment of oropharyngeal / esophageal candidiasis are
available at the Infectious Diseases Society of America.
Part C -Text 2: Questions 15-22
15. According to paragraph 1, the abode for candida yeasts is;
A. Intestinal tract
B. Mucous membrane
C. Skin
D. All of the above

16. According to paragraph 2, oral candidiasis is;


A. Prevalent among children.
B Prevalent among babies.
C. Common among people suffering from AIDS.
D. Common among female patients with AIDS.

17. According to paragraph 2, candida yeasts;


A. grow on the mucous membrane.
B. grow under the skin of the mouth.
C. grow only inside the throat area.
D. grow expeditiously in the esophagus.

18. The word manifestation in paragraph 3 may mean;


A. To describe
B. To multiply
C. To show
D. To disguise
19. What does paragraph 4 indicate?
A. Who gets affected with oral candidiasis?
B. How to prevent oral candidiasis.
C. Risk and prevention.
D. None of the above.

20. According to paragraph 4, candida infections;


A. Occur among people affected with AIDS, diabetes etc.
B. Occur less common among people with a low immunity rate.
C. Can be prevented using off-the-shelf medical products such as CHX mouthwash.

21. The word sporadic in paragraph 4 means;


A. Always
B. Not regular
C. Intermittently
D. Every so often

22. What is the central idea presented in paragraph 5?


A. The treatment process of candidiasis.
B. Treatment and outcome.
C. Medication for candidiasis.
D None of the above.

END OF READING TEST, THIS BOOKLET WILL BE COLLECTED


Reading test 13 : Answer Key
Part A - Answer key 1 – 7
1 .C
2.D
3.D
4 .A
5 .C
6 .A
7 .B

Part A - Answer key 8 – 14


8 .thalidomide
9 .somatic mutation
10 .phlebotomy requirement decreases
11. Myelosuppression
12 .Endogenous erythroid colony
13 .Erythropoietin (EPO)14 .phlebotomy

Part A - Answer key 15 – 20


15.Hydroxyurea (hydrea)
16. Jak - 2 Kinase
17. Myeloid Progenitor
18.Endogenous Erythroid Colonies (eec)
19 .Hematopoietic
20. Erythropoietin
Reading part B– answer key

1 Statistics showing the curtailment ratio of the affected people.


2.Conditions which might arise after splenectomy.
3. Rescue Protocol
4. Higher among people who are 20+ years.
5 .Albumin (cut-off): Alb>3.5; 2.5 ≤ Alb < 3.5; Alb<2.5; for scores 1, 2, 3 respectively.
6 .Functioning of the ultrasound.

Reading test - part C – answer key

Text 1 - Answer key 7 – 14


7. disease that can be prevented
8. Obviated
9. Annual cases of anaplasmosis in the US.
10. Cases of anaplasmosis became known only after 2000
11. Ticks are present throughout the US
12. Immune-compromised individuals
13. Blood transfusion and organ transplant risks associated with anaplasma species
14. Why tests for anaplasmosis appear negative

Text 2 - Answer key 15 – 22


15. All of the above
16. Common among people suffering from AIDS.
17. grow expeditiously in the esophagus.
18. To show
19. Risk and prevention.
20. Less prevalent among adults.
21. Not regular
22. Treatment and outcome.
READING TEST 14
READING SUB-TEST : PART A
 Look at the four texts, A-D, in the separate Text Booklet.
 For each question, 1-20, look through the texts, A-D, to find the relevant information.
 Write your answers on the spaces provided in this Question Paper.
 Answer all the questions within the 15-minute time limit.
 Your answers should be correctly spelt.
TEXT BOOKLET - CYTOKINES

Text A

Cytokines

Cytokines are messengers secreted by various cell types in the body in response to
a wide variety of physiological stimulus. Cytokines aid in normal physiological processes
such as growth, differentiation, hematopoiesis, as well as several inflammatory and
immune responses. The cytokines as present in minimal levels in hepatic circulation
during normal physiological status and are necessary for hepatic homeostasis. However,
the cytokines have been observed to play an active role in mediating the inflammatory
progression of NAFLD as characterized by apoptotic and necrotic lesion in liver leading to
fibrosis. The cytokines involved in hepatic inflammation, are categorized under several
subfamilies – Tumor necrosis factor-α (TNF-α), Tumor growth factor β (TGF-β),
Interleukins and chemokines.
Text B

Tumor Necrosis Factor α


TNF-α is a pro-inflammatory cytokine secreted by several cell types such as neutrophils,
macrophages, T and B lymphocytes, endothelial cells, mast cells, fibroblasts etc. In liver,
hepatocytes and Kupffer cells are the major contributors of TNF- α. TNF-α plays a central
role in the initiation of inflammatory cascade and its progression from steatosis to
steatohepatitis. Experiments with mice models for obesity have shown the importance of
TNF-α in NAFLD, where anti- TNF-α drug therapy showed promising results. Increased
free fatty acid level in obesity stimulates the hepatocytes to secrete TNF induced
expression of inflammatory genes. A positive correlation has been observed between
TNF-α level in serum and degree of fibrosis in patients with NAFLD. A study with
pentoxifylline an inhibitor of TNF-α has shown suppressive effect on elevation of serum
transaminases and triglycerides in experimental NAFLD induced rats. The study also
demonstrated that NAFLD induced TNF-α expression stimulates endoplasmic reticulum
stress which further mediation the progression of steatosis to fibrosis. -Jun N-terminal
Kinase 1 (JNK1) a stress activated protein kinase is activated by TNF-α which leads to
initiation of an autocrine/paracrine loop resulting in enhanced TNF- α production in liver.
TNF- αα activates inhibitory kappa b kinase β (IKKβ) which resulting in the translocation
of NFκB into nucleus
Text C

Transforming growth factor β


TGF-β is a cytokine secreted by hepatocytes and Kupffer cells in response to degradation
changes in liver. TGF β activates the resting stellate cell by transforming them into active
myofibroblasts, which secrete extracellular matrix protein to initiate the fibrosis process.
Earlier studies have shown up regulated expression of TGF-β following experimentally
induced hepatic damage such as in CCl4 poisoning. Elevated TGF have shown in patients
with liver fibrosis. Earlier studies have shown that TGF marker for the progression of
steatohepatitis Hence detection of TGF -β level would be helpful in marking NASH stage
of NAFLD. Studies have observed polymorphisms in TGF-β1 gene in obese NAFLD patients
with advanced stages of hepatic fibrosis. Xiao and Ho have recently reported that
administration of Epigallocatechin gallate (EGCG) reduced hepatic severity in NAFLD by
suppression of TGF/SMAD pathway. Studies with TGF mice showed protective effect
against experimentally induced NAFLD with methionine and choline deficient (MCD) diet
which was mediated through smad2 activation.
Text D
Interleukins
Interleukin-6 is a proinflammatory cytokine which has been implicated in metabolic
syndrome. IL-6 also plays several other functions such as inducer of immune response,
hematopoiesis and oncogenesis. Certain studies have reported IL-6 to be antihepatotoxic
and mediate hepatic regeneration after partial hepatectomy in mice. IL-6 has also been
considered to reduce hepatic oxidative stress and to curtail mitochondrial dysfunction
role of IL-6. However, studies have reported the role of IL 6 as an acute phase
inflammatory mediator leading to secretion of inflammatory serum proteins. Hence, the
possibility of its role in pathogenesis of NAFLD cannot be ignored. A positive correlation
has been found in patients with NASH and circulating IL 6 knockout mice models have
level shown reduced severity when subjected to experimental NAFLD. Yamaguchi et al,
has shown that inhibition of IL-6 receptor with Tocilizumab enhanced hepatic steatosis
but protected against extensive hepatic damage in MCD diet induced NASH. Recently,
Hamirani et al, have observed a positive correlation between C reactive protein (CRP) and
IL6 levels and increased liver fat accumulation in NAFLD patients as verified with CT
PART A -QUESTIONS AND ANSWER SHEET
Questions 1-7
For each question, 1-7, decide which text (A, B, C or D) the information comes from.
You may use any letter more than once.

In which text can you find information about;


1. Biological cell that synthesizes the extract collagen. Answer _____________

2. Arranged in a radiating pattern like that of a star. Answer _____________

3 .Production or formation, development, and differentiation of blood cells. Answer ____________

4 .Evokes a specific functional reaction in an organ or tissue. Answer ______________

5. Hepatoprotection in most of the cases d. Answer ______________

6. Impairment of the normal processes of synthesis and elimination of triglyceride fat. Answer __

7. The tendency towards a relatively stable equilibrium between interdependent elements, especially
as maintained by physiological processes. Answer _____________

Questions 8-14
Answer each of the questions, 8-14, with a word or short phrase from one of the texts.
Each answer may include words, numbers or both. Your answers should be correctly spelt.

8 What plays an active role in mediating the inflammatory progression of NAFLD Answer ___

9 What is the term used to indicate the thickening and scarring of connective tissue? Answer ___

10 What does the TNF-α level in serum indicate in patients with NAFLD? Answer ______________

11 What reduced the hepatic severity in NAFLD patients? Answer ______________

12 What does IL-6 secrete, as an acute phase inflammatory mediator? Answer ______________

13 What is known to be enhancing or contributing more to TNF- α? Answer______________

14 What is the terms used to show the derivative, which decreases the viscosity of blood?___
Questions 15-20

Complete each of the sentences, 15-20, with a word or short phrase from one of the texts.
Each answer may include words, numbers or both. Your answers should be correctly spelt.

15. _____________ relates to a hormone which has effect only in the vicinity of the gland secreting it.

16. It can be said that disturbances in the normal functioning of the ER often lead to cell____ response.

17. ___________ also plays is reported to initiate immune response.

18. ____________ delivered results as expected

19. ______ is characterized by inflammation of the liver with concurrent fat accumulation in liver.

20 . _____ denotes a cell-produced substance that has an effect on the cell by which it is secreted

END OF PART A, THIS QUESTIONS PAPER WILL BE COLLECTED


READING SUB-TEST : PART B
In this part of the test, there are six short extracts relating to the work of health professionals .
For questions 1-6, choose the answer (A, B or C) which you think fits best according to the text. Write
your answers on the separate Answer Sheet
Questions 1-6

1. The given notice talks about;


A. US Death Rate with respect to maternity cases.

B. Scope and importance.

C. How to deal with preventable mortality?

Variety of challenges by Obstetricians and Gynecologists

The biggest challenge lies in managing the care of patients of all ages from puberty
through menopause and beyond; treating both acute and chronic health conditions;
and of course, managing risk through the pregnancy and childbirth process.
The United States has a higher ratio of maternal deaths than at least 40 other countries, even
though it spends more money per capita on maternity care than any other.
The lack of a comprehensive, confidential system of ascertainment of maternal
death designed to record and analyse every maternal death continues to subject
U.S. women to the unnecessary risk of preventable mortality. Maternal deaths
must be reviewed to make motherhood safer.

The status of maternal deaths in the United States was part of a larger report on the
global, regional, and national levels and causes of maternal mortality from 1990-
2015. The findings suggest that only 16 countries will achieve a target of a 75% reduction in
the maternal mortality ratio (or number of maternal deaths per 100,000 live births) by 2017.
2. As per the case study, pick the right statement;
A. The baby was born with the infection.

B. The infection surfaced itself when the child turns 1 year 6 months.

C. The child was normal at the age of 1 year and 6 months.

Case Study

The mother of the other patient was a white 30-year-old housewife. This was her
second pregnancy, it was planned and there had been an ovarian infection during
prenatal and she had been carrying twins, but one fetus had died. Delivery was
vaginal, birth weight was 1,260 g, gestational age was 30 weeks, SNAPE-PE was 0
and sex was male. The child presented petechiae at birth and persistent tachypnea
for several days. At 1 year and 6 months he was still excreting CMV in urine, had
normal neurological development for his corrected age and his sight and hearing
examination findings were normal.
3. Pick the right statement;
A. Improper care during pregnancy can lead to complications.

B. Continous examination is necessary to thwart complications.

C. Major complications will always lead to death.

Complications

Some complications are common to every expecting mother while others can be
specific to limited expecting females only. According to the statistics, it is found
that 90% of the females in some or the other stages of pregnancy have
knowledgeable complications. While some difficulties are easy on the body like
mild nausea and morning sickness but other complications like asthma, diabetes,
thyroid diseases and hypertension necessities to be taken care of under the
excellent guidance of gynecologists and obstetrics. Some of the medical
complications develop to be fatal to the fetus as well as the mother. Women
suffering from other medical problems like HIV, Urinary Tract Infections, Ectopic
Pregnancies and it’s should go for immunization and medical assistance from time
to time to keep the intensity of the complications under control. Some of the
medical complications like Diabetes and Hypertension are so chronic that
continue to persist in the body even after the liberation of the baby. Shunning
smoking, drinking alcohol and maintaining a healthy lifestyle decreases the
chances of having complications in pregnancy.
4. The notice gives information about;
A. Indications of splenectomy.

B. Percentage of disease conditions.

C. Various conditions, which arise due to Splenectomy.

ELECTIVE SPLENECTOMY
ITP : 57 %
Congenital spherocytosis : 12 %
Hemolytic anemia : 10 %
Hodgkin’s disease : 5 %
AIDS related thrombocytopenia : 3 %
Lymphoma : 3 %
Leukemia: 2.5 %
Others (sarcomas, splenic metastases,…)
2. SPLENECTOMY IN EMERGENCY:
Trauma
5. According to Markov model:
A. Screening for NASH in Primary Care shall be simple.

B. Screening for NASH in Diabetic Care or Clinics shall be simple.

C. Use of ultrasound is not fully recommended.

Screening For NAFLD


A recent, cost-effective analysis using a Markov model suggested that screening for NASH in
individuals with diabetes is not cost-effective at present, because of disutility associated with
available treatment. Given that liver biochemistries can be normal in patients with NAFLD,
they may not be sufficiently sensitive to serve as screening tests, whereas liver ultrasound or
TE are potentially more sensitive, but their utility as screening tools is unproven. Some experts
recently have called for “vigilance” for chronic liver disease (CLD) in patients with type 2
diabetes, but not routine screening.
6. Pick the incorrect statement;
A. Bil (cut-off): Bil<1.5; 1.5 ≤ Bil ≤ 2.5; Bil>2.5; scores 1, 2, 3 respectively.

B. Alb (cut-off):Alb>3.5; 2.5 ≤ Alb ≤ 3.5; Alb<2.5; scores 1, 2, 3 respectively.

C. Platelets (cut-off):Plt<100; 100 ≤ Plt ≤ 150; Plt ≥150;scores 1,2,3 respectively
READING SUB-TEST : PART C
In this part of the test, there are two texts about different aspects of healthcare.
For questions 7-22, choose the answer (A, B, C or D) which you think fits best according to the text.
Write your answers on the separate Answer Sheet

Text 1: Brain Imaging Technologies


Brain imaging has greatly advanced in the last 20 years, due to a better understanding of
the electromagnetic spectrum and radiofrequency waves, in relation to protons in indivi
molecules within the cells of the brain. New technologies allow non-invasive spatial
mapping, (morphology), and observations of processes within the brain during set tasks.
sequencing scanned sections of the brain, activity between neurons in different parts of the
brain can be observed and monitored. More recent technologies using a higher frequency
resolution can identify the distribution of individual metabolites (large complex molecules), a
pharmaceutical drugs. There are a number of scanning techniques, their purpose and
limitations are described below.

A computerised tomography (CT) scans use X-rays to show the structure of the brain,
with details such as blood perfusion, (plates a and b); the resultant images are two
dimensional and of comparatively low resolution, however, the quality has been
much improved since 1998. With improved technology, the single section has now
become as multisection and the speed has increased eight times, giving well-defined
3-D pictures. A CT scan may reveal underdeveloped parts of the brain or sites of
injury from impact, tumours, lesions or infection. Before a CT scan, the patient may
drink but is asked not to eat for four hours beforehand, and not to take strenuous
exercise. A CT brain scan, the preferable scanning method by doctors, will take about
30 minutes and the patient must lie still for the duration.

An MRI scanner uses a strong magnetic field and radio waves to create pictures of
the tissues and other structures inside the brain, on a computer. The magnetic field
aligns the protons (positively charged particles) in hydrogen atoms, like tiny magnets. Sh
bursts of radio waves are then sent to knock the protons out of position, and as they
realign, (relaxation time), they emit radio signals which are detected by a receiving device
the scanner. The signals emitted from different tissues vary, and can, therefore, be
distinguished in the computer picture. An MRI scanner can create clear detailed pictures
of the structure of the brain and detect any abnormalities or tumours. Sometimes a dye, or tracer,
such as gadolinium
may be introduced via a vein in the arm, to improve contrast in the image. Images
can be enhanced by differences in the strength of the nuclear magnetic resonance
signal recovered from different locations in the brain.

Functional magnetic resonance imaging (fMRI) can show which part of the brain
is active, or functioning, in response to the patient performing a given task, by
recording the movement of blood flow. All atoms and molecules have magnetic
resonance, emitting tiny radio wave signals with movement, because they contain
protons. Different molecules have different magnetic resonance and two
components of blood are tracked to observe brain activity. Haemoglobin in the
blood carries oxygen; oxyhaemoglobin, around the brain and when it is used up, it
becomes deoxyhaemoglobin. Where the oxygen is being ‘used up’ shows the site
of activity in the brain. The picture is made by monitoring the ratio of the tiny
wave frequencies between these two states whilst the patient carries out a task, e.g.
tapping a finger, which highlights the area of the brain functioning to carry out this task.

Positron emission tomography (PET) scanning produces a three-dimensional image


of functional processes in the brain, (not just the structure). PET is a nuclear
medicine imaging technique, which requires the patient to receive a small injection of
radioactive material (a sugar tracer; fluorodeoxyglucose), into the bloodstream. The
radioactive material causes the production of gamma-rays, these are a form of
electromagnetic radiation like X-rays, but of higher energy. The radioactive material
is transported around the body and into the brain. A ring of detectors
outside the head is used to detect pairs of gamma rays emitted indirectly by the
positron-emitting radionuclide (tracer), in each part of the brain under examination.

The single photon emission computed tomography records the signals from
gamma rays, (singly, rather than when the emissions are opposite at 180º), using
two or more synchronised gamma cameras, and the multiple 2-D images are
computed, tomographically reconstructed, to 3-D. A section may be examined
from several angles, but is slightly less clear than a PET image. A SPECT scanner
is less expensive than a PET scanner and uses longer-lived, more easily obtained
radioisotopes.
Text 1: Questions 7-14
7. According to paragraph 1, technology;
A. Has made it easier to detect what is going on in a person brain
B. Helps define how neurons act and multiply
C. Makes the mapping of the brain a very simple task
D. Can help identify how big complex parts are distributed

8. According to paragraph 2, what is true about CT?


A. It works on the principles of X-rays
B. It provides two-dimensional images
C. It can help identify which parts of the brain are developed and which are not
D. It will always just take half an hour

9. The word eitty-bitty in paragraph 3 may means;


A. Effective
B. Gigantic
C. Small
D. Strong

10. According to paragraph 3, what is not true about an MRI?


A. During the scan, the tissues produce different signals and thus easily get identified.
B. Dye may be used to improve the quality of the image produced by the scan.
C. Protons emit radio signals.
D. The receiving device collects the emitted protons.

11. According to paragraph 4, a fMRI scan;


A. Produces more efficient images of the brain than an MRI.
B. Can detect which parts of the brain are active during movement.
C. Obtains the picture by closely analysing the frequencies of the emitted waves.
D. Not given
12. According to paragraph 5, a PET is;

A. An advanced technique like an MRI.


B. A non-magnetic technique for the detection of brain functioning.
C. A technique in which a patient is given a radioactive material injection.
D. Much more powerful than other techniques.

13. According to paragraph 6, what is true about a SPECT?

A. It produces quality images like an MRI


B. It is similar to a PET in imaging
C. It relies on gamma rays for detection
D. Not given

14. According to the information provided, which technique is considered the best of all by doctors?

A. MRI
B. fMRI
C. PET
D. SPECT
Text 2: A safer way to detect heart disease
Researchers have used a specialized type of MRI to detect 88% of cases of
coronary artery disease in a group of patients with chest pain. The results suggest
that the imaging technique can detect heart disease as accurately as conventional
methods, but with much less risk. Coronary artery disease is the most common
form of heart disease and the leading cause of death in the United States. It
occurs when fat and calcium accumulate in the arteries that supply blood to the
heart. Over time, less blood reaches the heart and heart muscle dies. If the plaque
blocks the arteries completely, a heart attack occurs.

Currently, the best way to detect the disease is through a coronary angiography. A
physician threads a tube into the heart, releases a dye, and uses X-ray images to
look for decreased blood flow. But there’s a small risk in this procedure that the
tube will pierce an artery, resulting in bleeding, or else scrape plaque from artery
walls, which, once the chunks of plaque are in the bloodstream, can lead to a heart
attack or stroke. Other, noninvasive tests such as cardiac ultrasounds are less
risky, but not as accurate. Ultrasound images can be poor in patients with other
conditions such as obesity, requiring doctors to resort to invasive tests.
While an MRI allows doctors to image the body using magnets and radio waves,
until recently it could not produce clear images of dynamically voyaging objects,
such as a beating heart. In the past two years, though, stronger magnets, more
powerful computers, and new software have improved MRI scanning. “Recent
developments allow us to acquire images of the heart in motion,” says Ricardo
Cury, director of clinical cardiac MRI at Massachusetts General Hospital in Boston
and leader of the study. Doctors can now watch the heart beating in real-time and
the images are now sharp. “It’s like opening up the heart and looking at it
directly,” says Renato Santos, a cardiologist at Wake Forest University Baptist
Medical Center. “Until recently, MRI was a research tool,” says Santos. “Now it’s
really a clinical tool…ready for prime time.”
Cury combined two cardiac MRI tests to improve the technique’s ability to
diagnose coronary artery disease. In his study, published in the July issue of
Radiology, researchers at MGH, Harvard Medical School, and Beneficencia
Portuguesa Hospital in Sao Paulo, Brazil, examined 46 patients. They began with
an MRI stress test, injecting a harmless dye and medicine that stresses the heart.
As the heart pumped, they used MRI to look for decreased blood flow or evidence
that the heart was working abnormally. Next, they examined still MRI images of
the heart for damaged areas or evidence of prior heart attacks. If patients were
abnormal in one or both tests, the doctors deduced blocked arteries.

Cury’s results reinforce those of an earlier, unrelated study in which researchers


at Duke University used the same techniques to successfully diagnose coronary
artery disease in 100 patients. The results are good news for patients. The MRI
exam is short and painless. By using it to triage people with chest pain but
actually without any disease, physicians might save such patients from
unnecessary invasive procedures. In cases where the heart disease is evident,
MRI can help doctorsdecide what to do next, for instance, whether surgery to
clear or bypass a blocked artery is necessary. After surgery, doctors can use MRI
to monitor arteries for future blockages noninvasively. Cury says that the 12
percent of cases misdiagnosed in his study are less than other noninvasive tests,
and in certain cases artifacts of the study’s design. He adds that MRI’s accuracy
will increase as doctors learn to make better diagnoses from MRI images.
“Obviously 100% is ideal,” says Santos. “I think MRI is going to get us closer to
that than our traditional methods.”
Text 2: Questions 15-22
15. Paragraph 1 talks more about;
A. breakthrough in imaging.
B. How heart diseases can be detected.
C. How new techniques are more efficient than traditional techniques.
D. How a heart attack occurs.

16. According to paragraph 1, a simple cause of a heart attack is;


A. Heart doesn receive enough blood.
B. Fat gets deposited in arteries.
C. Arteries carry less blood to the heart.
D. All of these

17. What risk is mentioned in paragraph 2?

A. The tube may puncture the artery.


B. The tube can cut off plaque.
C. The artery walls may release more plaque into the bloodstream.
D. All of these

18. Paragraph 2 talks more specifically about;

A. Techniques that are risky.


B. Why conventional techniques should not be used.
C. How expensive techniques are.
D. How risky CA can be.

19. According to paragraph 3, what is true about MRI scanning?

A. MRI scans in previous years were less effective.


B. It was not possible to capture motion-based images.
C. MRI scans of the new generation are more effective with power imaging qualities.
D. use of powerful computers and software have made MRI scanning more effective.
20. According to paragraph 3, what is possible with MRI now?

A. It is easy to see the heart, live in action


B. It is easy to track heart beating
C. It is easy to get more quality-based images
D. All of these

21. According to paragraph 4, which one of the following statements is true?

A. Cury led a team of doctors and researchers at MGH.


B. Cury detected blocked arteries.
C. The team of doctors compared 46 case studies.
D. Researchers began with the stress testing.

22. In paragraph 5, the word triage may mean;

A. To help
B. To examine
C. To show
D. To provide comfort

END OF READING TEST, THIS BOOKLET WILL BE COLLECTED


Reading test 14: Answer Key

Part A - Answer key 1 – 7


1: B
2: C
3: A
4: A
5: D
6: D
7: A

Part A - Answer key 8 – 14


8: cytokines
9: fibrosis
10: degree of fibrosis
11: Epigallocatechin gallate
12: inflammatory serum proteins
13: hepatocytes and Kupffer cells
14: Pentoxifylline

Part A - Answer key 15 – 20


15: Paracrine
16: Stress
17: Il-6
18: Tnf-± drug Therapy
19: Steatohepatitis
20: Autocrine
Reading test - part B – answer key
1. Is: Scope and importance.
2. The baby was born with the infection.
3. Continous examination is necessary to thwart complications.
4. Indications of splenectomy.
5. Screening for NASH in Diabetic Care or Clinics shall be simple.
6. Platelets (cut-off):Plt<100; 100 ≤ Plt ≤ 150; Plt ≥150; scores 1, 2, 3

Reading test - part C – answer key

Text 1 - Answer key 8 – 14


7: Can help identify how big complex parts are distributed
8: It can help identify which parts of the brain are developed and which are not
9: Small
10 Is: The receiving device collects the emitted protons.
11: Can detect which parts of the brain are active during movement.
12: A technique in which a patient is given a radioactive material injection.
13: It relies on gamma rays for detection
14: MRI

Text 2 - Answer key 15 – 22


15: A breakthrough in imaging.
16: All of these
17: The tube may puncture the artery.
18: Techniques that are risky.
19: It was not possible to capture motion-based images.
20: It is easy to get more quality-based images
21: Researchers began with the stress testing.
22: To examine

You might also like